You are on page 1of 259

See discussions, stats, and author profiles for this publication at: https://www.researchgate.

net/publication/315956583

MCQ in general surgery

Book · November 2014

CITATIONS READS
0 3,211

2 authors:

Adel Taha Denewer Islam Hany


Mansoura University Mansoura University
72 PUBLICATIONS   732 CITATIONS    42 PUBLICATIONS   16 CITATIONS   

SEE PROFILE SEE PROFILE

Some of the authors of this publication are also working on these related projects:

bladder cancer View project

Epidemiology and predictors of recurrence of Marjolin’s ulcer: experience from Mansoura University View project

All content following this page was uploaded by Islam Hany on 27 September 2018.

The user has requested enhancement of the downloaded file.


To
Prof. Dr. Khaleel El Etreby

This book is dedicated to his blessed soul ..

Surgical Oncology Unit Members


Preface

Dear Students of the 6th year,

These questions have been collected and selected from the most updated and
recent international books in general and special surgery.

The main format and presentations of the questions have been exclusively
revised to coincide with the recommended topics.

If you have any comments as to the contents or the usefulness of this book, send
them to my e-mail address: adeldenewer@mans.edu.eg

Thanks,

Prof. Dr. Adel Denewar


Head of Surgery Department,

Mansoura University
1

Vascular
1. About Claudication which of the following statements are true?

a. Intermittent claudication may be present at rest.


b. Intermittent claudication is commonly relieved by getting out of bed.
c. Intermittent claudication is most commonly felt in the calf.
d. Intermittent claudication distance is usually inconsistent on a day-to-day basis for a
given patient.
e. Intermittent claudication is thought to be due to nerve compression in the leg muscle
compartments.

2. About Investigations the following statements are true except?

a. Doppler ultrasound works on the basis of a frequency shift when sound waves hit
moving red blood cells.
b. ABPI means ‘ankle brachial pulsatility index.
c. An ABPI of greater than 0.9 is probably normal.
d. Duplex scans are a combination of wave form analysis and B-mode ultrasound.
e. Carotid surgery is often performed on the basis of duplex scans without further imaging.

3. Drugs which of the following drugs have been shown to improve claudication?

a. Beta-blockers
b. Aspirin
c. Oxpentifylline
d. Simvastatin
e. Prostacyclin.

4. Gangrene; which of the following is not a cause?

a. Buerger’s disease
b. Infection
c. Intra-arterial drug injection
d. Frostbite
e. Deep vein thrombosis insufficiency.

1 2 3 4
C B C E
2

5. About amputation, which of the following are not an indication for major
amputation?

a. Clostridium infection of the lower leg.


b. Severe trauma
c. Neurofibroma
d. Knee flexion contracture
e. Severe rest pain without gangrene.

6. In acute ischaemia pain is more prominent in:


a. At the site of obstruction
b. Most proximal part of the limb
c. Most peripheral part of the limb
d. All of the above

7. Indications of amputation include all the following except:


a. Fixed color changes
b. Absent pulsation
c. Tense calf
d. Bulging anterior leg compartment

8. about embolism, all the following are true except:


a. Young age
b. No collaterals
c. Trophic changes
d. The source of emboli may be undetectable

5 6 7 8
C C B C
3

9. The most urgent aspect in treatment of arterial embolism is:


a. Digitalis
b. Heparin
c. Morphine
d. Diuretics

10. The action of heparin in arterial embolism is to:


a. Deal with cardiac problem
b. Prevent propagation of thrombosis
c. Prevent further embolization
d. All of the above

11. In a traumatized patient showing signs of fracture and acute ischemic:


a. Urgent repair of the affected vessel
b. Urgent ligation of the affected vessels
c. Reduction of the fracture and waiting for return of pulse is the I st step
d. None of the above

12. Arterial embolism may result from:


a. Atherosclerosis
b. Bone fracture
c. Parasites
d. All of the above

9 10 11 12
B D C D
4

13. The most common cause of fat embolism is:


a. Weight gain
b. Weight loss
c. Bone fracture
d. None of the above

14. The most common site of arterial embolism


a. Common carotid artery
b. Brachial artery
c. Femoral artery
d. Popliteal artery

15. About acute arterial thrombosis all the following ore true except:
a. Old age
b. History of chronic ischemia
c. No collateral
d. There may be history of diarrhea

16. Sure signs of arterial injury include all the following except:
a. Signs of ischemia
b. Evidence of adjacent nerve injury
c. Pulsating hematoma
d. Palpable thrill at the site of injury

13 14 15 16
C C C B
5

17. In trauma causing arterial spasm treatment may include:


a. Pointing the artery with papaverine
b. Excision and grafting
c. Dilatation by Fogarty catheter
d. All of the above

18. Management of complete arterial rear may include all the following except
a. Dissection of the artery
b. Cut the minor bronchus
c. Suturing in transverse suture line
d. Saphenous grafting

19. About crush syndrome there will be


a. Alkaline urine
b. Small % of patient develops acute renal failure
c. Small % of the developed renal failure will need dialysis
d. Hypovolemic shock

20. Which of the following regarding varicose veins are true?

a. Varicose veins may cause venousclaudication.


b. Varicose veins may be associated with the fOXC2 gene.
c. The prevalence of varicose veins is twice as common in women than in men.
d. Right leg varicose veins are more common than left leg varicose veins.
e. Handheld Doppler provides an accurate assessment of saphenopopliteal
competence.

17 18 19 20
D C d B
6

21. Which of the following statements regarding lymphoedema are true?

a. Primary lymphoedema occurs in more than 5 per cent or the population.


b. Early treatment is usually successful.
c. Early treatment includes surgical drainage.
d. Fluid is relatively low in protein in lymphoedema.
e. Lymphoedema often involves the muscle compartments.

22 . Which of the following is not a risk factor for lymphoedema?

a. Limb surgery (e.g. varicose vein operation)


b. Obesity
c. Family history
d. A Baker’s cyst
e. Air travel.

23. A 55-year-old woman gives a history of tiredness, aching, and a feeling of heaviness in the
left lower leg for the past 3 months. These symptoms are relieved by leg elevation. She is also
awakened frequently by calf and foot cramping, which is relieved by leg elevation, walking, or
massage. On physical examination there are superficial varicosities, non pitting edema, and a
slightly painful, 2 cm. diameter superficial ulcer 5 cm. above and behind the left medial malleolus.
What is the most appropriate diagnosis?

a. Isolated symptomatic varicose veins.


b. Superficial lymphatic obstruction.
c. Deep venous insufficiency.
d. Arterial insufficiency.
e. Incompetent perforating veins.

24. The best treatment plan for the patient described in the preceding question should
include:

a. Varicose vein ligation and stripping as soon as possible.


b. Ulcer debridement, vein stripping, and skin grafting.
c. Ligation of the medial perforating veins.
d. Transposition of saphenous vein valve.
e. Leg elevation, external support, and ambulation without standing.

21 22 23 24
B D C e
7

25. In patients who develop a documented episode of deep venous thrombosis (DVT) the most
significant long-term sequale is:

a. Claudication.
b. Recurrent foot infection s.
c. Development of stasis ulcer.
d. Pulmonary embolization.
e. Diminished arterial perfusion.

26. A 28-year-old woman developed a painful thrombosis of a superficial varix in the left
upper calf 2 days previously. After spending the 2 days in bed with her leg elevated, she felt
better and the tenderness resolved; however, when out of bed she developed a twinge of
right-sided chest pain when walking and a feeling of heaviness in the calf. Which treatment is
most appropriate?

a. Check for leg swelling, tenderness, and Homan's sign, and obtain a Doppler ultrasound study.
b. Begin antibiotics for a probable secondary bacterial infection.
c. Order emergency venography, and if it is abnormal, begin heparin administration.
d. Begin ambulation and discontinue bed rest that probably caused muscle pain by hyperextension of the
knee.
e. If there is no pain on dorsiflexion of the left foot reassure her, since a negative Homan's sign precludes the
diagnosis of DVT.

27. In a 55-year-old grocery store cashier with an 8-month history of leg edema increasing over the
course of a work day, associated with moderate to severe lower leg bursting pain, the most
appropriate investigative study or studies are:

a. Doppler duplex ultrasound.


b. Brodie-Trendelenburg test.
c. Ascending and descending phlebography.
d. Measurement of ambulatory and resting foot venous pressure.
e. Venous reflux plethysmography.

28. In an otherwise healthy male with previously normal pulmonary and cardiac function,
how much of the pulmonary vascular bed must usually be occluded to produce an unstable
cardiovascular state (shock)?

a. 10%.
b. 20%.
c. 40%.
d. More than 50%.

25 26 27 28
C C A d
8

29. Lytic therapy in pulmonary embolism:

a. Should precede anticoagulation.


b. Can be considered for all patients.
c. Can be considered for hemodynamically unstable patients.
d. Is indicated for the majority of patients with documented pulmonary embolism.

30. The single most important indication for emergency pulmonary embolectomy is:

a. The likelihood of another episode of embolism.


b. The inability to determine whether the problem is acute pulmonary embolism or
acute myocardial infarction.
c. The presence of persistent and in tractable hypotension.
d. Pulmonary emphysema.

31. In prevention of the fat emboli syndrome the primary therapy can be
accomplished by which of the following?

a. Systemic anticoagulation achieving a partial thromboplastin time greater than 50


seconds.
b. Intravenous administration of alcohol.
c. Prophylactic administration of methyl prednisolone.
d. Maintaining a serum albumin value greater than 3 gm. per 100 ml. in the days
immediately following injury.

32. Significant tachypnea and hypoxia follow development of fat emboli


syndrome, and the goal of ventilatory support should be:

a. Keeping the respiratory rate below 30.


b. Preventing respiratory alkalosis.
c. Reversing pulmonary shunting using positive end-expiratory pressure.
d. Maintaining an adequate total volume.

29 30 31 32
C c d c
9

33. Which of the following statements correctly characterizes the healing of


prosthetic arterial grafts in humans?

a. Complete healing occurs within 3 months of graft implantation.


b. Complete healing occurs within 1 year of graft implantation.
c. Prosthetic grafts do not heal completely in humans.
d. Polytetrafluoroethylene (PTFE) grafts heal completely whereas Dacron grafts do
not.
e. Dacron grafts heal completely but PTFE grafts do not.

34. Which of the following con firms the diagnosis of transection of the descending
thoracic aorta?

a. Widened mediastinum.
b. Fractured first rib.
c. Left pleural effusion.
d. Positive aortogram.
e. All of the above.

35. When complications occur after operating on a descending thoracic aorta,


perhaps the most devastating is:

a. Recurrent nerve injury.


b. Bleeding with hemothorax.
c. Paraplegia.
d. Renal insufficiency.

36. The most common risk associated with carotid artery aneurysm is:

a. Thrombosis of the aneurysm.


b. Embolization of mural thrombus.
c. Rupture of the aneurysm.
d. Compression of the hypopharynx.

33 34 35 36
c d C b
10

37. The cause of subclavian arterial aneurysms is most often:

a. Sepsis.
b. A congenital defect.
c. Atherosclerosis.
d. Fibromuscular dysplasia.

38. Of the visceral aneurysms, which is the most common?

a. Celiac.
b. Superior mesenteric.
c. Hepatic.
d. Splenic.

39. Aneurysms of the renal artery are most common:

a. At its origin from the aorta.


b. In the main renal artery or the bifurcation in to th e primary branches.
c. Within the kidney.

40. An aortic abdominal aneurysm was first successfully resected by:

a. Matas.
b. Linton.
c. Dubost.
d. Non e of the above.

37 38 39 40
c d b c
11

41. Evaluation of the natural history of abdominal aortic aneurysms in patients


who are followed without any surgical procedure indicates that:

a. Approximately 20% are alive at the end of 5 years.


b. Seventy-five per cent of patients succumb by the end of the first year.
c. Aortic rupture is quite common in this group, occurring in more than half by the
second year.
d. None of the above.

42. The appropriate treatment in most situations of an aortic abdominal graft that
has become infected is:

a. Intravenous antibiotics and observation for future complication s.


b. Catheter drainage at the site of infection.
c. Replacement of the infected graft with another prosthetic graft.
d. Excision of the entire graft and insertion of ax illobifemoral grafts.

43. In a patient with an abdominal aortic aneurysm and a history of several previous
abdominal procedures for release of dense peritoneal adhesions causing episodes
of intestinal obstruction, consideration should be given to which one of the
following at operation?

a. Cardiopulmonary bypass.
b. An incision from the xiphoid process to the symphysis pubis.
c. Incision in the left flan k with a retroperitoneal approach.
d. An axillobifemoral graft.

44. The incidence of inflammatory aortic abdominal aneurysms with dense periaortic
adhesions and possible involvement of adjacent structures such as the duodenum,
renal vein, and ureter is approximately:

a. 2%.
b. 10%.
c. 25%.

41 42 43 44
a d c b
12

45. Carotid artery occlusive disease most often produces transient ischemic
attacks or stroke by which of the following mechanisms?

a. Reduction of flow to the affected area of the brain through stenotic or occluded
vessels.
b. Embolization of atheromatous debris and/or clot with occlusion of intracranial
branches of the carotid artery.
c. Thrombosis and propagation of the clot into the intracranial branches.
d. All of the above are equally common.

46. Which of the following does not describe intermittent claudication?

a. Is elicited by reproducible amount of exercise.


b. Abates promptly with rest.
c. Is often worse at night.
d. May be an indication for bypass surgery.

47. In terms of long-term graft patency, the best results in the femoral tibial bypass
position have been achieved with:

a. A modified human umbilical cord graft.


b. Polytetrafluoroethylene (PTFE [Gore-Tex]).
c. Saphenous vein allograft.
d. Segments of greater and lesser saphenous and cephalic veins spliced together.

48. Which of the following statements about femoral popliteal bypass grafting is/are
true?

a. Lon g-term graft surveillance by duplex scanning has no effect on graft patency
rates.
b. Graft failure and amputation occur in half the patients within 5 years.
c. If grafting is successful, long-term mortality is improved.
d. Patency rates of 80% to 90% at 1 year currently are expected.

45 46 47 48
B C D d
13

49. Which of the following statements about percutaneous renal artery transluminal angioplasty
(PRTA) are true?

a. Patients with renovascular hypertension are usually cured after successful PRTA.
b. Patients with renovascular hypertension due to atherosclerosis are more likely to benefit
from PRTA than those in whom it is due to fibromuscular dysplasia.
c. PRTA of ostial atherosclerotic lesions is more successful than PRTA of nonostial lesions.
d. PRTA is associated with a higher morbidity and mortality than angioplasty for peripheral
vascular disease.

50. Which of the following statements are true?

a. All arterial injuries are associated with absence of a palpable pulse.


b. Preoperative arteriography is required to diagnose an arterial injury.
c. The presence of Doppler signals indicates that an arterial in jury has not occurred.
d. Patients with critical limb ischemia have paralysis and paresthesia.
e. In all patients with multiple trauma, arterial in juries should be repaired before other injuries
are addressed.

51. A patient presents with a gunshot wound of the mid-neck. Although drunk, he exhibits no
lateralizing neurologic signs. After control of his airway is achieved, he is taken directly to the
operating room for control of hemorrhage. The common carotid artery has a 2-cm. destroyed
segment. There is also a major esophageal injury. The best treatment for this carotid injury is:

a. Vein graft replacement of the common carotid artery.


b. Ligation of the common carotid artery.
c. Ligation of the common carotid artery proximally with a subclavian carotid bypass.
d. Ligation of the common carotid artery with sympathectomy.
e. Prosthetic graft replacement of the common carotid artery.

52. A 35-year-old man involved in a motor vehicle accident presents with a knee
dislocation that is easily reduced. Radiography of the knee shows no fracture. Which of the
following statements about h is treatment are true?

a. If he has normal pulses he can be discharged.


b. If he has normal pulses he requires either close observation or arteriography.
c. If he has absent distal pulses and severe ischemia he should undergo arteriography in the
radiology suite.
d. A popliteal vein injury is best treated with ligation.
e. A popliteal artery injury should be repaired with the ipsilateral saphenous vein if available.

49 50 51 52
D d a b
14

53. A 24-year-old man is involved in an industrial accident in which he sustains a


crushed pelvis. Diagnostic peritoneal lavage is positive. At exploration, a large pelvic
hematoma is found. What is the best treatment?

a. Explore all the major arteries and veins of the pelvis and surgically control the
bleeding if possible.
b. Do not explore the pelvic hematoma. Close the abdomen and apply a MAST suit.
c. Do not explore the pelvic hematoma. Apply a pelvic fixator and send the patient
to radiology f or possible embolization of bleeding pelvic vessels.
d. Use sustained hypotensive anesthesia to try to control bleeding.
e. Open the pelvic hematoma and apply laparotomy pads with topical hemostatic
agents.

54. Which of the following statements about acute arterial occlusion today is/are not
true?

a. Most arterial emboli originate in the heart as a result of underlying cardiac


disease.
b. It can be treated under local anesthesia.
c. It is usually due to atherosclerotic disease.
d. Surgical treatment can usually be avoided if the lesion is diagnosed early.

55. Which of the following statements about lytic agents is/are true?

a. They were first introduced well after the advent of balloon embolectomy.
b. Streptokinase is nonantigenic.
c. Systemic use is the most effective means of delivery.
d. The interval to reperfusion limits their utility in the treatment of advanced
ischemia.

56. Which of the following is not an indication for postoperative heparinization ?

a. Suspected venous thrombosis.


b. Risk of embolism following acute myocardial infarction.
c. Advanced ischemia secondary to acute embolic occlusion.
d. Dissolution of residual thrombus after balloon thromboembolectomy.

53 54 55 56
C d d d
15

57. Which of the following is/are not true of the embolectomy catheter technique?

a. The balloon should be inflated by the same person who withdraws the catheter.
b. Distal exploration should be carried out in all major branches of the affected
extremity.
c. The balloon is designed to dilate as it traverses areas of luminal narrowing.
d. Removal of adherent thrombus requires alternate catheter-based therapy in
addition to balloon exploration.

58. Which of the following is the least reliable indicator of successful thrombectomy?

a. Vigorous back-bleeding after removal of thrombotic material.


b. Arteriographically demonstrated patency of all runoff vessels.
c. Normal distal pulses.
d. Return of normal skin color and temperature.

59. A 21-year-old woman presents with digital color changes in response to cold
stimulation. Physical examination and laboratory data, including an autoimmune
disease screen, are normal. She should be advised that:

a. Her condition is characteristic of vasospastic Raynaud's syndrome and, while


she may be at a slightly higher risk for developing a connective tissue disease in
the future, there is no evidence of one currently.
b. Her problem with her fingers will get progressively worse and she will eventually
lose fingers.
c. She has scleroderma, which will manifest itself at a later date.
d. Her problem is “all in her head.”

60. Obstructive Raynaud's syndrome can be differentiated from vasospastic Raynaud's


syndrome by the:

a. Ice water test.


b. Digital hypothermic challenge test.
c. Antinuclear antibody levels.
d. Digital blood pressure measurement.

57 58 59 60
C a a d
16

61. The most dangerous DVT which requires the longest period of treatment:

a. Calf vein DVT


b. Femoral vein DVT
c. Ileofemoral DVT
d. All of the above

62. Which of the following statements about upper extremity edema is/are true?

a. Lymphedema is more common than venous edema.


b. Signs and symptoms of venous obstruction include edema, distention of
superficial veins, tightness, aching, cyanosis and pain.
c. Distal venous obstructions are more likely than proximal venous obstructions to
cause symptoms in the upper extremity.
d. All patients with symptomatic upper extremity venous thrombosis should receive
fibrinolytic therapy.

63. Which of the following statements about chronic mesenteric ischemia due to
atherosclerosis is/are correct?

a. Postprandial pain in these patients is due to gastric hyperacidity and in most


cases is relieved with H 2 blockers.
b. Men are more often affected than women.
c. Mesenteric endarterectomy is the surgical treatment of choice, since long-term
patency rates are superior to mesenteric bypass.
d. Arteriography is no longer necessary in these cases since noninvasive
diagnosis can be established using duplex ultrasound scanning.
e. Surgical treatment is indicated to prevent intestinal infarction in symptomatic
patients.

64. Which of the following statements about angiotensin II is correct?

a. It is a decapeptide.
b. It is an enzyme found in high concentration in the pulmonary circulation.
c. It is a direct vasoconstrictor and stimulates aldosterone production.
d. It is a vasoconstrictor and inhibits aldosterone secretion.

61 62 63 64
c b e c
17

65. Which of the following statements about atherosclerotic obstruction of the renal arteries
is true?

a. Lesions are usually short.


b. These lesions are usually found in the distal renal arteries, particularly just beyond
branch points.
c. Ostial lesions are best treated with balloon angioplasty.
d. Lesion s of this type are the second most common cause of renal artery stenosis.

66. Which of the following statements about the treatment of renal artery stenosis is/are true?

a. Though a significant cause of hypertension, renal artery stenosis seldom results in loss
of renal function.
b. In patients with medically controlled renovascular hypertension there is no need to
consider revascularization.
c. Balloon angioplasty is more effective in patients with atherosclerotic disease as
compared with those with fibromuscular disease.
d. In patients with severe atherosclerosis of the aorta, bypass from the splenic or hepatic
arteries should be considered.

67. All but one of the following statements is true. Which is not true?

a. Successful clinical repair of injured veins had been affected by the turn of the twentieth
century.
b. Initial large experience in managing injured vein s came from the battlefields of
twentieth-century wars.
c. More than 50% of repaired injured veins thrombose.
d. Phlebography is useful in evaluating variable venous anatomy.
e. Repeated phlebography following attempted venous repair is useful in determining the
success rate.

68. Concerning in-vivo regulation of the anticoagulated state by endothelium, the following
is/are true:

a. Heparin-ATIII inactivates only thrombin


b. Thrombomodulin serves only to bind thrombin
c. Production of von Willebrand factor (VWF) inactivates platelets
d. Endothelial cells can secrete tissue factor

65 66 67 68
a d c d
18

69. Ten years after irradiation of the neck for a tonsillar carcinoma, a 59-year-old
woman is found to have symptomatic carotid artery disease. Arteriogram shows a 70%
irregular stenotic lesion. The following is/are true:

a. Replacement of the artery should be planned due to radiation induced arterial injury
b. The pathology is most likely to be an inflammatory reaction with endothelial
sloughing and thrombosis
c. If atherosclerotic disease is found, the plaque will be no different than nonirradiated
plaques
d. The patient should be managed medically because of the radiation arterial in jury

70. A 23-year-old woman with fever, myalgia and anorexia presents with hypertension
and a cool, ischemic left arm. Angiography shows multiple stenoses of the subclavian and
renal arteries. The following is/are true:

a. Coronary angiography is indicated with high likelihood of finding coronary disease


b. Endarterectomy of the lesions would be preferred to transluminal angioplasty
c. The presentation is more suggestive of Behcet’s disease than Takayasu arteritis
d. Preferred management consists of corticosteroids

71. Concerning the fibrinolytic system the following is/are true:

a. Plasminogen is an a-globulin
b. Fibrin but not fibrinogen is lysed by plasmin
c. The main inhibitor of plasmin is a2-macroglobulin
d. TAP is activated during fibrin bonding to plasminogen

72. An 82-year-old man with a long history of coronary and peripheral vascular disease
presents with an acutely ischemic right lower extremity. The following is/are true:

a. The first step in management should be an arteriogram


b. If intractable congestive heart failure is present, non-operative treatment with
heparin would be appropriate
c. If prolonged ischemia has occurred, reperfusion should be accompanied by sodium
bicarbonate
d. Regardless of the period of ischemia, fasciotomy should be based on the findings
postoperative

69 70 71 72
C D D b
19

73. Two days following coronary angiography and angioplasty, a 47-year-old male diabetic
develops painful blue toes on both feet. The following is/are true:

a. It is very unlikely that there is any connection between the catheterization and the
extremity problem
b. The appropriate treatment is vasodilators and an antiplatelet agent
c. If both superficial femoral arteries are obstructed, the most likely etiology is in-situ
microvascular thrombosis
d. If renal failure or pancreatitis develops, the outlook for long term survival is very poor

74. A 70-year-old man presents with sudden pain and ischemic changes in his left leg.
An arterial embolus is suspected. The following is/are true:

a. The most likely source of an arterial embolus is from intracardiac thrombus on a


previous MI
b. If atrial fibrillation (AF) is present, it is known that chronic AF is less likely to produce
embolism than paroxysmalAF
c. Currently, the most common cause of AF is ischemic rather than rheumatic heart
disease
d. Aspirin is more effective than Coumadin in AF for reducing risk of stroke and
cardiovascular mortality

75. A 67-year-old man with acute popliteal arterial embolism has a negative cardiac echo for
source of the thrombus. The following is/are true:

a. Most likely non -cardiac source is a thoracic aortic aneurysm


b. Embolism is more common from femoral than popliteal arterial aneurysms
c. Emboli from popliteal aneurysms are often clinically silent
d. Embolism is rare from subclavian artery aneurysms

76. Which of the following is/are appropriate candidates for exercise testing?

a. The patient with symptoms of intermittent claudication but normal resting ankle
brachial indices
b. The patient with rest pain, non healing ulcers or gangrene
c. If the resting ankle pressure is below 30–40 mmHg
d. The patient with blue toe syndrome and readily palpable pedal pulses

73 74 75 76
D C c a
20

77. Which of the following statements is/are true regarding the use of duplex
scanning as a means to follow and monitor bypass grafts?

a. Duplex scanning is accurate and cost effective


b. A localized increase in systolic velocity greater than 2 5% compared to adjacent
segments in the graft identifies a diameter reduction of at least 50%
c. Peak systolic velocities should be less than 40 cm/sec throughout the graft
d. Arterial venous fistulas associated with in situ bypass grafts are difficult to detect
with a duplex scanner

78. Which of the following is/are true with respect to hypertension?

a. There is a threshold effect of blood pressure on the risk of cardiovascular


complications
b. The risk of hypertension is essentially con fined to stroke
c. Common antihypertensive regimens may have adverse effects on a patient’s lipid
profile
d. Aggressive blood pressure reduction in patients with ischemic heart disease may
increase mortality and morbidity

79. Which of the following is/are true regarding treatment of diabetes?

a. Strict control with insulin but not oral hypoglycemic agents markedly reduces
the incidence of cardiovascular complications in diabetic persons
b. Vascular complications are directly proportion al to the degree of glycemic
control
c. The effects of diabetes are most marked in individuals with other risk factors
d. The impact of diabetes and cardiovascular risk is relatively uniform

80. Pressure in superficial veins of the leg during standing:


a. 60 mmHg
b. 70 mmHg
c. 80 mmHg
d. 90 mmHg

77 78 79 80
A c c c
21

81. Superficial thrombophelbitis may complicate:


a. Varicose veins
b. Trauma
c. Abscess
d. All of the above

82. About thrombophlebitis migrans:


a. It resolves spontaneously and reappear in another area
b. It is common with Burger's disease
c. It is common with visceral malignancy
d. All of the above

83. About phlegmasia cerulae dolens all the following ore true except:
a. There is massive iliofemoral DVT
b. Severe congestion and cyanosis of lower limb
c. Treated by anticoagulants
d. May lead to venous gangrene

81 82 83
D D C
22

Head & neck


l. The following salivary gland secretes viscid secretion rich in Ca:
a. Parotid.
b. Sublingual.
c. Accessory glands.
d. All of the above.
2. Serous salivary secretion is produced by:
a. Parotid gland.
b. Submandibular gland.
c. Sublingual gland
d. None of the above.
3. Parotid gland is the most common salivary gland affected by the following
pathologies except:
a. Acute bacterial sialadenitis.
b. Salivary stones. ,
c. Pleomorphic adenoma.
d. Adenolymphoma (Warthin'stumor)
e. Mumps.
4. Operoble carcinoma of parotid is best treated by:
a. Radiotherapy alone.
b. Chemotherapy alone.
c. Superficial conservative parotidectomy.
d. Radical parotidectomy with block dissection of LNs.

1) 2) 3) 4)

B A B D
23

5- Which one among the following statements about cystic hygroma is untrue?
a. Affects infants and young children.
b. Occurs chiefly in the neck, axilla and groin.
c. Presents as a large soft fluctuating translucent swelling.
d. is localized to the subcutaneous tissues.
e. May rupture subcutaneously with spontaneous cure.
6-The false statement about ranula is that it:
a. is a bluish cyst in the angle between the tongue and the floor of mouth.
b. Usually lies to one side of the middle line.
c. May extend into the submandibular region.
d. May assume an hour-gloss appearance.
e. is best treated by complete excision.
7. Which statement about salivary calculi is untrue?
a. Are commonest in the parotid gland.
b. May lie in the gland or its duct.
c. Produce recurrent pain and swelling during meals.
d. Contain a high proportion of calcium.
e. May require excision of the affected gland.
8. Concerning pleomorphic adenoma (mixed salivary tumor), which of following
statements is incorrect:
a. is the commonest salivary tumor.
b. Has a very heterogeneous histological structure.
c. is well-encapsulated.
d. Presents as a slow-growing firm swelling just below the lobule of the ear.
e. Tends to recur after inadequate excision.
5) 6) 7) 8)

D E A C
24

9. Among the following statements about branchial fistula, the incorrect one is
that it
a. Commonly results from rupture of a branchial cyst.
b. Usually at the lower third of the anterior border of the sternomastoid muscle.
c. is often bilateral.
d. Discharges clear mucoid fluid.
e, Requires removal of the whole track by the "step-ladder' operation.
10. True statements about Ludwig’s angina do not include that it:
a. is a virulent cellulitis of the floor of the mouth.
b. Usually results from infection with staphylococci.
c. Causes marked swelling in the submandibular region with severe edema of the
tongue.
d. May cause suffocation.
e. May require urgent operation.
11. Langenbeek’s repair:
a- Is a method of grafting in burns
b- Is a method of repair after surgical excision of SCC
c- Is a method of repair of cleft lip
d- Is a method of repair of cleft palate
12. About cleft upper lip all are correct except:
o- Median is the most common
b- Lateral is due to non-fusion of median nasal process (future frenulum) with
maxillary process
c- Lateral is more common on left side
d- Best time for repair is the earliest possible time (3 months)
e- Repair is mainly for cosmetic purposes
9) 10) 11) 12)

A B D A
25

13. About cleft palate all are correct except:


a- Usually associated with other congenital anomalies
b- May range from cleft uvulaup to tripartite deformity
c- Causes nasal regurgitation, nasal deformity
d- Best time for repair is 5 years old
14. All the followings are presentations of cancer tongue:
a- Asymptomatic
b- Referred otalgia
c- Fissured tongue
d- Halitosis
e- Al of the above.
15. All the followings are predisposing factors for cancer tongue except:
a- Cigarette smoking
b- Spicy food
c- Septic tooth
d- Sjogren syndrome
e- Syphilis
16. In unilateral hare-lip the following statements are true except that it:
a. Affects the upper lip only.
b. May be partial or complete.
c. Is due to failure to fusion between the maxillary process with the frontonasal
process and the maxillary process of the opposite side.
d. Is always associated with nasal deformity.
e. Produces no serious speech defects.

13) 14) 15) 16)

D E D E
26

17. The wrong statement about carcinoma of the lip is that it:
a. Affects males much more often than females.
b. Usually occurs on the lower lip.
c. May produce a "Kissing cancer on the other lip,
d. is most often a well-differentiated squamous cell carcinoma.
e. Spreads to the regional glands by lymphatic permeation.
18. A completely excised skin lesion of the face proved to be a basal cell
carcinoma. The further management of the case should be:
a. Lymph node dissection.
b. Radiotherapy.
c. Chemotherapy.
d. Regular follow up.
e. Reassurance of the patient.
19. Dental ulcer is characterized by the following features except that it:
a. Occurs in relation to a carious or irregular tooth.
b. Is usually painful.
c. Has an elongated shallow floor and a sloping edge.
d. Never causes enlargement of the regional lymph nodes.
e. May closely simulate malignant ulcer.
20. Carcinoma of the tongue infiltrating the mandible is best treated by:
a. Radiotherapy of both primary and regional gland.
b. Two-stage excision of primary and regional glands.
c. On block excision of primary and whole lymphatic area (Commando operation).
d. Radiotherapy for primary followed by radical neck dissection.
e. Excision of primary and radiotherapy to cervical lymph nodes.

17) 18) 19) 20)

B D D C
27

21. Concerning complete cleft palate, the untrue statement is that it:
a. is due to failure fusion of the palatal shelves of the maxillary processes with each
other and with the frontonasal process.
b. is often associated with cleft-lip and broadening of the face.
c. interferes with nutrition and speech.
d. Predisposes to upper respiratory tract infections.
e. Requires surgical repair after the second year of life.
22. The most frequent fracture of the face involves the:
a. Zygoma.
b. Moxilla.
c. Orbital floor.
d. Nasal bones.
e. Mandible.
23. Concerning fibrous epulis the incorrect statement is that it:
a. is a soft fibroma of the mucoperiosteum around a carious tooth.
b. Forms a small pedunculated swelling arising between two incisor teeth.
c. is covered by intact epithelium.
d. May be highly vascular or fibrous.
e. is treated by local excision.
24. Correct statements about dentigerous cyst include the following except that
it:
a. Occurs in children and adolscents in relation to a missing tooth.
b. is more common in the upper than in the lower jaw.
c. Presents as a globular swelling expanding the jaw.
d. Is lined with squamous epithelium.
21) 22) 23) 24)

E E E B
28

25. Which statement most accurately describes the anatomy of the sublingual glands?
a. They drain through the sublingual duct which opens into the floor of mouth.
b. They drain either directly on to the floor of mouth or into the submandibular duct.
c. They consist of two lobes separated by the mylohyoid muscle.
d. They are embedded in the intrinsic muscles of the ventral surface of the tongue.
e. They lie in the space between the mandible and the two bellies of digastric.
26. The term plunging ranula refers to which clinical entity?
a. A malignant congenital salivary mass arising from the submandibular gland
b. A benign salivary mass involving the parotid and submandibular glands
c. A mucous retention cyst originating from the sublingual glands, limited by the mylohyoid
muscle
d. A mucous retention cyst originating from the submandibular and sublingual glands which
perforates the mylohyoid muscle to enter the neck
e. A midline neck mass which moves on tongue protrusion.

27. Which of the following structures is not an anatomical relation to the submandibular
salivary gland?
a. The anterior facial vein
b. The facial artery
c. The inferior alveolar nerve
d. The lingual nerve
e. The hypoglossal nerve.
28. Which structure marks the posterior boundary of the submandibular duct which can
safely be accessed via an intra oral approach?
a. The third molar tooth
b.The body of the submandibular duct
c. The lingual nerve
d. The posterior edge of the mylohyoid
e. The marginal mandibular nerve
25) 26) 27) 28)
B D C C
29

29. Which structure attaches the deep lobe of the submandibular gland to the
lingual nerve?
a. The hypoglossal nerve
b.The submandibular ganglion
c. The deep cervical fascia
d. The tendon of digastric
e.The mylohyoid.
30. Which of the following is not a complication of submandibular gland
excision?
a. Frey’s syndrome
b. Anaesthesia of the ipsilateral tongue
c. Weakness of the corner of the mouth
d. Anaesthesia of submental skin
e. Paralysis of the ipsilateral tongue.
31. What percentage of submandibular tumors is malignant?
a. 20 per cent
b. 30 per cent
c. 40 per cent
d. 50 per cent
e. 60 per cent.
32. Which of the following is not a feature of salivary malignancy?
a. Facial nerve weakness
b. Rapid enlargement
c. Induration of the overlying skin
d. Cervical node enlargement
e. Rubbery consistency.
29) 30) 30) 31)
B A D E
30

33. Which is the most appropriate form of biopsy for a major salivary gland
tumour?
a. Open surgical biopsy to allow histology
b. Salivary washings
c. Frozen section during formal excision
d. Fine-needle aspiration cytology (FNAC)
e. Biopsy is contraindicated.
34. Which of the following structures does not lie in the parotid gland?
a. The facial nerve
b. Terminal branches of the external carotid
c. The glossopharyngeal nerve
d. The retromandibular vein
e. Lymph nodes.
35. What is the most common site for a parotid tumour?
a. At the anterior border of the masseter
b. Inferior to the angle of the mandible
c. As a parapharyngeal mass
d. Anterior to the ear
e. Behind the angle of the mandible.
36. Which nerve must be transacted as part of a superficial parotidectomy?
a. The facial nerve
b. The hypoglossal nerve
c. The greater auricular nerve
d. The accessory nerve
e. The auriculotemporal nerve.

33) 34) 35) 36)


D C E C
31

37. Which of the following landmarks is used to locate the facial nerve trunk?
a. The insertion of sternomastoid
b. The greater horn of the hyoid
c. The superior-most portion of the cartilaginous ear canal
d. The insertion of digastric
e. The insertion of masseter.

38. Which of the following is not a branch of the facial nerve?


a. Temporal
b. Oribtal
c. Zygomatic
d. Buccal
e. Cervical.
39. Which of the following branches of the facial nerve can be divided without
the need for immediate cable graft repair?
a. Temporal
b. Oribtal
c. Zygomatic
d. Buccal
e. Cervical.
40. What is Frey’s syndrome following parotidectomy?
a. Gustatory sweating
b. Dry mouth due to reduction in salivary flow
c. Development of a sialocele over the parotid bed
d. Cosmetic deformity due to loss of parotid bulk
e. Hyperplasia of the contralateral parotid gland.

37) 38) 39) 40)


D B D A
32

41. With which malignancy is Sjögren’s syndrome associated?


a. Acinic cell carcinoma
b. Adenoid cystic carcinoma
c. Carcinoma ex pleomorphic adenoma
d. Salivary sarcoma
e. Lymphoma.
42. The commonest discrete lump of the parotid gland is:
a.Pleomorphic adenoma
b.Mucoepidermoid carcinoma
c. Mumps
d.Parotid gland stone
e.Tuberculosis of the parotid lymph node
43. About submandibular salivary stones, all the following statements are true,
except:
a.The majority are radio-opaque
b.A stone can be detected by CT scan
c.A stone may present by a submandibular swelling that increases in size by eating
d.A submandibular salivary swelling can be rolled over the angle of the mandible
e.A stone in the gland is treated by submandibular sialadenectomy
44. About pleomorphic adenoma of the parotid gland, all the following
statements are true, except:
a.It has epithelial and mesenchymal components
b.The tumour is painless
c.It has an incomplete capsule
d.The tumour is usually present deep to the facial nerve
e. It may turn malignant
41 42 43 44
E A D D
33

45. About adenolymphoma, all the following statements are true, except:
a.The tumour is related to smoking
b.It may be bilateral
c.It has epithelial and lymphoid components
e.The tumour is cystic
e.Facial nerve palsy is characteristic of this tumour
46. All the following are possible complications of parotidectomy, except:
a.Loss of sensation on one side of the face
b.Inability to close the eyelids
c.salivary fistula
d.sweating on the side of the when eating(gustatory sweating)
e.Immobility of one angle of the mouth when patient smiles
47. About Ludwig’s angina, all the following statements are true, except:
a. This is tuberculous infection of the deep neck spaces
b.Diabetics are more prone to develop this condition
c.It causes laryngeal oedema
d. Oedema of the floor of mouth pushes the tongue upwards
e.Urgent incision and drainage is needed
48. A patient is diagnosed to have malignant metastasis in right upper deep
cervical lymph nodes. There are no symptoms to indicate the site of the primary.
The possible primary lesion may be any of the following, except
a. Nasopharyngeal carcinoma
b. Carcinoma of maxillary antrum
c. Papillary carcinoma of thyroid
d. Carcinoma of pyriform fossa of hypopharynx
e. Laryngeal carcinoma

45 46 47 48
E A A E
34

49. The commonest swellings in the carotid triangle are:


a.Thyroid nodules
b. Lymphadenopathy
c. Carotid body tumour
d.Branchial cyst
e.Lipoma
50. About cold abscess of the neck, all the following statements are true, except:
a.It is usually caused by caseation of tuberculous lymphadenitis
b. The abscess may have two components, one superficial, and one deep to the deep
fascia
c.The condition is mildly painful with low-grade fever
d. Anti-tuberculous treatment is an essential part of treatment
e. Drainage is achieved by incision
51. About branchial cyst and fistula, all are true, except:
a.A branchial cyst is observed in the neonate
b.A branchial fistula is present at birth
c.A branchial cyst is partially covered by the sternomastoid muscle
d.The main differential diagnosis of branchial cyst is cold abscess
e.A branchial cyst contains cholesterol crystals
52. About thoracic outlet syndrome, all statements are true, except:
a.In all cases a cervical rib is present
B.The commonest manifestations are caused by compression of the lower trunk of
the brachial plexus
c. There may be hyposthesia of medial side of forearm
d.It is one cause of Raynaud,s phenomenon
e.There may be chronic heaviness and oedema of the upper limb

49 50 51 52
B E A A
35

53. About tracheostomy, all the following statements are true, except:
a. Tracheostomy is indicated if mechanical ventilation is needed for a long time
b. Emergency tracheostomy is done at the scene of an accident if there is upper
airway obstruction
c. Percutaneous tracheostomy is suitable for critically-ill ICU patients
d. In adults, an ellipse is removed from the anterior wall of 2nd, 3rd and 4th tracheal
rings
e.One of its possible complications is subglottic stenosis

54. Midline cystic swelling in the neck, moves up &down with deglutition and
with protrusion of the tongue:
a) Thyroid cancer.
b) Branchial cyst.
c) Ranula.
d) Thyroglossal cyst.
55. The commonest site for lip cancer is :
a) Angle of the mouth.
b) Lower lip at junction of middle, outer thirds.
c) Upper lip central part.
d) Upper lip at junction of middle, outer thirds.

56. The anterior 2/3 of the tongue receives common sensation through
…………nerve:

a) Glossopharyngeal.
b) Chorda tympani of facial.
c) Lingual.
d) Hypoglossal.

53 54 55 56
B B B C
36

57. All of the following is true about ameloblastoma (previously called


adamantinoma) except………….:

a) It is located near symphsis menti usually.


b) It originates from Mallasez cells.
c) It grows both in horizontal and vertical rami.
d) It has equal lobulations.

58. Salivary gland stones are commonest in………..glands:

a) Submandibular.
b) Parotid.
c) Sublingual.
d) Minor.

59. All of the following is correct about salivary glands’ stones except….:
a) Neck US may be a useful investigation.
b) 90% of them are radiopaque.
c) In bidigital examination, they are better felt from inside.
d) Pain may refer to tongue or teeth.

60.All of the following is true about salivary glands’ tumors except:

a) 80% of them occur in parotid gland.


b) 80% of parotid gland tumors are benign.
c) 80% of parotid gland tumors occur in the deep lobe.
d) 80% of benign parotid tumors are pleomorphic adenomas.

57 58 59 60
A A A C
37

61.A 24 years old female presented with a neck swelling on the left side, on
clinical examination it was located in the carotid triangle deep to the
sternomastoid muscle ,mobile from side to side ,it was transopaque on
transillumination, Aspiration revealed mucoid fluid rich in cholesterol crystals,
your provisional diagnosis is………..:

a) Cervical lymph node level III.


b) Carotid body tumor.
c) Cystic hygroma.
d) Branchial cyst.
62-A43-year-old teacher underwent left parotidectomy. Upon awakening from
surgery, paralysis of the left lower lip was observed. This complication was most
likely due to injury to which of the following:
(A) Parotid duct
(B) Facial nerve - temporal branch
(C) Facial nerve - cervical branch
(D) Facial nerve - main trunk
(E) Platysma muscle
63. A9-year-old boy complains of a swelling on the left side of his neck in the
supraclavicular region. The swelling is translucent; a diagnosis of cystic hygroma
is established. What is true of cystic hygroma?
(A) It arises from sweat glands in the neck.
(B) It is usually an anterior midline structure.
(C) It may occur in the mediastinum.
(D) Its lesions are usually easy to enucleate.
(E) It is premalignant.
64-A 62-year-old man undergoes excision of a cylindroma of the submandibular
gland. He is most likely to have an injury to which of the following?
(A) Maxillary branch of the trigeminal nerve
(B) Lingual nerve
(C) Vagus nerve
(D) Floor of the maxilla
(E) Frontozygomatic branch of the facial nerve

61 62 63 64
D c c b
38

65-A 62-year-old alcoholic presents with an indurated ulcer, 1.5 cm in length, in the left
lateral aspect of her tongue (not fixed to the alveolar ridge). There are no clinically abnormal
glands palpable in the neck, and a biopsy of the tongue lesion reveals squamous cell
carcinoma. What should she undergo?
(A) Chemotherapy
(B) Local excision of the ulcer
(C) Wide excision and left radical neck dissection
(D) Antibiotic therapy and should be encouraged to stop smoking
(E) Wide excision of ulcer and radiotherapy
66- A 59-year-old woman has discomfort in the posterior part of her tongue. Abiopsy
confirms that the lesion is a carcinoma. What is true in carcinoma of the posterior third of the
tongue?
(A) Lymphoid tissue is absent.
(B) Lymph gland spread is often encountered.
(C) There is an excellent prognosis.
(D) The tissue is well differentiated.
(E) The recurrent laryngeal nerve is infiltrated.
67-Adenocarcinoma is the predominant malignant lesion in which of the following?
(A) Hard palate
(B) Lip
(C) Anterior two-thirds of the tongue
(D) Larynx
(E) Esophagus
68-A 58-year-old woman undergoes excision biopsy of a tumor in the left posterior
triangle of her neck. Histology suggests that this is a metastatic cancer. What is the most
likely site of the primary tumor?
(A) Ovary
(B) Adrenal gland
(C) Kidney
(D) Piriform fossa
(E) Stomach

65 66 67 68
c b A D
39

69-The classic complete neck dissection for palpable adenopathy in the posterior
triangle of the neck includes removal of which of the following?
(A) The transverse process, C2–C4
(B) The spinal accessory nerve
(C) Both thyroid lobes
(D) The trapezius
(E) The vagus
70. A 40-year-old woman is suspected of having a carotid body tumor. Which
one of the following is most characteristic of such a tumor?
(A) They secrete catecholamines.
(B) They are more common at sea level.
(C) They arise from structures that respond to changes in blood volume.
(D) They arise from the structures that respond to changes in PO2.
(E) They are usually highly malignant.
71. Following surgical resection of a large thyroid mass, a patient complains of
persistent hoarseness and a weak voice. What is the most likely cause of these
symptoms?
(A) Traumatic intubation
(B) Prolonged intubation
(C) Injury to the recurrent laryngeal nerve
(D) Injury to the superior laryngeal nerve
(E) Scar tissue extending to the vocal cords
72. What is the most common site for foreign bodies in the head and neck?
(A) Eye
(B) Ear
(C) Nose
(D) Throat
(E) Esophagus

69 70 71 72
B D C B
40

73-A 33-year-old female noted a discharge from a sinus in the overlying


skin below the right angle of the mandible. She recalls previous episodes of
fullness and mild pain in this region over the past several years. What is the
most likely cause?
A) Thyroglossal duct cyst
(B) Branchial cyst
(C) Teratoma
(D) Myeloma
(E) Trauma to the neck
74-A 5-year-old girl presents with difficulty breathing. On examination, of
the oral cavity a 3-cm mass is found in the midline on the posterior aspect
of the tongue. The most likely diagnosis is:
(A) Lingual tonsil
(B) Lingual thyroid
(C) Foreign body stuck to the tongue
(D) Dermoid
(E) Angioneurotic edema
75-Acute bacterial sialadenitis most commonly affects
a) Parotid
b) Submandibular
c) Submaxillary
d) All equal
76- percent of salivary stone occurr in
a) Parotid
b) Submandibular
c) Submaxillary
d) Minor salivary gland

73 74 75 76
B B A B
41

77-The most common salivary tumor


a) Pleomorphic adenoma
b) Adenolymphoma
c) Epidermoid carcinoma
d) Adenocarcinoma
78-Spot the wrong statement
a) 75% of salivary neoplasms arise in parotid
b) 80 % of parotid tumorsare begnin
c) 80%of begnin parotid tumours are pleomorphic adenoma
d) All are correct
79-warthins tumor refers to
a) Pleomorphic adenoma
b) Adenolymphoma
c) Mucoepidermoid tumour
d) Adenoid cystic carcinoma
80-Malignant parotid tumour is characterized by
a) Pain
b) Hard & fixed mass
c) Paralysis of facial muscles
d) All of the above

77 78 79 80
A D B D
42

81- Salivary carcinoma with poor prognosis is


a) Adenoid cystic carcinoma
b) Adenocarcinoma
c) Squamous carcinoma
d) Mucoepidermoid carcinoma
82-salivary carcinoma with perineural spread to brain is
a) Adenocarcinoma
b) Adenoid cystic carcinoma
c) Squamous carcinoma
d) Acinic cell carcinoma
83-adenolymphomas are common to
a) parotid gland
b) Submandibular gland
c) Minor salivary glands
d) All of the above
84-the most common indication for removal of sublingual salivary gland is
a) Sialadenosis
b) Neoplasm
c) Ranula
d) Lymphoma

81 82 83 84
C B A C
43

85- Parotid duct opens into mouth opposite


a) Upper third molar
b) Upper second molar
c) Behind third molar
d) Near to midline of hard palate
86-the treatment of submandibular calculus lying within the duct is to
a) Dilate the duct
b) Remove the stone by making an opening in the duct
c) Slit open the duct at the papilla
d) Remove the gland
87-Pharyngeal pouch should be suspected in presence of
a) Dysphagia
b) Regurgitation of undigested food
c) Aspiration pneumonitis
d) All of the above
88-regarding the pharyngeal pouch which statement is incorrect
a) It protrudes through killian 's dehiscence
b) It usually turns to the left side of the neck
c) It may be visible in the neck
d) It is twice as common in males as in females

85 86 87 88
B B D D
44

89- branchial cyst commonly arises from remnant of


a) First branchial arch
b) second branchial arch
c) third branchial arch
d) all of the above
90-regarding branchial cyst which statement is incorrect
a) Arise from the second branchial cleft
b) Usually appears between the ages of 20-25 years
c) Protrudes beneath the anterior border of the sternomastoid
d) Is usually lined by squamous epithelium
e) All are correct
91-branchial cyst is best differentiated from cold abscess by
a) Fluctuation
b) Transillumination
c) Contains cholesterol crystals
d) None of the above
92-cystic hygroma includes all except
a) Develops from jugular lymph sacs
b) Brilliantly translucent
c) Occupy the middle third of the neck
d) Enlarge with crying

89 90 91 92
B E C C
45

93-Inappropriate to cystic hygroma


a) Type of cavernous haemangioma
b) It can be earliest swelling of neck to appear in life
c) It can obstruct labour
d) It is brilliantly translucent
94- Treatment of Ludwig angina includes
a) Amoxcicillin plus metronidazole
b) Decompression of both submandibular triangles
c) Tracheostomy
d) All of the above
95-Ranula is a:
a) Cystic swelling in the floor of the month
b) Forked uvula
c) Sublingual thyroid
d) Thyroglossal cyst
96-The potato tumour of the neck is
a) Sternomastoid tumor
b) Carotid body tumour
c) Thyroid tumour
d) Parotid tumour

93 94 95 96
A D A B
46

97-Laryngocele is common to
a) Trumpet blowers
b) Glass blowers
c) Patient of chronic cough
d) All of the above
98-tuberculous cervical lymphadenitis commonly affects
a) Posterior triangle nodes
b) Upper deep cervical nodes
c) lower deep cervical nodes
d) submaxillary nodes
99-of the 800 lymph node of the body lymph nodes in the neck is aroud
a) 100
b) 200
c) 300
d) 400
100-cervical rib may present by which of the followings
a) Numbness in fingers
b) Bruit around clavicle
c) Lump in lower neck
d) All of the above

97 98 99 100
D B C D
47

101-pain in arm due to cervical rib is caused by


a) Compression of t1
b) Compression of c7
c) Muscle ischemia
d) All of the above
102-in radical neck dissection all the following are removed except
a) Accessory nerve
b) Internal jugular vein
c) Carotid artery
d) Submandibular gland
e) Sternomastoid muscle
103-cystic hygroma is a
a) Haemangioma
b) Sebaceous gland tumour
c) Meningioma
d) Lymphangioma
104- Which of the following is brilliantly translucent
a) Sebaceous cyst
b) Dermoid cyst
c) Cystic hygroma
d) branchialcyst

101 102 103 104


C C D C
48

105-What is wrong about Hodgkin lymphoma


a) Better prognosis than NHL
b) Few malignant cells outnumbered by inflammatory cells
c) Characterized by reed Sternberg cells
d) Diagnostic laparotomy is essential
106-most commom site of lymph nodes enlargement in Hodgkin lymphoma is
a) Abdominal
b) cervical
c) Axillary
d) Mediastinal
107-most common presentation of hodjkin lymphoma is
a) Leukocytosis
b) Fever
c) Painless LN enlargement
d) Pruritis
108-symptoms of of hodjkin llymphoma include all except
a) Pruritis
b) Headache
c) Weight loss
d) fever

105 106 107 108


D B C B
49

109-All true regarding adamantinoma exept:


a) X-ray honey comb appearance
b) Pathologically mimic b.c.c
c) Common in females
d) Can cross symphysis menti
110- All true regarding osteoclastoma except:
a) Fluid inside chocolate like
b) X ray appearance soap bubble appearance
c) Common in children
d) Pathologically giant cell tumor
111- All true regarding dentigerous cyst except:
a) An odontome
b) Contain ateeth inside
c) Treatment removal of the teath
d) Affect mandible more than upper jaw
112- Fissural cyst of the mandible:
a) Nasopalatine cyst is an example
b) Acquired by staph infection
c) Treatment by removal of the wall completely
d) Occur in fusion line of face

109 110 111 112


d c d b
50

Hernia
1. Which of the following is not a predisposing factor for a hernia?
a. Chronic obstructive pulmonary disease
b. Obesity
c. Urinary stones
d. Pregnancy
e. Peritoneal dialysis.
2. Which of the following are more common in multiparous women?
a. Indirect inguinal hernia
b. Lumbar hernia
c. Umbilical
d. Direct inguinal hernia
3. Which of the following is not a complication of an inguinal hernia?
a. Irreducibility
b. Inflammation
c. Strangulation
d. Obstruction
e. Bleeding.
4. Which of the following is not true in relation to strangulated
hernias?
a. They present with local and then generalized abdominal pain and
vomiting.
b. A normal hernia can strangulate at any time.
c. This is more common in femoral hernia.
d. They can be reliably excluded in irreducible hernias on clinical
examination.
e. They require urgent surgery.

1 2 3 4
c c e d
51

5. Regarding the anatomy of the inguinal canal, which of the following


are true?
a. In infants the internal and external rings are almost superimposed.
b. The inferior epigastric vessels lie posterior and lateral to the
internal ring.
c. The inguinal canal is about 10 cm long and is directed downwards,
medially and forwards.
d. The internal ring is a U-shaped opening in the external oblique
aponeurosis 1.25 cm above the mid-inguinal point.

6. Which of the following are true about a sliding inguinal hernia?


a. It is common in men.
b. It should be suspected clinically in small hernias confined to the
inguinal canal.
c. It is more common in the young patient.
d. It is impossible to control with a truss, and hence an operation is
indicated.

7. Which of the following statement regarding a femoral hernia are


true?
a. The femoral canal occupies the most lateral component of the
femoral sheath.
b. It can be easily controlled by truss.
c. Strangulation is the initial presentation in 40 per cent of patients.
d. An operation is only occasionally required.

8. Which of the following statements are true regarding umbilical


hernia?
a. Men are affected more frequently than women.
b. Irreducibility is common due to omental adhesions.
c. Mayo’s operation involves mesh repair of the hernia.
d. Infantile umbilical hernias need immediate surgery.
5 6 7 8
a d d c
52

9. Which of the following are true regarding ‘burst abdomens’?


a. The incidence is around 10–15 per cent.
b. Midline and vertical incisions are more likely to burst than
transverse incisions.
c. Catgut is associated with a lower risk of burst abdomen.
d. Most cases are managed conservatively.

10. In incompletely descended testis, which of the following


statements is false?
a. The incidence is 4 per cent.
b. A testis absent from the scrotum after 3 months is unlikely to
descend.
c. An incompletely descended testis tends to atrophy as puberty
approaches.
d. Early orchidopexy can preserve function.
e. Orchidopexy reduces the chances of developing a testicular
tumour.

11- The following statements are true about a hydrocele except


a. A hydrocele is a collection of fluid within the tunica vaginalis.
b. A congenital hydrocele causes an intermittent swelling.
c. An acute hydrocele in a young man may be a sinister finding.
d. Testicular pathology may cause a hydrocele.
e. Drainage is an effective treatment.

12. In testicular tumours, which of the following statements are false?


a. A scrotal lump that is inseparable from the testis is likely to be a
tumour.
b. Lymphatic spread is usually to the inguinal lymph nodes.
c. Teratomas occur in the third decade and seminomas in the fourth
decade.
d. Seminomas usually spread via the lymphatics.
9 10 11 12
b e e b
53

13- The following are True about the inguinal canal EXCEPT:
a. It is a 4 cm tunnel in the lower abdominal muscles.
b. It contains spermatic cord in Meals & Round ligament in female.
c. The floor is formed by the inguinal ligament & the roof is formed
by conjoint tendon.
d. Boundaries include external oblique muscle posterior & fascia
transversalis anteriorly.

14- Following organs can be found within the hernia sac EXCEPT:
a) Colon.
b) Ovary.
c) Urinary Bladder.
d) Prostate.

15- The following is the most common type of hernias:


a) Femoral.
b) Umbilical.
c) Inguinal.
d) Incisional

16- The following are causes of acquired hernia EXCEPT:


a) Cough.
b) Unobliterated processes vaginalis.
c) Pregnancy.
d) Ascites.

13 14 15 16
d d d b
54

17- Richter's hernia is a type of hernia with:


a) Two loop of the bowel in the sac.
b) Portion of the circumference of the intestine in the sac.
c) Portion of the Urinary Bladder in the sac.
d) Omentum in the sac

18- All the following are True when the sac content in omentum
EXCEPT:
a) Doughy in consistency.
b) Gives No gurgling sensation.
c) Dull on percussion.
d) Reduced by difficulty in Beginning.

19- Regarding the external ring, all are true EXCEPT:


a) 1/2 inch above Mid inguinal point.
b) Named also as (superficial ring).
c) Slit in the external oblique aponeurosis
d) It is not involved in the direct hernia
.
20- Injury of the ilio-inguinal nerve during hernia repair can cause:
a) Direct hernia due to paralysis of conjoint tendon.
b) Loss of erectile function.
c) Numbness of the scrotum & medial aspect of the upper thigh.
d) Testicular dragging pain.

17 18 19 20
B D A c
55

21- Boundaries of the Hasselbach's triangle are:


a. Inferior epigastric a. medially & conjoint tendon laterally.
b. Lateral border of rectus muscle medially & inferior epigastric a.
laterally.
c. Ilio-inguinal Nerve laterally & inguinal ligament medially.
d. ASIS laterally & pubic tubercle medially.

22- Indirect (=oblique) inguinal hernia:


a) Directed forward.
b) Occur in old age.
c) Can occur in any age & bilateral in 30 % of cases.
d) Commonly irreducible
.
23-All of the following is True about the clinical picture of the oblique
hernia EXCEPT:
a) The swelling increase in size with cough.
b) Giving gurgling sensation during reduction
c) Can reach the scrotum.
d) Direction of the reduction is "backward"

24- About the internal ring test, all are true EXCEPT:
a) Can differentiate between indirect, direct & femoral hernia.
b) Not usually done.
c) Oblique hernia does not come out except after thumb release.
d) The test starts by asking the patient to reduce hernia.

21 22 23 24
b c d a
56

25- Hernial sac can be identified by all of the following EXCEPT:


a. Opaque.
b. Pearly white in color.
c. Containing the vas difference & spermatic a.
d. Has defined edges.

26- Herniorrhaphy has the following characters EXCEPT:


a) Repair of the hernia by absorbable sutures.
b) Should not be performed under tension.
c) Indication in large hernias & good muscular in elderly.
d) Bassini repair is one of its techniques

27- Regarding the direct hernia, all the following are true EXCEPT:
a) Neck of the sac is Medical to the inferior epigastric a.
b) The sac is within the cord including its contents.
c) Usually accuse in elderly and > 50 % is bilateral.
d) Un-commonly complicated.

28- Any of the following can be a complication of the hernia EXCEPT:


a) Bleeding.
b) Strangulation.
c) Obstruction.
d) Rupture due to massive trauma.

25 26 27 28
C A B a
57

29- Irreducible hernia can be presented by any of the following


EXCEPT:
a. Not tense.
b. Not tender.
c. No sign of I.O.
d. Lost impulse on cough.

30- Strangulation hernia can be presented by:


a) Lost expansible an cough, tenderness tension & irreducibly.
b) Tenderness with intact impulse or cough.
c) No tension
d) Hydrocele of hernia sac.

31- The most common type of hernia liable for strangulation is:
a) Incisional.
b) Para-umbilical.
c) Femoral.
d) Direct.

32- The commonest hernia type that can be seen strangulated is:
a) Incisional
b) Inguinal.
c) Para-umbilical.
d) Femoral.

29 30 31 32
D A C b
58

33- The pathological sequence of the strangulated hernia includes all of


the following except:
a) Impeded venous return.
b) Intestinal Obstruction.
c) Impaired arterial blood flow.
d) Gangrene.

34- During the hernial repair, all the following are "wrong" except:
a. Viable intestine is returned back to the abdomen.
b. The omentum never to be excised.
c. The mesh should be applied to cover the cord.
d. Herniorrhaphy should be performed with good tension.

35- The ONLY cause of last expansile impulses on cough during hernia
examination is:
a) Irreducible hernia.
b) Obstructed hernia.
c) Strangulated hernia.
d) Retrosternal goiter.

36- The synthetic material used in herniopalsty is:


a) Prolene mesh
b) Vicryl mesh
c) Double faced mesh
d) Silk mesh

33 34 35 36
B A C a
59

37- Which of the following is not a predisposing factor for a hernia?


a. Chronic obstructive pulmonary disease
b. Obesity
c. Urinary stones
d. Pregnancy
e. Peritoneal dialysis.

38. Which of the following are more common in multiparous women?


a. Indirect inguinal hernia
b. Lumbar hernia
c. Umbilical
d. Direct inguinal hernia

39. Which of the following is not a complication of an inguinal hernia?


a. Irreducibility
b. Inflammation
c. Strangulation
d. Obstruction
e. Bleeding.

40. Which of the following is not true in relation to strangulated


hernias?
a. They present with local and then generalised abdominal pain and
vomiting.
b. A normal hernia can strangulate at anytime.
c. This is more common in femoral hernia.
d. They can be reliably excluded in irreducible hernias on clinical
examination.
e. They require urgent surgery.

37 38 39 40
C C E D
60

41. Regarding the anatomy of the inguinal canal, which of the following
are true?
a. In infants the internal and external rings are almost superimposed.
b. The inferior epigastric vessels lie posterior and lateral to the internal
ring.
c. The inguinal canal is about 10 cm long and is directed downwards,
medially and forwards.
d. The internal ring is a U-shaped opening in the external oblique
aponeurosis 1.25 cm above the mid-inguinal point.

42. Which of the following is false about a sliding inguinal hernia?


a. It is far more common in men.
b. It should be suspected clinically in small hernias confined to the
inguinal canal.
c. It is impossible to control with a truss, and hence an operation is
indicated.
d. It is unnecessary to excise the sac and attempts to dissect the bowel
wall can be dangerous.

43. Which of the following statements regarding a femoral hernia is


true?
a. It is more common in women.
b. The femoral canal occupies the most lateral component of the
femoral sheath.
c. It can be easily controlled by truss.
d. An operation is only occasionally required.

44. Which of the following statements is true regarding umbilical


hernia?
a. The umbilical hernia in an infant is through the umbilical cicatrix
but is usually paraumbilical in adults.
b. Men are affected more frequently than women.
c. Mayo’s operation involves mesh repair of the hernia.
d. Infantile umbilical hernias need immediate surgery.

41 42 43 44
A B A A
61

45. Which of the following are causes of umbilical discharge?


a. Persistent vitellointestinal duct
b. Persistent urachus
c. Pilonidal sinus
d. Omphalitis
e. all of the above

46. Which of the following is true regarding ‘burst abdomens’?


a. The incidence is around 10–15 per cent.
b. Midline and vertical incisions are more likely to burst than
transverse incisions.
c. Catgut is associated with a lower risk of burst abdomen.
d. Most cases are managed conservatively.

45 46

E B
62

Testis

1. As regard the embryology of testis all are correct except:

a. Develops from the genital ridge.


b. Is embryologically on intraperitoneal structure.
c. Derives its blood supply directly from aorta.
d. Torsion only occurs on top of anomalous testis.

2-The tunica vaginalis corresponds to:


a. Internal oblique.
b. Transversalis fascia.
c. Peritoneum.
d. None of the above.

3. Empty scrotum may be due lo:


a. Testicular agenesis.
b. Fetal testicular torsion.
c. Arrested testis.
d. Retractile testis.
e. All of the above

4-lnversion of testis may be:


a. Anterior.
b. Anterolateral.
c. Lateral.
d. A&C

1 2 3 4
B C E d
63

5-All of the following are causes of unilateral testicular arrest except:


a. Testicular dysgenesis.
b. Low maternal HCG.
c. Malformed inguinal canal.
d. Short testicular artery.

6- Testicular arrest is more:


a. On left side.
b. On right side.
c. Equal on both sides.

7-Risk factors of testicular arrest include:


a. +ve family history.
b. Down syndrome"
c. Teratogenic drugs.
d. Chronic maternal illness.
e. All of the above.

8. Testicular arrest is associated with urinary anomalies in:


a. 5% of cases.
b. 15%of cases.
c. 40% of cases.
d. 50% of cases.

5 6 7 8
B B E b
64

9- Most common site of arrest of testis is:


a. Abdominal cavity.
b. inguinal canal.
c. Pelvis.
d. Superficial inguinal pouch.

10- The following ore complications of arrested testis:


a. Torsion.
b. Trauma.
c. Tumor.
d. All of the above.

11- The most diagnostic investigation in arrested testis is:


a. US.
b. CT.
c. Laparoscopy.
d. Doppler.

12- Testis not found by laparoscopy may be due to:


a. Testicular agenesis.
b. Fetal testicular torsion.
c. Maldescended testis.
d. All of the above.

9 10 11 12
B D C d
65

13- Best time for orchiopexy is:


a. At l st day of life.
b. 6-l5 months.
c. At 5 years.
d. At puberty.

14- Best treatment of retractile testis is:


a. Reassurance.
b. Medical treatment.
c. Orchiopexy.
d. Orchidectomy.

15- Concerning undescended testis, the following statements ore true


except that it:
A . Affects about 1% of all males
b. is commoner on the right than on the left side
c. is bilateral in about 20% of cases
d. May be intro-abdominal or extra-abdominal
e. is rarely associated with inguinal hernia

16. The following statements about ectopic testis ore true except that
it:
a. is probably due to rupture of the scrotal tail of the gubernaculum
b. May lie in the groin or pubic region.
c. Does not develop normally.
d. Has normal long spermatic cord.
e. Can be easily replaced in the scrotum.

13 14 15 16
B A E c
66

17. Best treatment for testicular torsion is:


a. Resuscitation.
b. Early operation to untwist the testis.
c. Orchiopexy for other testis.
d. All of the above in sequence.

18. Regarding choriocarcinoma all are correct except:


a. it is a subtype of teratoma.
b. Rarely gives lung metastasis.
c. Secretes HCG in large amounts.
d. All are true

19. Regarding incidence of testicular neoplasm:


a. Most testicular neoplasms are malignant.
b. Mostly occurs above 60 yeors of age.
c. Most common neoplasm is teratoma.
d. All of the above.

20. The first LN station draining testis is:


a-inguinal.
b-internal iliac.
c- Paraaortic.
d. Supraclavicular.

17 18 19 20
D B A c
67

21- Most common presentation of testicular neoplasm is


a. Severe dull aching pain.
b. Accidentally discovered painless mass.
c. Secondary hydrocele.
d. Bone pain &/or hemoptysis.

22-Radiosensitive testicular neoplasm is:


o. Seminoma
b. Teratoma.
c. None of the above.
d. All of the above

.
23-Precocious puberty is character of:
a. Sertoli cell tumor.
b. Leydig cell tumor.
c. Seminoma.
d. Teratoma.

24. The most malignant testicular tumor is:


a. Seminoma
b. Embryonal carcinoma
c. Choriocarcinoma
d. Terotocarcinoma
e. Teratoma

21 22 23 24
B A B c
68

25-The management of testicular tumors includes the following except:


a. Testicular biopsy
b. Radical orchidectomy
c. Simple orchidectomy
d. Radiotheropy
e. Chemotherapy

26-What is not true of interstitial cell tumors of testis:


a. Leydig cell tumor musculinizes.
b. Sertoli cell tumor feminizes.
c. Prepubertal tumors are from Sertoli cell.
d. Sertoli cell tumors are benign and orchidectomy is curative.

27-Which of the following has earlier pulmonary metastases:


A. Seminoma
b. Teratoma.
c. Choriocarcinomo.
d. Embryonol cell carcinoma.

28-Regarding testicular tumors, the untrue is:


a. seminomas and teratomas ore more common thon non-germ cell
tumors.
b. Seminomas send pulmonary metastasis.
c. They have on increased incidence in undescended testis.
d. a-fetoprotein and B-HCG ore tumor markers.
e. The treatment of choice is radical orchidectomy.

25 26 27 28
A C C b
69

29-Complications of varicocele include the following except:


a. Sub fertility.
b. Secondary hydrocele.
c. Malignancy.
d. Testicular atrophy.

30-lndications of surgical correction of varicocele include the following


except:
o. Severe persistent pain.
b. Thrombophlebitis.
c. Secondary varicocele.
d. All cases should be treated surgically for fear of complications
.
31-The main disadvantage of palomo operation is:
a. Testicular artery injury.
b. High recurrence.
c. Hernia formation.
d. lnjury to vas.

32-The vaginal hydrocele is characterized by all of the following except:


a. Fluctuation is positive
b. Eversion and excision of the tunica is the main line of treatment
c. The swelling is usually found above the neck of the scrotum
d. Aspiration is followed by recurrence

29 30 31 32
C D B c
70

33. Size fluctuation is a character of:


a. lnfantile hydrocele.
b. Congenitol hydrocele.
c. Both A ond B
d. None of the above.

34. In incompletely descended testis, which of the following


statements is false?
a. The incidence is 4 per cent.
b. A testis absent from the scrotum after 3 months is unlikely to
descend.
c. An incompletely descended testis tends to atrophy as puberty
approaches.
d. Early orchidopexy can preserve function.
e. Orchidopexy reduces the chances of developing a testicular tumour.

35. The hazards of incomplete descent of testis include all the


following except:
a. Sterility in bilateral cases
b. Trauma
c. Torsion
d. Increased liability to malignant change in later life
e. Epididymo-orchitis.

36. Which of the following statements are true with regard to


testicular torsion?
a. The typical symptom is sudden agonizing pain in the scrotum.
b. It is most common above 25 years of age.
c. Inversion of the testis and a transverse lie are not acause.
d. Doppler ultrasound (US) scan should be done to confirm the
diagnosis.
e. The emergency operation of fixation of the testis to the tunica
albuginea with non-absorbable sutures should be done on both
sides.

33 34 35 36
B E E e
71

37. The following statements are true about a hydrocele except


a. A hydrocele is a collection of fluid within the tunica vaginalis.
b. A congenital hydrocele causes an intermittent swelling.
c. An acute hydrocele in a young man may be a sinister finding.
d. Testicular pathology may cause a hydrocele.
e. Drainage is an effective treatment.

38. In epididymo-orchitis, which of the following is false?


a. The causes are chlamydia and gonococci as sexually transmitted
infections.
b. The initial symptoms are those of urinary tract infection (UTI).
c. In mumps 18 per cent of men may develop the condition.
d. It is a common postoperative complication after prostatectomy.
e. Treatment is doxycycline for chlamydia or a broad-spectrum
antibiotic.

39. In testicular tumours, which of the following statements is false?


a. A scrotal lump that is inseparable from the testis is likely to be a
tumour.
b. Lymphatic spread is usually to the paraaortic lymph nodes.
c. Teratomas occur in the third decade and seminomas in the fourth
decade.
d. Seminomas usually spread via the lymphatics.

40. Which of the following statements is false with regard to the


management of testicular tumours?
a. Tumour markers are measured and a chest X-ray carried out.
b. Initial surgical treatment is orchidectomy through the groin.
c. Staging is done by CT scan and MRI.
d. Seminomas are radiosensitive.
e. Secondaries from teratoma are always treated by surgery.

37 38 39 40
E D D d
72

Anal Canal
1. Which of the following is false regarding anal canal anatomy?
a. The anorectal ring marks the junction between the rectum and the
anal canal.
b. The puborectalis muscle is concerned with the continence
mechanism.
c. The intersphincteric plane is not of much significance.
d. The internal sphincter is the thickened distal continuation of the
circular muscle layer of the rectum.
e. The dentate line is an important landmark representing the site of
fusion of the proctodaeum and postallantoic gut.

2. Which of the following is false regarding anal glands?


a. These are found in the anal mucosa..
b. They drain via ducts into anal sinuses at the level of the dentate
line.
c. They are widely considered as the potential source of anal sepsis.
d. They secrete mucus, which helps to ease defaecation.

3. Which of the following statements regarding the congenital


anomalies of the anal canal is false?
a. The dorsal part of the cloacal membrane is called the anal
membrane.
b. Imperforate anus is divided into two types – high and low –
depending on the level of rectal termination in relation to the
urinary bladder.
c. Low defects are easier to correct but more prone to constipation.
d. Postanal dermoid usually remains asymptomatic until adult life.
e. A postanal dimple is of no clinical significance.

4. Which of the following statements regarding pilonidal sinus is false?


a. It is usually acquired.
b. The primary sinuses are always in the midline between the
sacrococcygeal joint and the tip of coccyx.
c. Bascom’s procedure involves a midline incision directly over the
sinus cavity.
d. It is more common in women.
1 2 3 4
C A B d
73

5. Which of the following statements about anal incontinence is


false?
a. The most common cause of anal sphincter disruption is obstetric
damage.
b. Anal incontinence cannot occur in the absence of sphincter
disruption.
c. Double overlap repair is the standard technique to repair
discrete muscle disruption.
d. Sacral nerve stimulation works by neurophysiological
modulation of the hindgut by stimulation of the sacral nerve
roots.

6. Which of the following statements regarding anal fissure is false


a. It is most common in the posterior wall.
b. The common finding is a hypotonic anal sphincter.
c. Operation can be avoided in about 50 per cent of patients by the
use of local agents like glyceryl trinitrate (GTN) or Diltiazem.
d. The most important complication after lateral internal
sphincterotomy is incontinence, which may affect up to 30 per
cent of patients.

7. Which of the following statements about haemorrhoids is true?


a. Most haemorrhoids are congenital.
b. Pain is rare in uncomplicated grade 1 haemorrhoids.
c. Piles which remain permanently prolapsed are called grade 3
haemorrhoids.
d. Profuse haemorrhage is never seen.

8. Which of the following statements regarding management of


piles is false?
a. Banding of piles involves application of tight elastic bands onto
the base of the pedicle which cause ischaemic necrosis of the
piles.
b. Cryotherapy and infrared coagulation are also methods of
treating piles.
c. Milligan–Morgan haemorrhoidectomy is a closed technique
where the piles are excised and the mucosa completely closed.
d. Stapled haemorrhoidectomy is a method where a special
stapling gun is used to excise a strip of mucosa and submucosa.
5 6 7 8
B c B c
74

9. Which of the following statements regarding pruritus ani is


false?
a. It is a common problem.
b. Threadworms need to be excluded, especially in young
patients.
c. Skin biopsies may sometimes be needed to confirm diagnosis.
d. Washing the area with soap can be helpful.

10. Which of the following statements regarding perianal


abscesses is true?
a. They present as a painful, throbbing swelling in the perianal
area.
b. They are always associated with an underlying anal fistula.
c. Fistulotomy is advised if a fistula is found at the time of
draining the abscess.
d. Finding Gram-positive organisms on pus culture is associated
with an underlying anal fistula.

11. Which of the following statements about anal fistula is true?


a. It may be found in association with ulcerative colitis.
b. Trans-sphincteric fistulae usually have an external opening
close to the anal verge.
c. Posterior fistulae are more likely to have a curved track and a
horseshoe communication.
d. Fistulotomy is the treatment of choice in trans-sphincteric
fistula involving more than 30 per cent of the external
sphincter.

12. Which of the following statements regarding anal warts is


false?
a. They are associated with HPV infection..
b. The incidence has decreased over the last three decades.
c. Acetic acid is helpful in diagnosis.
d. Treatment options include 25 per cent podophyllin and
surgical excision

9 10 11 12
D A C b
75

13. The risk factors for anal squamous cell carcinoma (SCC)
include the following except:
a. Human papillomavirus (HPV) infection
b. HIV infection
c. Renal transplant
d. Other genital cancers
e. Rectal cancer.

14. Which of the following statements regarding anal cancers is


false?
a. They constitute less than 2 per cent of all bowel cancers.
b. They are usually SCCs.
c. The primary groups of lymph nodes are the iliac nodes in the
pelvis.
d. Pain and bleeding are the most common symptoms.

13 14
E C
76

Biliary, Liver and portal hypertension

1. Kasai operation is for treatment of:


a- lntrahepatic biliary atresia
b- Extrahepatic biliary atresia
c- Caroli's disease
d- Choledochal cyst

2. The most accurate investigation to diagnose cholecystitis is:


a- CT scan
b- lV cholangiography
c- HIDA scan
d- US
e- MRI

3. The best of the following investigations of gall stones is:


a- Oral cholangiography
b- US
c- Plain X-ray
d- ERCP

4. With intraoperative T-tube cholangiography, incidence of missed stones is:


a-0%
b- 5%
c- 20%
d- 30%

1 2 3 4

B C B E
77

5. Without intraoperative T-tube cholangiography incidence of missed stone


is:
a- 1O-2O%
b- 40-50%
c- 70%
d- 80%

6. Which percentage of gall stones is rodio-opaque?


a- 10%
b- 40%
c- 70%
d- 90%

7. The commonest presentation of gall stones is:


a- Asymptomatic
b- Biliary colic
c- Charcot’s triad
d- Jaundice

8. The best treatment of asymptomatic gall stone in diabetics is:


o- Prophylactic cholecystectomy
b- ESWL
c- Follow up
d- None of the above

5 6 7 8

A A A C
78

9.Which of the following statements(s) about gallstone ileus is/are not true?
o- The condition is seen most frequently in women older than 70.
b- Concomitant with the bowel obstruction, air is seen in the biliary tree.
c- The usual fistula underlying the problem is between the gallbladder and the
ileum.
d- When possible, relief of small bowel obstruction should be accompanied by
definitive repair of the fistula since there is of significant incidence of
Recurrence if the fistula is left in place.
e- Ultrasound studies may be of help in identifying gallstone of the obstructing
agent.

10. Gas in the biliary system in a plain X-ray is diagnostic of


a. Choledochoduodenal fistula
b. intestinal obstruction
c. Emphysematous Cholecystitis
d. Viral hepatitis
e. A and C

11. Characters of pain in acute cholangitis are the following except:


a- Starts as colicky pain
b- Becomes dull aching with time
c- Throbbing pain means empyema formation
d- Referred to groin and inner side of thigh

12. As regards empyema of gall bladder all are correct except:


a- is a complication of gall stones.
b- Leads to gall bladder mass.
c- ls an indication for cholecystectomy .
d- Should be treated by cholecystectomy.

9 10 11 12

C E D C
79

13. Which statement about acute non calcular cholecystitis is correct?


a- The disease is often accompanied by or associated with other conditions.
b- The diagnosis is often difficult.
c- The mortality rote is higher than that for acute calculous cholecystitis.
d- The disease has been treated successfully by percutaneous
cholecystectomy.
e- All of the above

14. Which of the following statements about cholangitis is incorrect?


a- Charcot’s triad is always present.
b- Associated biliary tract disease is always present.
c- Chills and fever are due to the presence of bacteria in the bile duct system.
d- The most common cause of cholangitis is choledocholithaisis .

15. T-tube should be removed:


o- Once cholangiography is done to ovoid fibrosis
b- After 4 days of operation if there is no missed stone
c- After of least 10 days of operation whatever the result of cholangiography
d- None of the above

16. As regards treatment of acute cholecystitis all correct except:


a- Cholecystectomy is the best treatment in early cases
b- Morphine is given to decrease pain of anxiety of patient
c- Modified Fowler's position is preferred
d- Failure of conservative treatment indicates cholecystectomy

13 14 15 16

E A C B
80

17. Saint’s triad includes all the following except:


a- Achalasia
b- Hiatus hernia
c- Chronic calcular cholecystitis
d- Diverticular disease of the colon

18. Which of the following lesions are believed to be associated with the
development of carcinoma of the gallbladder?
o- Cholecystoenteric fistula.
b- A calcified gallbladder.
c- Adenoma of the gallbladder.
d- Xanthogranulomatous cholecystitis.
e- All of the above.

19. The preferred treatment for carcinoma of the gallbladder is:


a- Radical resection that includes gallbladder in continuity with the right
hepatic lobe and regional lymph node dissection.
b- Radiation therapy.
c- Chemotherapy.
d- Combined treatment involving surgical therapy, chemotherapy, and
radiation.
e- Palliative treatment.

20. Which of the following is true about post cholecystectomy syndrome?


a- Postoperative stricture is the most common cause of the syndrome.
b- Dyskinesia may be the cause
c- ERCP is the investigation of choice
d- Usually resolves on medical treatment
e- All of the above

17 18 19 20

A E E E
81

21. Indications of alpha feto-protein measuring include:


o. Sudden deterioration of hepatic patient.
b. Focal lesion by abdominal US.
c. Follow up after surgical removal of tumour.
d. All of the above.

22. Mortality from rupture liver is about:


a. 5%.
b. 20%.
c. 4O%.
d. 50%.

23. Treatment of traumatic injury to the liver includes all except:


o. Resection debridement of devitalized tissues
b. Ligation of exposed blood vessels and bile ducts
c. Pocking with gauze.
d. Pringle's manoeuvre
e. Partial hepatectomy

24 .The following is not a common complication(s) of hydatidosis:


a. Anaphylactic shock.
b. Obstructive jaundice.
c. Malignant transformation of surrounding tissue
d. Multi-system spread.

21 22 23 24

D B C C
82

25. The most common presentation of hydatidosis is:


a. Asymptomatic.
b. Obstructive jaundice.
c. Anaphylactic shock.
d. Cough and haemoptysis.

26 .As regard treatment of hydatidosis the best is:


o. Follow up.
b. Enculeation.
c. Removal en block.
d. Segmentectomy.

27. High fever in amoebic hepatitis indicates:


o. Amoebic liver abscess.
b. Systemic spread.
c. Secondary infection in amoebic liver abscess.
d. All of the above.

28. The most common type of bilharzia preportal fibrosis is:


a. Fine.
b. Coarse.
c. Mixed.

25 26 27 28

A B C C
83

29. Stage Ill of hepatic bilharziasis means:


a. Hepatomegaly.
b. Hepatosplenomegaly.
c. Splenomegaly with shrunken liver.
d. None of the above.

30. The most common complication of hepatic hydatid disease is:


a. Jaundice
b. Rupture into peritoneal cavity
c. Suppuration
d. Rupture into biliary channel

31. Rupture of amoebic liver abscess occurs most often ln:


a. Peritoneal cavity
b. Pleural cavity
c. Pericardial cavity
d. Duodenum
e. Colon

32. Premalignant syndrome may present by the following except:


o. Polycythaemia.
b. Fever of unknown aetiology.
c. Hypocalcaemia.
d. Hypertension and diabetes.
e. Haemoptysis.

29 30 31 32

C D B C
84

33. Angiography for detection of hepatoma should be done through:


a. Portal vein.
b. Hepatic artery.
c. Retrograde through hepatic veins.
d. Any of the above.

34. Most common predisposing factor for hepatoma is:


o. Hepatitis B .
b. Hepatitis C.
c. Chronic irritation.
d. Portal hypertension.

35. Tumour marker in fibrolamellar hepatoma is:


o. Alpha feto-protein.
b. Carcino-embryonic Ag.
c. CA 19-9.
d. Carboxythrombin .
e. None of the above.
36. Which of the following statements(s) is/are true about benign lesions of
the liver:
a. Adenomas are true neoplasms with o predisposition for complications and
should usually be resected.
b. Focal nodular hyperplasia (FNH) is a neoplasm related to oral contraceptive
pills and usually requires resection.
c. Hemangiomas are the most common benign lesions of the liver that come to
the surgeon's attention.
d. Nodular regenerative hyperplasia does not usually accompany cirrhosis.

33 34 35 36

B C D A
85

37. Haemangioma in ultrasound is:


a. lsoechoic
b. Hypo echoic
c. Hyper echoic
d. Any of the obove

38. Rupture of on amoebic liver abscess occurs most often into the:
a. Peritoneal cavity.
b. Pleural cavity.
c. Pericardial cavity.
d. Duodenum.
e. Colon.

39. The most common malignant tumours of the Iiver are:


o. Hepatomas.
b. Cholangiomas.
c. Angiosarcomas.
d. Metastatic deposits.
e. Lymphomas.

40 The most appropriate treatment for amoebic Iiver abscess is by:

a. Emetine hydrochloride.
b. Metronidazole.
c. Aspiration.
d. Open drainage.
e. Excision.
.

37 38 39 40

C B D B
86

41. Normal portal venous pressure is:


a. 5-7 mmHg.
b. 8-12 mmHg.
c. l0-15 mmHg.
d. 15-20 mmHg.

42. Which of the following treatments most effectively preserves hepatic


portal perfusion?
a. Distal splenorenal shunt.
b. Conventional splenorenal shunt.
c. Endoscopic sclerotheropy.
d. Side-to-side portocaval shunt.

43. Which of the following complications of portal hypertension often


require surgical intervention (for more than 25% of patients)?
o. Hypersplenism.
b. Varceal haemorrhage.
c. Ascites.
d. Encephalopathy.

44. A serum bilirubin of2-3 mg % conforms to Child's criteria of chronic liver


disease:
a. Class A.
b. class B.
c. class C.
d. All of the above.

41 42 43 44

B A B B
87

45. The wrong statement about gastro-esophogeol bleeding is:


a. Bleeding may often be severe to cause collapse.
b. Endoscopic sclerotherapy con arrest bleeding.
c. Prophylactic sclerotherapy in GE varices obviates chance of bleeding and
prolongs survival.
d. Tamponed by sungestaken tube is effective.

46. Varceal bleeding not responding to drug and sclerotherapy is treated by:
a.Embolisation .
b.Surgical ligation.
c.TIPSS.
d.Liver transplant.

47. Which of the following procedures is associated with least risk of hepatic
encephalopathy?

a.Mesocaval shunt .
b. Proximol splenorenal shunt.
c. Distal splenorenol shunt' (Wonen shunt).
D. Side to side portocaval shunt.

48. A small cirrhotic liver with grossly enlarged caudate lobe demands
exclusion of:
a. Portal vein thrombosis.
b. Budd-Chiari syndrome.
c. Hepatoma .
d. Primary scleroising cholangitis

45 46 47 48

C C C B
88

49 .The most feared complication of Denver shunt is:


a. infection.
b. Malfunction.
c. DC.
d. Rupture.

50. An appropriate initial therapy for diagnosed bleeding oesophageal varices


is:
a. lV vasopressin
b. Endoscopic sclerotheropy .
c. Emergency portocaval shunt .
d. Emergency esophogeol transection
e. Esophogeal balloon tamponed

51. Regarding portal vein thrombosis, the untrue statement is:


o. Occur in patient with thrombocytopenia
b. Occurs after severe appendicitis
c. is a cause of mesenteric bowel ischemia
d. Cause splenomegaly
e. Cause portal hypertension

49 50 51

A A A
89

Thyroid gland

l. As regard embryology of thyroid:


a) Develops from 1st branchial Arch.
b) Develops from 4th branchial Arch.
c) Both A and Above.
d) Develops from 2nd branchial pouch
e) None of the above

2. C-cells of thyroid gland:


a) Develop from ultimobranchial body.
b) Produce calcitonin.
c) Are the origin of medullary carcinoma.
d) All of the above.

3. As regard to T3:
a) Less concentration thon circulating T4.
b) More potent thon T4.
c) Mostly bound to TBG.
d) All of the above.

4. The correct sequence of events for the metabolism of iodine and


synthesis thyroid hormone is:
a) Trapping, organification, coupling, release, oxidation.
b) Oxidation, trapping, coupling, organification, release.
c) Coupling, organification, trapping, oxidation, release.
d) Trapping, coupling, oxidation, release, organification.
e) Trapping, oxidation, organification, coupling. release

1 C
2 D
3 D
4 E
90

5. The daily requirement of iodine is:


a) 50-20 pg.
b) 80-90 ug.
c) 100-125 prg.
d) 15O-l65 ug.
e) None of the above

6. Most diagnostic single investigation for toxic adenoma is:


a) T3-T4.
b) US.
c) Thyroid scan.
d) FNABC.

7. The following statements regarding TSH measuring are true except:


a) lt is increased after total thyroidectomy.
b) Normal TSH is about 5 micro units/liter.
c) lt is the most sensitive test for mild cases.
d) All of the above

8. Worm nodule means:


a) Inactive nodule.
b) Usually cancerous.
c) Active nodule.
d) Toxic adenoma.

5 C
6 C
7 B
8 C
91

9.As regards FNAB all cancel except:


a) Outpatient procedure.
b) Cheap and safe.
c) Requires general anesthesia.
d) Can not differentiate follicular adenoma from carcinoma.

10. Presentations of ectopic thyroid include:


a) Dysarthria.
b) Midline neck swelling.
c) Myxedema if removed by mistake.
d) Any of the above.

11. The most common site of thyroglossal cyst is:


a) Subhyoid.
b) Suprahyoid.
c) Sublingual.
d) At thyroid cartilage.

12. The following ore true regarding thyroglossal cyst except:


a) Usually presents as midline neck swelling.
b) Moy be confused with ectopic thyroid.
c) Best treatment is follow up.
d) Should be excised completely for fear of complications.

9 C
10 D
11 A
12 C
92

13. Preparation of retrosternal goiter for surgery includes:


a) Neomercazole.
b) Lugol’s iodine.
c) Propranolol.
d) A and C.

14. Sporadic goiter may occur due to the following except:


a) Cabbage.
b) Water pollution by excreta.
c) Iodine deficiency.
d) Percolates.

15. Autoimmune manifestations of Grave’s disease include the


following except:
a) Palmar erythema.
b) Clubbing.
c) Pretibial myxedema.
d) Exophthalmos.

16. All of the following ore recognized complications of neomercazole


except:
a) Goiter.
b) Agranulocytosis.
c) Renal failure.
d) Hepatotoxicity.

13 C
14 C
15 A
16 C
93

17. Thyrotoxicosis in children all correct except:


a) Usually goes into spontaneous remission.
b) Medical treatment alone can control the disease.
c) Radioactive iodine is the ideal treatment.
d) Thyroidectomy should be near total to avoid recurrence.

18. Which of the following treatment schedule for diffuse toxic goiter is
true:
a) Over 45yeors: radioactive iodine.
b) Under 45 years: with large goiter: surgery.
c) Under 45 years: with small goiter: antithyroid drugs.
d) None of the above.
e) All of the above.

19. Toxic goiter hos the following signs except:


a) Flapping tremors of the hand
b) Exophthalmos
c) Diarrhea
d) Menstrual irregularities

20. 3 hours post-thyroidectomy, 30 years old woman developed


agitation and difficulty breathing, tachycardia and dry dressing but
inferior cervical swelling. The most appropriate immediate step is:
a) insertion of an oro-tracheal tube
b) Reopening of the cervical wound
c) Estimation of serum calcium level
d) IV morphine

17 C
18 E
19 A
20 B
94

21. On the 1 St postoperative day after total thyroidectomy, patient


complains of tingling of the finger, lips and the serum calcium level of
5.5 mg/dL. The next step should be:
a) Observation only
b) Administration of vitamin D2 or D3 50000-100000 units/ day
c) Administration of vitamin D3 1-2pgldoy
d) Administration of calcium gluconate 3-5g /day, by slow IV drip

22. Hashimoto disease may present by:


a) Thyrotoxicosis.
b) Myxedemo.
c) Goiter.
d) Any of the above.

23. The following type of thyroiditis mimics malignancy:


a) Hashimoto thyroiditis.
b) Riedle thyroiditis.
c) De Quervain's thyroiditis.
d) All of the above.

24. the most common cause of goitrous hypothyroidism in adults is:


a) Graves' disease.
b) Riedel’s thyroiditis.
c) Hashimoto’s disease.
d) De Quervain's thyroiditis.

21 D
22 D
23 B
24 C
95

25. Clinical picture of thyroid carcinoma include:


a) Dyspnea and dysphagia.
b) Referred otology.
c) Horsiness.
d) Lateral aberrant thyroid.
e) All of the above

26. A familial form of medullary thyroid carcinoma (MTC) should be


suspected whenever:
a) The tumor is multifocal.
b) The tumor is bilateral foci of tumor are present in both thyroid
lobes.
c) Pathologic examination of the resected thyroid gland reveals the
presence of C-cell hyperplasia in areas of the gland adjacent to
foci of MTC.
d) All of the above.

27. A thyroid nodule could be malignant if it shows:


a) rapid growth
b) Pain referred to the ear
c) Hardness
d) Associated hoarseness of voice
e) All of the above

28. Which of the following statements regarding follicular adenoma is


true:
a) It presents clinically as solitary nodule.
b) Distinction between follicular adenoma and carcinoma can only be
made by histological examination.
c) In adenoma there is no invasion of the capsule or of pericapsular
blood vessels.
d) Preferable treatment is lobectomy
e) All of the above.

25 E
26 D
27 E
28 E
96

29. In thyroid carcinoma, mediastinal node involvement is a feature of


which type:
a) Follicular.
b) Anaplastic.
c) Papillary.
d) Medullary.

30. The term lateral aberrant thyroid implies


a) congenital aberrant thyroid tissue lateral to the thyroid
b) metastasis in cervical lymph node from on occult thyroid
carcinoma
c) a metastasis from carcinoma of the larynx
d) a type of bronchial cyst
e) that a loose piece of thyroid has become implanted in a
thyroidectomy scar

31. Hoarseness of voice denotes:


a) Compression of the superior laryngeal nerve
b) infiltration of the recurrent laryngeal nerve
c) infiltration of the superior laryngeal nerve
d) Tracheal compression

32. All are causes of hypercalcemia, except:


a) Metastatic cancer
b) Sarcoidosis
c) Multiple myeloma
d) Vitamin D intoxication
e) Medullary carcinoma of the thyroid

29 D
30 B
31 B
32 E
97

33. A 17 year old girl presented with o 2.5 cm nodule in the right lobe
of the thyroid gland and enlarged three cervical LNs confirmed by US.
FNA cytology revealed malignant cells with vesicular nuclei, the most
probable diagnosis is:
a) Lymphoma
b) Anaplastic carcinoma
c) Follicular carcinoma
d) Papillary carcinoma
e) Medullary carcinoma

34. A 30-yeor old female presents for evolution of a palpable thyroid


nodule Tc99 scan demonstrated a single cold nodule it may be the
following EXCEPT:
a) Carcinoma
b) Non-functioning adenoma
c) Thyroid cyst
d) Colloid nodule
e) Autonomous nodule

35. The most frequent variety of thyroid cancer is:


a) Follicular carcinoma
b) Papillary carcinoma
c) Anaplastic carcinoma
d) Medullary carcinoma

36. Thyroid disease treatment:


a) Lymphoma >( irradiation and chemotherapy)
b) Follicular adenoma> ( lobectomy)
c) Follicular carcinoma> (total thyroidectomy and radioiodine)
d) Follicular carcinoma> (by histopothological surprise completion
thyroidectomy and radioiodine)
e) All of the above

33 D
34 E
35 B
36 E
98

37. Thyroid carcinoma during pregnancy, appropriate treatment is:


a) rodioiodine l3l
b) Chemotherapy
c) Surgery
d) Wait for delivery

38. Recurrent goiter may be due to:


a) Inadequate initial removal.
b) Persistence of etiology.
c) Foreign body reaction.
d) Any of the above

39. Excision of o thyroglosssal cyst should include removal of


a) Thyroid isthmus.
b) Pyramidal lobe.
c) Body of the hyoid bone.
d) Foramen cecum.
e) Remnants of the thyroglossal duct.

40. ln physiological goiter the following statements are true except


that it:
a) Affects males more often thon females.
b) Presents as fullness of the neck (Venus neck).
c) Is characterized by uniform smooth enlargement with fleshy or
firm consistency.
d) Moy be associated with toxic or pressure symptoms.
e) Usually resolves spontaneously.

37 C
38 D
39 C
40 D
99

41. The best routine managment for multinodular goitre is by:


a) Hemithyroidectomy.
b) Partial thyroidectomy.
c) Bilateral wedge resection.
d) Subtotal thyroidectomy.
e) Thyroxin administrotion.

42. Among the following statements about retrosternal goiter, the


false one is that it:
a) Usually arises in aberrant intra-thoracic thyroid tissue.
b) Is particularly common in males.
c) Moy present with symptoms of mediastinal compression
(syndrome).
d) Is often associated with palpable enlargement of the thyroid.
e) Is best removed through a cervical incision.

43. A middle-aged female presented with an asymptomatic nodule in


the right lobe of the thyroid. She gave a history of irradiation in
childhood. The nodule was cold on radioactive iodine scanning and the
sonogram reveled that it was a solid mass. The appropriate
management of this case is:
a) Aspiration biopsy.
b) Treatment with thyroxin.
c) Right lobectomy.
d) Subtotal thyroidectomy.
e) Total thyroidectomy.

44. Voice fatigue after thyroidectomy is due to injury to which of the


following nerves?
a) Superior laryngeal.
b) External laryngeal.
c) Internal laryngeal.
d) Recurrent laryngeal.
e) Vagus.

41 D
42 A
43 C
44 B
100

45. Medical treatment of thyrotoxicosis is least useful in:


a) Cases with true exophthalmos.
b) Pregnant females.
c) Secondary thyrotoxicosis.
d) Uncomplicated thyrocardic potients.
e) Post-operative recurrence.

46. Thyrotoxicosis during pregnancy is best treated by:


a) Subtotal thyroidectomy.
b) Corbimazole.
c) Beta blockers.
d) Lugol’s iodine.
e) Rodioiodine.

47. The following statements about treatment of thyrotoxicosis by


radioactive iodine are fine except that:
a) Is contraindicated in patients below the age of 40.
b) Is particularly useful in elderly and thyrocardiac patients.
c) Produces its beneficial effects within o few days.
d) Moy be followed by myxedemo.
e) Carriers the risk of occurrence of thyroid cancer.

48. Hypothyroidism is most often due to:


a) Multinodulor goitre.
b) Solitary adenoma,
c) Thyroid cancer.
d) Chronic thyroiditis.
e) iotrogenic courses.

45 C
46 B
47 C
48 E
101

49. True statements about papillary carcinoma of the thyroid do not


include that it:
a) Often affects adolescents.
b) ls a slow-growing tumor.
c) May be hormone-dependent.
d) Metastasize early by the blood stream.
e) ls rodioresistant.

50. ln subacute thyroiditis (de Quervain's disease), it is untrue that it:


a) Is a virus infection related to influenza or mumps
b) Has a sudden onset with fever and painful swelling of the gland.
c) Moy cause radiating pain in the ear.
d) Never resolves spontaneously.
e) Responds well to prednisone.

51. In hashimoto’s disease (lymphadenoid goitre), the false statement


that it :
a) Usually affects menopausal women.
b) Is characterized by uniform smooth enlargement of the gland with
hard consistency.
c) May be associated with hypothyroidism.
d) Has a characteristic histological picture.
e) Is best treated by radiotherapy.

52. The most frequent cause of primary hyperparathyroidism is:


a) Parathyroid adenoma.
b) Idiopathic parathyroid hyperplasia.
c) Primary parathyroid carcinoma.
d) Familial hyperparathyroidism.
e) Ectopic production of parathormone.

49 D
50 D
51 E
52 A
102

53. Hypertension in a patient with a family history of medullary thyroid


carcinoma is most often due To:
a) Renal artery stenosis.
b) Glomerulonephritis.
c) Cushing’s syndrome.
d) Hyperparathyroidism.

54- All the following are True about the thyroglossal cyst except:
a) Painless swelling.
b) Cystic in consistency.
c) Moves up & Down with swallowing but not with protrusion of
the tongue.
d) If infected, can form a thyroglossal fistula.

55- Regarding thyroiditis, all are true except:


a) Inflammatory conditions of the thyroid gland.
b) It's rare.
c) Painless.
d) Dramatic response to prednisone & anti-inflammatories.

56- The simple Goiter has the following featuring except:


a) More common on females.
b) Presented by diffuse, soft enlargement of the gland in cases if
diffuse simple goiter.
c) Shows toxic manifestations.
d) Can be treated surgically.

53 E
54 C
55 C
56 C
103

57- The simple nodular goiter can be complicated by one of the


following EXCEPT:
a) 2ry thyrotoxicosis.
b) Haemorrhage.
c) Turn malignant in 30% of cases
d) Stridor.

58- Retrosternal goiter has the following features EXCEPT:


a) Dullness on direct sterna percussion.
b) Not an indication for surgery.
c) Diagnosis by X-ray.
d) More common in male.

59- All are features of Grave's disease EXCEPT:


a) Affects mainly elderly males more than females.
b) Shows clinical picture of hyperthyroidism.
c) Thyroid profile has a "low" TSH value.
d) Can be accompanied by exophthalmos.

60- Exophthalmos can be detected by all of the following EXCEPT:


a) Elevated free T3 and T4.
b) Clinically by starring look and loss of forehead wrinkling on looking
upwards.
c) Exophthalmometer.
d) Autoimmune survey to detect the TSA (Thyroid Stimulating
Antibodies)

57 D
58 B
59 A
60 A
104

61- As differentiation between primary and 2ry toxic goiter all are true
EXCEPT:
a) The 2ry is more common in elderly.
b) The 2ry appears with gradual onset.
c) The 2ry is presented by diffuse enlargement of the gland.
d) Nervous manifestation, metabolic disorders and eye signs are
more prominent in the 1ry form.

62- All are indications of surgery in Goiter EXCEPT:


a) Retrosternal goiter.
b) 2ry toxic goiter.
c) Thyroiditis.
d) Failure of medical treatment

63- Pre-operative preparation for the patient include all of the


following EXCEPT:
a) CBC, coagulation profile, liver and kidney profile.
b) CXR and indirect laryngoscopy.
c) Thyroid profile and neck ultrasound.
d) Recurrent laryngeal nerve conduction.

64- Regarding the recurrent laryngeal nerve, all are true EXCEPT:
a) If injured bilaterally, completely, the patient will be aphonic.
b) Stridor will occur in unilateral complete injury.
c) Can be injured during devascularization of the inferior lobe.
d) Can be non-recurrent only on the right side.

61 C
62 C
63 D
64 B
105

65- Most common type of the thyroid malignancy is:


a) Papillary carcinoma.
b) Anaplastic carcinoma.
c) Follicular carcinoma.
d) Metastatic deposits.

66-Anaplastic carcinoma has the following characters:


a) More common in elderly, bad prognosis, early spread and
aggressive.
b) Main way of spread is blood.
c) Can occur in children.
d) Characterized pathologically by Psammoma Bodies.

67- The following is not one of the characters of the malignant thyroid
cyst:
a) Its aspirate is haemorrhagic.
b) Cytology shows malignant cells.
c) Rapid re-accumulation after aspiration.
d) The cyst will be completely disappeared after aspiration.

68- Malignant goiter, if painful the pain will be referred to the ear
through:
a) Phrenic nerve.
b) Sympathetic plexus.
c) Bronchial plexus.
d) Arnold's nerve ( branch of vagus nerve)

65 A
66 A
67 D
68 D
106

69- The following is a cause of painful goiter:


a) Acute thyroiditis.
b) Simple nodular goiter.
c) Retrosternal goiter.
d) Goiter with pregnancy.

70- In a case of goiter, the following may be cause of un-equal pulse


during examination:
a) Diffuse goiter.
b) Cancer thyroid.
c) Retrosternal goiter.
d) Hashimoto's disease.

71- Hard goiter can be felt during examination of the gland in the
following cases:
a) Malignancy.
b) Calcified simple nodular goiter.
c) Riedle's thyroiditis.
d) Tense thyroid cyst.

72-All of the following are causes of dullness over the manubrium stain
EXCEPT:
a) Ectopic thyroid.
b) Pneumothorax.
c) Pre-tracheal lymph nodes.
d) Retrosternal goiter.

69 A
70 C
71 A
72 B
107

GIT

1- As regard Meckel's diverliculum all are correct except:


a- It occurs due to persistence of proximal urachus
b- It arises from antimesentric border
c- It lies 60 cm (2 feet) from caecum
d- It's 2 inches long

2-lndications of surgical removal in accidentally discovered Meckel's


during laparotomy include the following except:
a- Narrow mouthed
b- Adhesions
c- Diabetics
d- Young age of patient

3- Most common complication of Meckel's is:


a- Diverticulitis
b- Intestinal obstruction
c- Peptic ulceration
d- Littre's hernia

4- Meckel's diverticulum most commonly presents by:


a. Gastrointestinal bleeding.
b. Obstruction.
c. Diverticulitis.
d. Intermittent abdominal pain.

1 2 3 4

A B A C
108

5-True statements about chronic duodenal ulcers do not include that


they:
a- Are never malignant.
b- Occur equally in both sexes.
c- Produce epigastric pain several hours after eating.
d- Have a periodic clinical course.
e- May heal under medical treatment.

6. The cardinal symptoms of uncomplicated duodenal ulcer do not


include:-
a. Anorexia.
b. Localized midepigastric pain.
c. Hunger pain.
d. Nocturnal pain.
e. Periodic remissions and exacerbations.

7- Perforated duodenal ulcer is best treated by:


a. Gostroduodenal suction ond antibiotics.
b. Simple closure over on omental patch.
c. Truncal vagotomy.
d. Super selective vagotomy.
e. Portiol gostrectomy.

8- The most frequent complication of regional illeitis is


a. Abscess formation.
b. Internal fistulae.
c. Hemorrhoge.
d. Perforation.
e. Intestinal obstruction.

5 6 7 8

B A B B
109

9- The least common site for cancer in the alimentary tract is:
a. Pharynx.
b. Esophagus.
c. Stomach.
d. Small bowel.
e. Colon and rectum.

10- Common causes of secondary intussusception include the following


except:
a. Small intestinal tumor.
b. sigmoid Cancer.
c. Meckel's diverticulum.
d. Henoch shonlein purpra.

11- The following are risk factors for volvulus of pelvic colon except:
a. Old age.
b. Chronic constipation.
c. Short sigmoid.
d. Adhesions.

12- Conservative treatment is indicated in early uncomplicated


intestinal obstruction in the following cases except:
a. Adhesive l.O.
b. Primary intussusception.
c. Secondory intussusception.
d. Volvulus.

9 10 11 12

D B C C
110

13- As regard hydrostatic barium reduction of intussusception all are


correct except:
a. Should be tried in all cases.
b. Successful in about half of cases.
c. Success is proved by filling of terminal ileum.
d. May be complicated by perforation.

14- As regard Volvulus neonatorum all are correct except:


a. Never to occur without anomalies.
b. Usual presentation is screaming and non bile stained vomiting.
c. Bleeding per rectum is common.
d. Primary resection is usually incompatible with life.

15- The most common cause of death in low intestinal obstruction is:
a. Toxemia.
b. Dehydration.
c. Electrolyte imbalance.
d. Generalized peritonitis.

16- Colicky pain is absent in the following types of intestinal


obstruction:
a. Strangulated hernia.
b. Volvulus.
c. Paralytic ileus.
d. lntussusception.

13 14 15 16

A B A C
111

17- The following types of intestinal obstruction may present without


absolute constipation except:
a. Early cases of high I.O.
b. Gall stone ileus.
c. Mesenteric vascular occlusion.
d. Paralytic ileus.
e. Richter's hernia.

18- Signs of strangulated intestinal obstruction include:


a. Nasogastric suction does not relief pain.
b. Rebound tenderness.
c. Leucocytosis.
d. All of the obove.

19- The following are recognized causes of paralytic ileus. The most
common is:
a. Diabetic ketoacidosis.
b. Drugs.
c. Peritoneal irritation.
d. Postoperative.
e. Spinal injury.

20- As regard meconium ileus:


a. Occur during first few days of neonatal life.
b. Due to inspissated meconium.
c. May be associated with cystic fibrosis.
d. All of the above.

17 18 19 20

D D D D
112

21- The most common injured artery in mesenteric vascular occlusion


is:
a. Celiac artery.
b. Gastroduodenal artery.
c. Superior mesenteric artery.
d. lnferior mesenteric artery.

22- In children and adolescents, the commonest cause of intestinal


obstruction is:
a. Adhesions
b. lntussusception
c. Stranguloted hernias
d. Neoplasm
e. Paralytic ileus

23- Sigmoid volvulus has been associated with each of the following
except:
a. Chronic constipation and laxative abuse.
b. Chronic rectal prolapse.
c. Chronic traumatic paralysis.
d. Medical management of Parkinson's disease.

24- Uncomplicated meconeum ileus is best treated by:


a. Laparotomy and evacuation
b. Transverse colostomy
c. N-acetyl cysteine barium enema
d. Wait and watch

21 22 23 24

C C B C
113

25- Example of third space sequestration is:


a. Burn
b. Fracture hematoma
c. Small bowel obstruction
d. All of the above

26- A 20-year-old man swallowed two open safety pins. X-rays show
pins in the small intestine, the most appropriate management of this
point is:
a. IV antibiotics
b. 250 ml magnesium citrate orally
c. lmmediate surgery
d. Serial abdominal exam & x-rays if required

27-In generalized peritonitis, the symptomatology does not include:


a. Fever and tachycardia.
b. Severe colicky abdominal pain.
c. Diffuse tenderness and rigidity.
d. Silent abdomen on auscultation.
e. Shifting dullness.

28- In acute intestinal obstruction, it is untrue that :


a. Abdomen should always the examined for laparotomy scars and
external hernias.
b. Signs of peritonitis indicate strangulation.
c. The serum amylase is often raised.
d. A rectal examination is essential.
e. Plain X-ray examination is not helpful in the diagnosis.

25 26 27 28

D D B E
114

29- In intestinal strangulation, it is untrue that


a- it is difficult to differentiate from simple occlusion.
b- May complicate closed-loop obstruction.
c- Causes bleeding into the affected bowel.
d- Frequently causes peritonitis.
e- Requires urgent laparotomy.

30- The commonest cause of pyloric obstruction in adults is:


a. Prepyloric gastric ulcer.
b. Benign gastric tumor.
c. Carcinoma of the stomach.
d. Hypertrophic pyloric stenosis.
e. Chronic duodenal ulcer.

31- Chronic pyloric obstruction produces all of the following except:


a. Alkalosis.
b. Hypokalemia.
c. Hypochloremia.
d. hypermagnisemia.
e. Dehydration.

32- A 40-year-old female presented with colicky abdominal pain,


vomiting and constipation of 48 hours duration. She gave a history of
previous laparotomy and examination revealed abdominal distension
with loud bowel sounds. The most probable diagnosis is:
a. Adhesive intestinal obstruction.
b. Gall-stone ileus.
c. Internal hernia.
d. Paralytic ileus.
e. Mesenteric vascular occlusion.

29 30 31 32

A E D A
115

33- Concerning infantile intussusception the following statements are


true except that it:
a. Usually occurs during the first year of life.
b. Causes recurrent attacks of severe colicky abdominal pain.
c. ls characterized by the passage of "red-current jelly" stool.
d. Can be diagnosed without radiological examination.
e. Always requires urgent operation.

34- As regard treatment of congenital megacolon:


a- Mild cases should be prepared for elective surgery
b- Emergency cases need urgent decompression for fear of Obstructive
enterocolitis.
c- Duhamel's operation preserves stretch receptors and rectal capacity
d- All ore true

35- Bleeding per rectum is recognized feature in all except:


a- colon Cancer
b- Hirschsprung disease
c- Ulcerative colitis
d- Diverticular disoeose
e- Internal piles

36- Complications of congenital megacolon include the following


except:
a- Failure to thrive
b- Bad chest
c- Cancer colon
d- Obstructive enterocolitis.

33 34 35 36

E D B C
116

37- Hirschsprung disease may presents by the following except:


a- No meconium for 24 hours
b- Persistent non-bile stained vomiting since birth
c- Chronic constipation relieved only by glycerin suppositories or mother
finger
d- Acute intestinal obstruction

38- The commonest cause of acquired megacolon is:


a- Bad bowel habits
b- Anal fissure
c- Ameobiasis
d- Bilhoziasis

39- The usual presentation of uncomplicated diverticulosis is:


a- Asymptomatic
b- Dull aching right iliac fossa pain
c- Fever and throbbing pain
d- Bleeding per rectum

40-The best investigation of diverticular disease is:


a- Endoscopy
b- Barium enema
c- Angiography
d- Abdominal US

37 38 39 40

B A A B
117

41- Which of the following is not true of diverticular disease:


a- It is more common in the United States and Western Europe than in
Asia and Africa.
b- A low-fiber diet may predispose to development of diverticulosis.
c- It involves sigmoid colon in more than 90% of patients.
d- Sixty percent develop diverticulitis sometime during their lifetime.
e- It is the most common cause of massive lower gastrointestinal
hemorrhage.

42-Barium enema findings suggestive of UC are all Except:


a- Loss of haustrations.
b- Granular mucosa.
c- Sow tooth appearance.
d- Pseudo polyps.

43- As regard crohn's disease all are correct except:


a- Associated with gall stones in many cases
b- Most acceptable etiological theory is autoimmune
c- Is has a transmural affection
d- It does not increase incidence of colonic carcinoma

44-Which of the following statement(s) about complete rectal prolapse


is true:
a- Rectal prolopse results from intussusception of the rectum and recto
sigmoid.
b- The disorder is more common in men than in women.
c- Continence nearly always is recovered after correction of the
prolopse.
d- All of the above are true.

41 42 43 44

D C D A
118

45- Gardner's syndrome includes all the following except:


a- Familial polyposis coli
b- Osteoma of the mandible
c- Desmoid tumors
d- Hyperparathyroidism
e- Subcutaneous cysts

46-The following inherited disorder is autosomal recessive:


a- Peutz-jeger's syndrome
b- Familial polyposis coli
c- Turcot's syndrome
d- Gardner syndrome

47- The most common site for colon cancer is:


a- Caecum
b- Ascending colon
c- Transverse colon
d- Descending colon
e- Sigmoid and rectum

48- True statements regarding intestinal carcinoma that arises


following ulcerative colitis include the following except:
a- It is more malignant than the carcinoma that occurs in otherwise
normal adults.
b- The incidence of carcinoma increases with the duration of active
ulcerative colitis.
c- The carcinoma occurs only in the rectum.
d- The carcinoma is frequently multicentric.

45 46 47 48

D C E C
119

49- Which of the following is a precancerous state in the large bowel:


a- Diverticular disease
b- Bilharzial Colitis
c- Peutz-Jegher's syndrome
d- Gardner's syndrome

50- Peutz-Jegher syndrome can present by the followings:


a- Bleeding per rectum
b- Colics and vomiting
c- Anemia of unknown etiology
d- Oral pigmentation
e- Any of the above

51- Correct statements about Hirschsprung's disease include the


following except that it
a. Is due to congenital absence of ganglion cells from the rectum and
port of the colon.
b. Presents with constipation since birth.
c. Is characterized by o tight empty rectum on rectal examination.
d. Rarely needs rectal biopsy for definitive diagnosis.
e. Always requires surgical treatment.

52- Sigmoid volvulus is characterized by the following features except:


a. Occurring typically in elderly men.
b. Acute onset of severe cramping abdominal pain.
c. Absence of abdominal distension.
d. Absolute constipation with distressing tenesmus.
e. Characteristic signs in the barium enema.

49 50 51 52

D E D C
120

53- The following statements about diverticular disease of the colon


are true except that it:
a. Increases in incidence with advancing age.
b. Does not involve the rectum.
c. Is essentially due to o high-residue diet.
d. May be asymptomatic.
e. Moy cause massive rectal bleeding.

54- Which statement among the following is untrue about cancer of


right colon?
a. Is commonest in the caecum.
b. Rarely presents with intestinal obstruction.
c. Moy present with anemia of obscure origin.
d. Is rarely palpable on abdominal examination.
e. Is best treated by right hemicolectomy.

55- Carcinoma of the left colon differs from that of the right colon in
the following except that it:
a. Is usually a stenosing scirrhous lesion.
b. Frequently presents with intestinal obstruction.
c. Rarely causes diarrhea.
d. Is usually impalpable on abdominal examination.
e. Carriers a better prognosis after radical resection.

56- The following statements about partial rectal prolopse are true
except that it:
a. Consists of a double layer of mucous membrane.
b. Is commonest in elderly people.
c. Is often associated with poor sphincter tone.
d. Is rarely associated with hemorrhoids.
e. Is best treated by ligature-excision of prolapsing mucosa.

53 54 55 56

C D C D
121

57- An elderly male with history of habitual constipation presented


because of progressive abdominal distension with diarrhea and
tenesmus. He should be suspected to suffer from:
a. Carcinoma of the rectum.
b. None-specific ulcerative colitis.
c. Amoebic colitis.
d. Crohn's disease.
e. Proctocolitis.

58- The best screening investigation for cancer of the colon in the
general population is:
a. Abdominal ultrasound.
b. Barium enema.
c. Stool occult blood test.
d. Carcinoembryonic antigen (CEA) assay.
e. Colonoscopy.
59. A 25-year-old male develops diarrhea and colicky abdominal pain.
Ulcertive colitis is diagnosed on colonoscopy. Which of the following
findings is consistent with the diagnosis?
(A) The rectum is not involved.
(B) The disease is confluent, there are no skip areas in the colon and the
rectum is involved.
(C) The full thickness of the bowel wall is involved.
(D) Microscopic examination of the mucosa reveals normal cells without
evidence of dysplasia.
(E) The incidence of colorectal cancer is equal to that of the general
population
60. A 35-year-old man has known ulcerative colitis. Which of the
following is an indication for total proctocolectomy?
(A) Occasional bouts of colic and diarrhea
(B) Sclerosing cholangitis
(C) Toxic megacolon
(D) Arthritides
(E) Iron deficiency anemia

57 58 59 60

A C B C
122

61.A 55-year-old man presents with left lower quadrant (LLQ)


abdominal pain of 2-day duration, associated with constipation. On
physical examination, he has tenderness localized to the LLQ with
fullness in that area leukocyte count is 22,000 and temperature is
101.5°F. Which would be the best diagnostic study to evaluate this
man?
(A) Diagnostic laparoscopy
(B) Barium enema
(C) Plain abdominal roentgenogram
(D) Computed tomography (CT) of the abdomen/pelvis with orally (PO)
and intravenous (IV) contrast
(E) Colonoscopy
62.A patients CT scan reveals diverticulitis confined to the sigmoid
colon. There is no associated pericolic abscess. What is best course of
treatment?
(A) Bowel rest, nasogastric suction, IV fluids, and broad spectrum
antibiotics
(B) Urgent surgical resection
(C) Steroids
(D) Diverting colostomy
(E) Ileostomy

63. The standard initial therapy for acute sigmoid volvulus is:
(A) Laparotomy to reduce the volvulus and replace the sigmoid colon to
its normal position
(B) IV neostigmine
(C) Colonoscopy
(D) Ileostomy
(E) Rigid sigmoidoscopy
64. 3 cm in length. What is the most common complication of Meckel’s
diverticulum among adults?
(A) Bleeding
(B) Perforation
(C) Intestinal obstruction
(D) Ulceration
(E) Carcinoma
61 62 63 64

D A E C
123

65. A 30-year-old male is diagnosed with Peutz- Jeghers syndrome. What findings
are consistent with the diagnosis?
(A) Adenomas
(B) Hamartomas
(C) Adenomatous polyps
(D) Villoglandular polyps
(E) Villotubular polyps
66.A 79-year-old man has had abdominal pain for 4 days. An operation is
performed, and a gangrenous appendix is removed. The stump is inverted. Why
does acute appendicitis in elderly patients and in children have a worse prognosis?
(A) The appendix is retrocecal.
(B) The appendix is in the preileal position.
(C) The appendix is in the pelvic position.
(D) The omentum and peritoneal cavity appear to be less efficient in localizing the
disease in these age groups.
(E) The appendix is longer in these age groups
67. A 12-year-old boy complains of pain in the lower abdomen (mainly on the right
side). Symptoms commenced 12 hours before admission. He had noted anorexia
during this period. Examination revealed tenderness in the right iliac fossa, which
was maximal 1 cm below Mc Burney’s point. In appendicitis, where does the pain
frequently commence?
(A) In the right iliac fossa and remains there
(B) In the back and moves to the right iliac fossa
(C) In the rectal region and moves to the right iliac fossa
(D) In the umbilical region and then moves to the right iliac fossa
(E) In the right flank
68. On examination, patients presenting with appendicitis typically show maximal
tenderness over which of the following?
(A) Inguinal region
(B) Immediately above the umbilicus
(C) At a point between the outer one-third and inner two-thirds of a line between
the umbilicus and the anterior superior iliac spine
(D) At a point between the outer two-thirds and inner one-third of a line between
the umbilicus and the anterior superior iliac spine
(E) At the midpoint of a line between the umbilicus and the anterior superior iliac
Spine

65 66 67 68

B D D C
124

69. A 29-year-old woman presents to her physician’s office with pain in


the right iliac fossa. Examination reveals tenderness in this region. Her
last menstrual cycle was 2 weeks previously and findings on
gynecologic examination and leukocyte count are normal. A
provisional diagnosis of acute appendicitis is made. She should be
informed that operations to treat this condition reveal acute
appendicitis in what percentage of cases?
(A) A small percentage of cases
(B) 50–89% of cases
(C) 90–99% of cases
(D) More than 99% of cases
(E) No reliable statistics are available
70. A 28-year-old man is admitted to the emergency department
complaining of pain in the umbilical region that moves to the right iliac
fossa. Which is a corroborative sign of acute appendicitis?
(A) Referred pain in the right side with pressure on the left (Rovsing) sign
(B) Increase of pain with testicular elevation
(C) Relief of pain in lower abdomen with extension of thigh
(D) Relief of pain in lower abdomen with internal rotation of right thigh
(E) Hyperanesthesia in the right lower abdomen
71. A 54-year-old man with diarrhea is found to have ulcerative colitis.
Colectomy should be advised in patients with ulcerative colitis who
have symptoms that persist for more than which of the following?
(A) 1 month
(B) 6 months
(C) 1–5 years
(D) 10–20 years
(E) More than 25 years
72. A 48-year-old woman develops colon cancer. She is known to have
a long history of ulcerative colitis. In ulcerative colitis, which of the
following is a characteristic of colon cancer?
(A) Occurs more frequently than in the rest of the population.
(B) Is more likely to occur when the disease is confined to the left colon.
(C) Occurs equally in the right and left side.
(D) Has a synchronous carcinoma in 4–5% of cases.
(E) Has an excellent prognosis because of physician awareness
69 70 71 72

C A D C
125

73. A72-year-old woman is scheduled to undergo right hemicolectomy


for cancer of the cecum. In this condition, she can anticipate
subsequent recurrence
(A) Of 20–30% if confined to the mucosa
(B) Close to 100% if there is lymph node involvement
(C) Which will not result in small-bowel obstruction
(D) Which will not result in hydronephrosis
(E) Which with microscopic lymph node metastasis would have a lower
rate than that with macroscopic spread
74. An 83-year-old man is diagnosed on colonoscopy to have cancer of
the colon. He refuses surgical intervention and after a 3-month follow
up period is admitted to the emergency department with large-bowel
obstruction. Carcinoma of the colon is most likely to obstruct if found
in the
(A) Cecum
(B) Ascending colon
(C) Descending colon
(D) Rectum
(E) Transverse colon
75. A43-year-old man is seen in his physician’s office for severe pain in
the perineum. Examination reveals exquisite tenderness in the area to
the right side of the anal verge due to a perianal abscess. Rectal
examination is refused. What should be the next step in management?
(A) Drainage of the abscess in the office under local anesthesia.
(B) Excision of the vertical fold of Morgagni.
(C) Drainage under general anesthesia and immediate colonoscopy.
(D) CT scan of the abdomen.
(E) Insertion of a rectal tube.
76.A 64-year-old man undergoes CEA surveillance for cancer, because
his brother and father both had colon cancer. What information should
he be provided?
(A) CEA is highly sensitive for diagnosis.
(B) If CEA is elevated preoperatively, it implies unresectable disease.
(C) Increases in CEA after resection may indicate tumor recurrence.
(D) CEA is highly specific for the presence of colon cancer.
(E) CEA is present in normal adult colonic mucosa
73 74 75 76

E C A C
126

77. A 70-year-old man presents with pallor and breathlessness on


exertion. He does not complain of abdominal pain. He has microcytic,
hypochromic anemia. What is the most probable cause?
(A) Diverticulosis of the colon
(B) Peptic ulcer disease
(C) Crohn’s disease
(D) Ulcerative colitis
(E) Carcinoma of the right colon
78. A 25-year-old man has recurrent, indolent fistula in ano. He also
complains of weight loss, recurrent attacks of diarrhea with blood
mixed in the stool, and tenesmus. Proctoscopy revealed a healthy,
normal-appearing rectum. What is the most likely diagnosis?
(A) Crohn’s colitis
(B) Ulcerative colitis
(C) Amoebic colitis
(D) Ischemic colitis
(E) Colitis associated with acquired immunodeficiency syndrome (AIDS)
79. A 65-year-old woman with a history of chronic constipation is
transferred from a nursing home because of abdominal pain and
marked abdominal distention. On examination, her abdomen is found
to be distended and tender in the LLQ. What is the most likely
diagnosis?
(A) Appendicitis
(B) Carcinoma of the colon
(C) Volvulus of the sigmoid colon
(D) Volvulus of the cecum
(E) Small-bowel obstruction
80. A 40-year-old man with a long history of bloody diarrhea presents
with increased abdominal pain, vomiting, and fever. On examination,
he is found to be dehydrated and shows tachycardia and hypotension.
The abdomen is markedly tender with guarding and rigidity. What is
the most likely cause?
(A) Toxic megacolon in ulcerative colitis
(B) Small-bowel perforation from regional enteritis
(C) Perforated carcinoma of the sigmoid colon
(D) Volvulus of the sigmoid colon
(E) Acute perforated diverticulitis

77 78 79 80
E A C A
127

81.A 55-year-old woman presents with pain in the LLQ of the abdomen
and fever of 102°F. On examination, she is found to be dehydrated and
has tenderness in the LLQ. A CT scan shows a mass in the LLQ involving
the sigmoid colon. There is a minimal amount of free fluid and no free
air. What should the initial treatment of this patient include?
(A) IV fluids, penicillin, and steroids
(B) IV fluids, cefoxitin, and nasogastric drainage
(C) IV fluids, blood transfusion, and laparotomy
(D) Immediate laparotomy
(E) Bowel preparation followed by Laparotomy
82. A 60-year-old man complains of recurrent attacks of painless rectal
bleeding. Colonoscopy reveals normal mucosa between the cecum and
the anal verge. What is the most helpful test to determine the cause of
bleeding?
(A) Angiography to look for angiodysplasia
(B) Technetium scan for Meckel’s diverticulum
(C) Upper GI endoscopy for peptic ulcer
(D) Small-bowel series for tumor
(E) Ultrasound for abdominal aortic Aneurysm
83. A 55-year-old man has had previous hemicolectomy for a
carcinoma of the right colon. At this time, 3 years after the primary
resection, a CT scan shows a solitary lesion in the right lobe of the liver.
What is the next step in management?
(A) Laser cauterization
(B) Radiotherapy
(C) Hepatic artery catheterization and local chemotherapy
(D) Symptomatic treatment with analgesics, because the colon disease is
now stage IV
(E) Exploratory laparotomy and resection of the tumor.
84. A 68-year-old man presents with crampy abdominal pain and
distention with vomiting. Findings on physical examination are positive
for healed abdominal scars. X-rays reveal multiple gas fluid levels. The
WBC count is 12,000. What is the most likely diagnosis?
(A) Small-bowel intestinal obstruction due to adhesions
(B) Hernia
(C) Appendicitis
(D) Inflammatory bowel disease
(E) Gallstones and ascites
81 82 83 84
B A E A
128

85 A 64-year-old man has a benign lesion of the colon. He is informed


that the lesion does not predispose to colon cancer. What is the lesion
he has?
(A) Ulcerative colitis
(B) Villous adenoma
(C) Hyperplastic polyp
(D) Adenoma in familial polyposis
(E) Colon mucosa in a patient with colon Carcinoma
86. A 55-year-old woman presents with vague RLQ abdominal pain. A
palpable mass is noted on abdominal examination. The mass is
painless, well defined, mobile, and nonpulsatile. What is the most
likely diagnosis?
(A) A mesenteric cyst (B) Appendix mass
(C) Perforated tubo-ovarian abscess
(D) Cholecystitis
(E) Meckel’s diverticulum
87. A 74-year-old woman complains of vomiting and intermittent
colicky abdominal pain. X-rays reveal fluid levels and air in the biliary
tree. What is the likely cause?
(A) Abdominal adhesions
(B) Gallstone ileus
(C) Carcinoma of the right colon
(D) Abdominal lympho sarcoma
(E) Previous choledochoduodenostomy
88. A 68-year-old male musician presents to the emergency
department with a sudden onset of colicky abdominal pain and
massive vomiting of 4-hour duration. Examination shows an elevated
WBC of 13,200 with a HCT of 45%. Electrolytes and blood urea nitrogen
(BUN) are normal. An erect film of the abdomen reveals dilatation of
the stomach with distended loops of bowel. What is his clinical
diagnosis?
(A) Complete proximal intestinal obstruction
(B) Incomplete proximal intestinal obstruction
(C) Complete ileal obstruction
(D) Incomplete ileal obstruction
(E) Small-bowel perforation
85 86 87 88

C A B A
129

89. Following resection of the left colon, a 67- year-old obese woman
develops left-sided leg edema due to deep-vein thrombosis. She is
placed on anticoagulants, but after 2 weeks of warfarin (Coumadin),
she develops a pulmonary embolus with slight hypoxemia. What
should the next step in management involve?
(A) Increasing the dose of anticoagulants
(B) Discontinuing anticoagulants
(C) Use of an inferior vena cava (IVC) filter
(D) CT scan of the leg and abdomen
(E) Femoral vein ligation

90. A64-year-old woman presents with a strangulated femoral hernia.


At operation, what is the criterion used to determine the viability of a
loop of bowel?
(A) Increased peristalsis
(B) Absent arterial pulsation
(C) Venous engorgement
(D) Intraoperative CT scan
(E) Serum amylase

89 90

C B
130

91. A 45-year-old man complains of burning epigastric pain that wakes him up at
night. The pain is relieved by eating or using over-the counter antacids and H2
blockers. Diagnosis is best confirmed by which of the following?
(A) Urea breath test
(B) Serum gastrin levels
(C) Barium meal examination
(D) Upper endoscopy
(E) Upper endoscopy and biopsy
92. A 64-year-old woman with arthritis is a chronic NSAID user. She develops
severe epigastric pain and undergoes an upper endoscopy. She is told that she has
an ulcer adjacent to the pylorus. Which of the following is TRUE about the pylorus?
(A) It cannot be palpated at laparaotomy.
(B) It is not covered completely by omentum.
(C) It is a distinct anatomic entity that can
be distinguished during laparotomy.
(D) It is a true physiologic sphincter.
(E) It is a site where cancer is rarely found
93. A 30-year-old executive learns that he has a duodenal ulcer. His
gastroenterologist prescribes and outlines medical therapy. The patient worries
that if medical therapy fails he may need surgery. Which of the following is the
best indication for elective surgical therapy for duodenal ulcer disease?
(A) An episode of melena
(B) Repeated episodes of pain
(C) Pyloric outlet obstruction due to scar formation from an ulcer
(D) Frequent recurrences of ulcer disease
(E) Referral of pain to the back, suggestive of pancreatic penetration

94. A44-year-old dentist was admitted to the hospital with a 1-day history of
hematemesis caused by a recurrent duodenal ulcer. He has shown considerable
improvement following operative treatment by a truncal vagotomy and
pyloroplasty, 10 years prior to this incident. Which is TRUE of truncal vagotomy?
(A) It is performed exclusively via the thorax.
(B) It can be performed in the neck.
(C) If complete, it will result in increased acid secretion.
(D) It requires a gastric drainage procedure
(E) It has been abandoned as a method to treat ulcer disease

91 92 93 94
E C C D
131

95. A42-year-old executive has refractory chronic duodenal ulcer disease. His
physician has suggested several surgical options. The patient has chosen a parietal
(highly selective) vagotomy instead of a truncal vagotomy and antrectomy
because?
(A) It results in a lower incidence of ulcer recurrence.
(B) It benefits patients with antral ulcers the most.
(C) It reduces acid secretion to a greater extent.
(D) The complication rate is lower.
(E) It includes removal of the ulcer.

96. A63-year-old woman is admitted to the hospital with severe abdominal pain of
3-hour duration. Abdominal examination reveals board-like rigidity, guarding, and
rebound tenderness. Her blood pressure is 90/50 mm Hg, pluse 110 bpm (beats per
minute), and respiratory rate is 30 breaths per minute. After a thorough history
and physical, and initiation of fluid resuscitation, what diagnostic study should be
performed?
(A) Supine abdominal x-rays
(B) Upright chest x-ray
(C) Gastrograffin swallow
(D) Computerized axial tomography (CAT) scan of the abdomen
(E) Abdominal sonogram

97. A frail elderly patient is found to have an anterior perforation of a duodenal


ulcer. He has a recent history of nonsteroidal anti-inflammatory drug (NSAID) use
and no previous history of peptic ulcer disease. A large amount of bilious fluid is
found in the abdomen. What should be the next step?
(A) Lavage of the peritoneal cavity alone
(B) Lavage and omental patch closure of the ulcer
(C) Total gastrectomy
(D) Lavage, vagotomy, and gastroenterostomy
(E) Laser of the ulcer

98. Three months after recovery from an operation to treat peptic ulcer disease, a
patient complains that she has difficulty eating a large meal. A 99m Tc-labeled
chicken scintigraphy test confirms a marked delay in gastric emptying. A delay in
gastric emptying may be due to which of the following?
(A) Zollinger-Ellison syndrome (ZES)
(B) Steatorrhea
(C) Massive small-bowel resection
(D) Previous vagotomy
(E) Hiatal hernia

95 96 97 98
D B B D
132

99. A64-year-old supermarket manager had an elective operation for duodenal


ulcer disease. He has not returned to work because he has diarrhea with more
than 20 bowel movements per day. Medication has been ineffective. The exact
details of his operation cannot be ascertained. What operation was most likely
performed?
(A) Antrectomy and Billroth I anastomosis
(B) Gastric surgery combined with choleystectomy
(C) Truncal vagotomy
(D) Highly selective vagotomy
(E) Selective vagotomy
100. A40-year-old man has had recurrent symptoms suggestive of peptic ulcer
disease for 4 years. Endoscopy reveals an ulcer located on the greater curvature of
the stomach. A mucosal biopsy reveals Helicobacter. pylori. What is TRUE
About H. pylori?
(A) Active organisms can be discerned by serology.
(B) It is protective against gastric carcinoma.
(C) It is associated with chronic gastritis.
(D) It causes gastric ulcer but not duodenal ulcer.
(E) It can be detected by the urea breath test in <60% of cases.

101. A 63-year-old man has an upper gastrointestinal (UGI) study as part of his
workup for abdominal pain. The only abnormal finding was in the antrum, where
the mucosa prolapsed into the duodenum. There were no abnormal findings on
endoscopy. What should he do?
(A) Sleep with his head elevated.
(B) Be placed on an H2 antagonist.
(C) Undergo surgical resection of the antrum.
(D) Be observed and treated for pain accordingly.
(E) Have laser treatment of the antral mucosa.

102. A63-year-old man underwent gastric resection for severe peptic ulcer disease.
He had complete relief of his symptoms but developed “dumping syndrome.” This
patient is most likely to complain of which of the following?
(A) Gastric intussusception
(B) Repeated vomiting
(C) Severe diarrhea
(D) Severe vasomotor symptoms after eating
(E) Intestinal obstruction

99 100 101 102


C C D D
133

103. A 65-year-old man was admitted to the hospital for severe bilious vomiting
following gastric surgery. This occurs in which circumstance?
(A) Following ingestion of gaseous fluids
(B) Spontaneously
(C) Following ingestion of fatty foods
(D) Following ingestion of bulky meals
(E) In the evening

104. A 64-year-old man has had intermittent abdominal pain as a result of


duodenal ulcer disease for the past 6 years. Symptoms recurred 6 weeks before
admission. He is most likely to belong to which group?
(A) A and secretor (blood group antigen in body fluid)
(B) B and Lewis antigen
(C) AB
(D) O and nonsecretor
(E) O and secretor
105. A64-year-old man was evaluated for moderate protein deficiency. He
underwent a gastrectomy20 years earlier. He is more likely to show which of the
following?
(A) Porphyria
(B) Hemosiderosis
(C) Aplastic anemia
(D) Hemolytic anemia
(E) Iron deficiency anemia

106. A68-year-old woman has been diagnosed with a benign ulcer on the greater
curvature of her stomach, 5 cm proximal to the antrum. After 3 months of
standard medical therapy, she continues to have guaiac positive stool, anemia, and
abdominal pain with failure of the ulcer to heal. Biopsies of the gastric ulcer have
not identified a malignancy. The next step in management is which of the
following?
(A) Treatment of the anemia and repeat all studies in 6 weeks
(B) Endoscopy and bipolar electrocautery or laser photocoagulation of the gastric ulcer
(C) Admission of the patient for total parenteral nutrition (TPN), treatment of anemia, and
endoscopic therapy
(D) Surgical intervention, including partial gastric resection
(E) Surgical intervention, including total Gastrectomy

103 104 105 106


B D E D
134

107. Over the past 6 months, a 60-year-old woman with long standing duodenal
ulcer disease has been complaining of anorexia, nausea, weight loss and repeated
vomiting. She recognizes undigested food in the vomitus. Examination and workup
reveal dehydration, hypokalemia, and hypochloremic alkalosis. What is the most
likely diagnosis?
(A) Carcinoma of the fundus
(B) Penetrating ulcer
(C) Pyloric obstruction due to cicatricial stenosis of the lumen of the duodenum
(D) ZES (Zollinger Ellison Syndrome)
(E) Anorexia nervosa

108. A 50-year-old woman presents with duodenal ulcer disease and high basal
acid secretory outputs. Secretin stimulated serum gastrin levels are in excess of
1000 pg/mL. She has a long history of ulcer disease that has not responded to
intense medical therapy. What is the most likely diagnosis?
(A) Hyperparathyroidism
(B) Pernicious anemia
(C) Renal failure
(D) ZES
(E) Multiple endocrine neoplasia

109. A50-year-old man presents with vague gastric complaints. Findings on


physical examination are unremarkable. The serum albumin level is markedly
reduced (1.8 g/100 mL). A barium study of the stomach shows massive gastric folds
within the proximal stomach. These findings are confirmed by endoscopy. What is
the correct diagnosis?
(A) Hypertrophic pyloric stenosis
(B) Gallstone ileus
(C) Mallory-Weiss tear
(D) Hypertrophic gastritis
(E) Crohn’s disease

110. A 2-cm ulcer on the greater curvature of the stomach is diagnosed in a 70-
year-old woman by a barium study. Gastric analysis to maximal acid stimulation
shows achlorhydria. What is the next step in management?
(A) Antacids, H2 blockers, and repeat barium study in 6 to 8 weeks
(B) Proton pump inhibitor (PPI) (e.g., omeprazole) and repeat barium study in 6 to 8 weeks
(C) Prostoglandin E (misoprostol) and repeat barium study in 6 to 8 weeks
(D) Immediate elective surgery
(E) Upper endoscopy with multiple biopsies (at least 8 or 9) for the ulcer

107 108 109 110


C D D E
135

111. A55-year-old school bus driver was diagnosed 3 months ago with an antral
ulcer. He was treated for H. pylori and continues to take a PPI. Repeat endoscopy
demonstrates that the ulcer has not healed. What is the next treatment option?
(A) Treatment with H2 blockers
(B) Vagotomy alone without additional surgery
(C) Endoscopy and laser treatment of the ulcer
(D) Distal gastrectomy with gastroduodenal anastomosis (Billroth I)
(E) Elevating the head of the bed when asleep

112. A 55-year-old man complains of anorexia, weight loss, and fatigue. AUGI study
demonstrates an ulcerated lesion at the incisura. Where is the incisura?
(A) Cardia
(B) Fundus
(C) Greater curvature
(D) Lesser curvature
(E) Gastrocolic ligament

113. A 36-year-old man presents with weight loss and a large palpable tumor in the
upper abdomen. Endoscopy reveals an intact gastric mucosa without signs of
carcinoma. Multiple biopsies show normal gastric mucosa. A UGI study shows a
mass in the stomach. At surgery, a 3-kg mass is removed. It is necessary to remove
the left side of the transverse colon. What is the most likely diagnosis?
(A) Gastric cancer
(B) Gastrointestinal stromal tumor (GIST)
(C) Choledochoduodenal fistula
(D) Eosinophilic gastroenteritis
(E) Linitis plastica

114. A 74-year-old man presents with anorexia and self-limited hematemesis.


During endoscopy a mass is discovered and a biopsy is done. A hematopathologist
diagnoses non-Hodgkin’s lymphoma. What is the recommended therapy?
(A) Chemotherapy alone
(B) Immunotherapy
(C) Radiation and chemotherapy
(D) Surgery, radiation, and chemotherapy
(E) Surgery alone

111 112 113 114


D D B C
136

115. A63-year-old woman is admitted to the hospital with a UGI bleed that
subsides spontaneously within a short time after admission. A barium study shows
a gastric ulceration that is described by the radiologist as having a “doughnut sign.”
What is the most likely diagnosis?
(A) Lipoma
(B) Gastric ulcer
(C) Ectopic pancreas
(D) GIST
(E) Carcinoma

116. A50-year-old woman is diagnosed with multiple hyperplastic polyps in the


stomach during endoscopy and biopsy. How are these best treated?
(A) Total gastrectomy
(B) Partial gastrectomy
(C) Staged endoscopic removal after brushing for cytologic examination
(D) Ablation by laser
(E) No treatment other than repeated endoscopy and multiple brush biopsies

117. A 64-year-old woman presents with severe upper abdominal pain and
retching of 1-day duration. Attempts to pass a nasogastric tube are unsuccessful. X-
rays show an air-fluid level in the left side of the chest in the posterior
mediastinum. An incarcerated paraesophageal hernia and gastric volvulus is
diagnosed. What is the next step in management?
(A) Insertion of a weighted bougie to untwist the volvulus
(B) Elevation of the head of the bed
(C) Placing the patient in the Trendelenburg position with the head of the bed lowered
(D) Laparotomy and vagotomy
(E) Surgery, reduction of the gastric volvulus, and repair of the hernia

118. A 78-year-old woman undergoes an uncomplicated minor surgical procedure


under local anesthesia. At the completion of the operation, she suddenly develops
pallor, sweating, bradycardia, hypotension, abdominal pain, and gastric distension.
What is the next stem in management?
(A) Rapid infusion of 3 L of Ringer’s lactate
(B) Digoxin
(C) Insertion of a nasogastric tube
(D) Morphine
(E) Neostigmine

115 116 117 118


D C E C
137

119. A 48-year-old man undergoes surgery for a chronic duodenal ulcer. The
procedure is a truncal vagotomy and which of the following?
(A) Gastroenterostomy
(B) Removal of the duodenum
(C) Closure of the esophageal hiatus
(D) Incidental appendectomy
(E) No further procedure

120. A healthy 75-year-old man bleeds from a duodenal ulcer. Medical


management and endoscopic measures fail to stop the bleeding. What is the next
step in management?
(A) Continued transfusion of 8 U of blood
(B) Administration of norepinephrine
(C) Oversewing of the bleeding point
(D) Oversewing of the bleeding point, vagotomy and pyloroplasty
(E) Hepatic artery ligation

121. A 60-year-old woman complains of early satiety and undergoes an upper


endoscopy. A small mass is seen in the antrum with sparing of the mucosa. GIST is
suspected. A CAT scan of the chest, abdomen, and pelvis is performed.
What does she require next?
(A) Fulguration of the tumor
(B) Distal gastrectomy
(C) Laser therapy followed by radiation therapy
(D) Chemotherapy alone
(E) Total gastrectomy

122. A 67-year-old woman complains of paresthesias in the limbs. Examination


shows loss of vibratory sense, positional sense, and sense of light touch in the
lower limbs. She is found to have pernicious anemia. Endoscopy reveals an ulcer in
the body of the stomach. What does she most likely have?
(A) Excess of vitamin B12
(B) Deficiency of vitamin K
(C) Cancer of the stomach
(D) Gastric sarcoma
(E) Esophageal varices

119 120 121 122


A D B C
138

123. A 79-year-old retired opera singer presents with dysphagia, which has become
progressively worse during the last 5 years. He states that he is sometimes aware
of a lump on the left side of his neck and that he hears gurgling
sounds during swallowing. He sometimes regurgitates food during eating. What is
the likely diagnosis?
(A) Carcinoma of the esophagus
(B) Foreign body in the esophagus
(C) Plummer-Vinson (Kelly-Patterson) syndrome
(D) Zenker’s (pharyngoesophageal) diverticulum
(E) Scleroderma

124. A symptomatic patient has a barium swallow that reveals a 3-cm Zenker’s
diverticulum. The next step in management is?
(A) H2 blockers
(B) Anticholinergic drugs
(C) Elemental diet
(D) Bougienage
(E) Surgery (cricopharyngealmyotomy and diverticulectomy)

125. A30-year-old psychiatric patient has a barium swallow after removal of a


foreign body to rule out a small perforation of the esophagus. No perforation is
seen, but an epiphrenic diverticulum is visualized. An epiphrenic diverticulum may
be associated with which of the following?
(A) Duodenal ulcer
(B) Gastric ulcer
(C) Cancer of the tongue
(D) Cancer of the lung
(E) Hiatal hernia

126. A 53-year-old moderately obese woman presents with heartburn aggravated


mainly by eating and lying down in the horizontal position. Her symptoms are
suggestive of gastroesophangeal reflux disease (GERD). Which of the following
statements is TRUE?
(A) It is best diagnosed by an anteroposterior (AP) and lateral film of the chest.
(B) It may be alleviated by certain drugs, especially theophylline, diazepam, and calcium channel
blockers.
(C) It is not relieved by cessation of smoking.
(D) If it is associated with dysphagia, it suggests a stricture or motility disorder.
(E) It should be immediately treated with Surgery

123 124 125 126


D E E D
139

127. A 64-year-old man has symptoms of reflux esophagitis for 20 years. The
barium study shown (Fig. 5–2) demonstrates a sliding hiatal hernia. Which is TRUE
in sliding hiatal hernia?
(A) A hernia sac is absent.
(B) The cardiais displaced into the posterior mediatstinum.
(C) Reflux esophagitis always occur.
(D) A stricture does not develop.
(E) Surgery should always be avoided.

128. A 45-year-old man presents with a long history of heartburn, especially at


night. He uses three pillows to sleep and has medicated himself with a variety of
antacids over the past 15 years. Recently he has been complaining of dysphagia
that he localized to the precordial area. Which is the most likely diagnosis?
(A) Adenocarcinoma of the esophagus
(B) Angina pectoris
(C) Benign peptic stricture of the esophagus
(D) Achalasia of the esophagus
(E) Lower esophageal ring (Schatzki’s ring)

129. A 52-year-old gastroenterologist suffers from intermittent dysphagia


attributed to the presence of a lower esophageal stricture. The doctor’s condition
is characterized by which of the following?
(A) A full thickness scar in the upper esophagus
(B) Symptoms of mild-to-moderate dysphagia
(C) A low incidence in men
(D) The absence of a sliding hiatal hernia in most case
(E) The need for antireflux surgery at an early stage

130. A 60-year-old man presents with excruciating chest pain. The pain follows an
episode of violent vomiting that occurred after a heavy meal. Subcutaneous
emphysema was noted in the neck. X-rays shows air in the mediastinum and neck,
and a fluid level in the left pleural cavity. What is the most likely diagnosis?
(A) Perforated duodenal ulcer
(B) Spontaneous rupture of the esophagus
(C) Spontaneous pneumothorax
(D) Inferior wall myocardial infarction
(E) Dissecting aortic aneurysm

127 128 129 130


B C B B
140

131. A 45-year-old pilot has retrosternal burning; especially when he eats and lies
down to go to sleep. He has self-medicated himself with over the counter
heartburn medications. Upper endoscopy reveals an erythematous and inflamed
distal esophagus. In severe reflux esophagitis, the resting pressure of the LES is
decreased. This may be physiologically increased by which of the following?
(A) Pregnancy
(B) Glucagon
(C) Gastrin
(D) Secretin
(E) Glucagon

132. A46-year-old man has a long history of heartburn (GERD). His barium study
shows an irregular, ulcerated area in the lower third of his esophagus. There is
marked mucosal disruption and overhanging edges. What is the most likely
diagnosis?
(A) Sliding hiatal hernia with GERD
(B) Paraesophageal hernia
(C) Benign esophageal stricture
(D) Squamous carcinoma of the esophagus
(E) Adenocarcinoma arising in a Barrett’s Esophagus

133. A 46-year-old man present with dysphagia of recent onset. His esophogram
shows a lesion in the lower third of his esophagus. Biopsy of the lesion shows
adenocarcinoma. His general medical condition is excellent, and his metastatic
workup is negative. What should his management involve?
(A) Chemotherapy
(B) Radiation therapy
(C) Insertion of a stent to improve swallowing
(D) Surgical resection of the esophagus
(E) Combination of chemotherapy and radiation therapy

134. A 25-year-old man arrives in the emergency department in respiratory


distress following a motor vehicle collision. A chest x-ray shows abdominal viscera
in the left thorax. What is the most likely diagnosis?
(A) Traumatic rupture of the diaphragm
(B) Sliding esophageal hernia
(C) Short esophagus with intrathoracic stomach
(D) Rupture of the esophagus
(E) Bochdalek hernia

131 132 133 134


C E D A
141

135. Following an emergency operation for hepatic and splenic trauma, the
surgeon inserts a finger into the foramen of Winslow in an attempt to stop the
bleeding. Which is TRUE of the hepatic artery?
(A) It is called the common hepatic artery at this level.
(B) It is medial to the common bile duct and anterior to the portal vein.
(C) It is posterior to the portal vein.
(D) It is posterior to the inferior vena cava.
(E) It forms the superior margin of the epiploic foramen.

136. A 26-year-old man is diagnosed with adenocarcinoma of the stomach. He


wants to know what could have caused him to develop this condition. He does an
internet search. Which of the following is a risk factor for developing gastric
cancer?
(A) Exposure to ionizing radiation
(B) Blood group B
(C) A diet high in fiber
(D) H. pylori infection
(E) North American descent

137. A 44-year-old woman is scheduled for gastric surgery. She has no comorbid
disease. The anesthesiologist has difficulty inserting the orotracheal tube. In
between intubation attempts he uses an ambu-bag to oxygenate the patient. The
patient’s abdomen gets distended and tympany is noted in the left upper
quadrant. Suddenly the patient becomes hypotensive. Which of the following can
cause a vosogvagal response during anesthesia?
(A) The gastric remnant following a distal gastrectomy
(B) Corrosive gastritis
(C) Pernicious anemia
(D) Gastric volvulus
(E) Acute gastric dilatation

138. A 40-year-old woman complains of heartburn located in the epigastric and


retrosternal areas. She also has symptoms of regurgitation. Endoscopy shows
erythema of the esophagus consistent with reflux esophagitis. The patient has
tried conservative measures, including PPls with no improvement in symptoms.
Which of the following is TRUE?
(A) Manometry does not add any additional information.
(B) The 24-hour pH test is no longer used.
(C) If endoscopy has been done, an esophagogram is unnecessary.
(D) Nissen fundoplication is the surgical treatment of choice.
(E) Toupet fundoplication is 360 curve

135 136 137 138


B D E D
142

139. A50-year-old man is involved in a major motor vehicle collision and suffers
multiple traumas. He is admitted to the intensive care unit. After 2 days of hospital
admission he bleeds massively from the stomach. What is the probable cause?
(A) Gastric ulcer
(B) Duodenal ulcer
(C) Hiatal hernia
(D) Mallory-Weiss tear
(E) Erosive gastritis

140. A 65-year-old lawyer has an elective colon resection. On postoperative day


number five, the patient develops fever, leukocytosis, and increasing abdominal
pain and distension. An anastomotic leak is suspected. During the preparation for a
CAT scan, fresh blood and coffee grounds are seen in the nasogastric tube.
Acute stress gastritis is best diagnosed by?
(A) CAT scan
(B) UGI series
(C) Angiogram
(D) Capsule endoscopy
(E) Upper endoscopy

141. A73-year-old woman is admitted to the hospital with a mild UGI hemorrhage
that stopped spontaneously. She did not require transfusion. She had ingested
large amounts of aspirin in the past 4 months to relieve the pain caused by severe
rheumatoid arthritis. Endoscopy confirms the presence of a duodenal ulcer. A
biopsy is done. What is the next step in the management of a duodenal ulcer
associated with a positive biopsy for H. pylori?
(A) H2 blockers
(B) Bipolar electrocautery of the ulcer
(C) Triple therapy
(D) Photocoagulation
(E) Elective surgery

142. A52-year-old artist develops epigstric pain that is relieved by antacids. She
also complains that her stool has changed color and is black and tarry. What is the
most important cause of the entity presenting above other than H. pylori?
(A) Submucosal islet cells
(B) Hyperglycemia
(C) Diet
(D) Acid secretion
(E) Acute erosive gastritis

139 140 141 142


E E C D
143

143. An elderly patient delayed seeking medical attention for his early satiety and
weight loss because he attributed these changes to aging. When he underwent
upper endoscopy a large mass was seen in the stomach. Which statement is TRUE
regarding gastric carcinoma?
(A) During resection, it is safe to leave cancer at the cut edges.
(B) The incidence is increased in patients with gastric ulcer disease.
(C) Draining lymph nodes should not be removed.
(D) It is caused by diverticulitis.
(E) It is associated with hyperchlorhydria.

144. A 50-year-old gynecologist complains of dysphagia, regurgitation, and weight


loss. She also states that she feels as if food is stuck at the level of the xiphoid. An
upright chest x-ray shows a dilated esophagus with an air-fluid level. Which of the
following is FALSE?
(A) A barium swallow will show a “bird’s peak” deformity
(B) Manometry will demonstrate that the LES fails to relax during swallowing.
(C) Upper endoscopy should be avoided because of the risk of complications.
(D) Medical treatment includes nitrates and calcium channel blockers.
(E) Intersphincteric injection of botulinum toxin can be therapeutic.

145. A patient has been diagnosed with achalasia. He refused surgery initially,
preferring to try nonoperative therapy. He tried life style modification, calcium
channel blockers, botulin toxin injection, and endoscopic pneumatic dilatation.
None of the treatments alleviated his symptoms. What are his surgical options?
(A) Esophagectomy
(B) Surgical esophagomyotomy proximal to the LES
(C) Modified Heller myotomy and partial fundoplication
(D) Repeat pneumatic dilation using pressures of loops
(E) Nissen fundoplication

146. A 50-year-old man presents to the emergency department with chest pain.
The patient is evaluated for a myocardiac infarction. The workup is negative. On
further questioning, his symptoms include dysphagia (with both liquids and solids).
Which of the following is TRUE?
(A) A barium swallow will always show a corkscrew esophagus.
(B) Manometry shows simultaneous high-amplitude contractions.
(C) Initial evaluation should exclude coronary artery disease.
(D) A pulsion diverticulum may be present.
(E) Patients refractory to medical management may respond to long esophagomyotomy.

143 144 145 146


B C C A
144

147. A 60-year-old man has been having vague symptoms of upper abdominal
discomfort, early satiety, and fatigue. He is referred to a gastroenterologist, who
performs an upper endoscopy. Although a discrete mass is not visualized, the
stomach looks abnormal. It does not distend easily with insufflation. A biopsy
shows signet ring cells. Which of the following is TRUE?
(A) Signet ring cells are typically found in intestinal type gastric adenocarcinoma.
(B) Signet ring cell cancer is the most common type of gastric cancer.
(C) “Leather bottle stomach” is a term used to describe a nondistensible stomach infiltrated by
cancer.
(D) The gross appearance of the stomach always shows classic findings of linitus
plastica.
(E) Linitus plastica has an excellent prognosis.

148. Apatient presents to the emergency department with obstructive jaundice. A


percutaneous transhepaticcholangiogram and biliary drainage is performed.
Shortly afterward, the patient develops a UGI bleed. What is the most likely cause?
(A) The patient has developed stress gastritis.
(B) The patient has ingested NSAIDs after the procedure.
(C) The patient has developed hemobilia.
(D) The patient is bleeding from esophageal varices.
(E) The catheter has migrated from the biliary tree into the stomach

149. A known HIV positive patient complains of severe odynophagia. He avoids


eating and drinking because of the intense pain, and he has lost a significant
amount of weight. Which of the following is TRUE?
(A) Esophagectomy is the treatment of choice.
(B) Cancer is the only condition that can explain these findings.
(C) UGI series is not useful.
(D) Candida is the most common cause of infectious esophagitis.
(E) Esophageal candidiasis is almost certain if the patient has oral thrush.

150. A 54-year-old man presents with a massive UGI bleed. After resuscitation,
endoscopy is performed. No esophageal varices, gastritis, or gastric ulcers are
seen. After copious irrigation, a pinpoint lesion is seen near the GE junction.
What can be said about this lesion?
(A) It is a carcinoid.
(B) It is related to alcohol use.
(C) It is exclusively a mucosal lesion.
(D) Surgery if first-line therapy.
(E) Bleeding is from a submucosal vessel

147 148 149 150


C C D E
145

151. A patient has compressive symptoms of the esophagus. He has a barium


esophagram that shows posterior extrinsic compression of the esophagus. Which
of the following is true?
(A) Vascular rings are acquired atherosclerotic lesions.
(B) Both the trachea and esophagus can be affected by vascular rings.
(C) The two most common types of complete vascular rings are double aortic arch and left aortic
arch.
(D) There is no role for Echo and Doppler.
(E) Surgery involves division of the esophagus.

152. A 70-year-old man has surgery for an abdominal aortic aneurysm. About 1
month later the patient presents with a massive UGI bleed. Which of the following
statements is TRUE?
(A) He should be given PPLs and observed in the intensive care unit.
(B) Most aortoenteric fistulas are primary.
(C) Most aortoenteric fistulas occur between the aorta and duodenum.
(D) It is not improtant to separate the aorta from the eosphagus after aortic surgery.
(E) This condition is always fatal.

151 152
B C
146

ORTHOPEDICS

1. First bone to ossify

a. Femur.
b. Tibia.
c. Clavicle.
d. Sternum.

2. Which of the following fractures are more common to be impacted


the reaction is:

a. Neck femur fracture.


b. Humeral supracondylar fracture.
c. Colle's fracture.
d. Clavicle fracture.

3. In stage of primary callus in bone healing

a. Acidic.
b. Alkaline.
c. Neutral.
d. Any of the above.

4. Which of the following ore factors affecting bone healing:

a. Type of bone.
b. lntra-articular fracture.
c. Drugs.
d. Surgical intervention.
e. All of the above

1 2 3 4
C C B E
147

5. As regards nonunion all are correct except:

a. Diagnosed if there is no healing after 6 months.


b. Characterized by thinning of bone ends in avascular nonunion.
c. Should be treated conservatively in avascular nonunion.
d. May results from excess union triols.
e. Is rare in fractures of highly vascularised bones, e.g. clavicle.

6. Malunion may result in the following except:

a. Angulations.
b. Rotation.
c. Elongation.
d. Shortening.

7. As regards open fracture all are correct except:

a. Gustilo et al classification is useful in determining method of fixation.


b. Wound debridement means removal of foreign bodies and devitalized
tissues.
c. Closure of skin wounds is always indicated.
d. All bluish non contracting non bleeding muscles should be excised.

8. O.R.I.F is indicated in the following except:

a. Unstable fractures.
b. Open fracture.
c. With neurovascular injuries (when exploration is needed).
d. Pathological fractures.

5 6 7 8
C C C B
148

9. The following fractures ore more common in postmenopausal


females except:
a. Fracture surgical neck of humours.
b. Fracture neck femur.
c. Supracondylar fracture humours.
d. Colle's fracture.

10. Delayed union:

a. May be caused by infection.


b. Systemic steroids may cause delayed union.
c. ls diagnosed when there is non-union after 1.5 times of the expected
time for union.
d. ls treated by treatment of the cause + rigid fixation.
e. All of the above.

11. The followings ore complications of fracture except:

a. Neurogenic shock.
b. Stroke.
c. Fat embolism.
d. Renal failure.
e. Sepsis.

12. The following types of shock may complicate femoral fracture


except:

a. Hypovolemic.
b. Neurogenic.
c. Carcinogenic.
d. Septic.

9 10 11 12
C E B C
149

13. Which statement is unique regarding Sudek's atrophy:

a. Occurs most common after wrist and ankle injuries.


b. ls characterized by severe pain and stiffness.
c. Followed by osteoarthritis of the near joints.
d. May require sympathectomy.
e. ls a type of osteodystrophy.

14. Immobilization of fractures of long bones should include

a. Fractured bone only


b. Joint involved in the fracture
c. Proximal joint
d. Both proximal and distal joints
e. Distal joint

15. Causes of gangrene after fracture in a limb do not include:

a. Direct crushing of the tissues


b. injury to the main vessels
c. Tight plasters
d. Septic infection
e. Clostridial infection

16. The most common site of fracture clavicle is:

a. Middle.
b. Medial end.
c. Between middle and lateral thirds.
d. Between middle and medial thirds.

13 14 15 16
C D D c
150

17. Most common complication of fracture clavicle is:

a. Brachial plexus injury.


b. Shoulder stiffness.
c. Mal union (of no functional significance).
d. None of the above.

18. The most common type of shoulder dislocation is:

a. Posterior.
b. Anterior.
c. lnferior.
d. Superior.

19. The commonest complication of anterior shoulder dislocation is:

a. Axillary artery injury.


b. Rotator cuff tear.
c. Recurrent dislocation.
d. None of the above.

20. The treatment of recurrent shoulder dislocation is:

a. Kocher's method.
b. Hippocrotic method.
c. Bonkort's operotion.
d. Arthrodesis.

17 18 19 20
C B C c
151

21. Which fracture description does not match its name?


A Lisfranc = dorsal dislocation of tarsometatarsal joint +/- # 1st
cuneiform or 2nd MT
B Monteggia = # proximal third ulna and dislocation of radial head
C Clay-shoveller’s = avulsion of C3, 4,5 spinous processes
D Bennett’s = intra-articular # base of thumb metacarpal and proximal,
radial and dorsal displacement of distal fragment

22. Which of the following is FALSE regarding C-spine injuries?


A Hangman’s # is a hyperextension injury
B Anterior wedge # is usually mechanically stable
C Anterior tear-drop # is nearly always mechanically unstable
D C2 and C3 are the most commonly injured vertebra.

23. Regarding knee injuries, which of the following is TRUE?


A Segond fractures are important markers of PCL disruption
B Fracture types I, II and III involve the medial tibial plateau with
increasing articular depression
C Medial tibial plateau fractures are twice as common as lateral fractures
D Lateral plateau fractures are associated with ACL and MCL disruption

24. Regarding forearm fractures, which of the following is TRUE?


A Radial neck fractures with up to 30 degrees tilt can be managed
conservatively
B Post interosseous nerve injury is common with Monteggia fracture
dislocation
C X-rays in Galeazzi injury shows a radial fracture with dorsolateral tilt
D Radial nerve injury is common in Colles fracture

21 22 23 24
c d d b
152

25. Regarding forearm fractures


A Bartons fracture involves the distal radio-ulnar joint
B Monteggia fracture dislocation involves fracture of the radius
proximally and distal radio-ulnar joint disruption
C Monteggia fracture requires POP application in supination
D Galeazzi fractures require fixation of the related joint disruption in
most cases as well as ORIF of the bone fracture

26. The following statements are true EXCEPT


A Volkmann’s ischaemic contracture of the fore-arm and hand may
develop despite wrist pulses being present and the forearm skin being
well perfused
B ‘Fat pad’ sign is helpful for diagnosing occult #’s outside the elbow
joint capsule
C On the lateral XR of the elbow, the radio-capitellar line should intersect
the middle 1/3 of the capitellum
D 25 % of supra-condylar #’s are undisplaced and is the most common #
involving the elbow in children

27. In which area do clavicular fractures most commonly occur?


A Inner 1/3.
B Junction of inner 1/3 and middle 1/3.
C Proximal end
D Junction of middle 1/3 and outer 1/3.

28. Which of the following regarding scapular fractures is FALSE?


A Most commonly involve the blade.
B Are commonly overlooked.
C Are associated with thoracic injury in 80% of cases.
D Commonly managed non-operatively.

25 26 27 28
c b d c
153

29. Which of the following is TRUE with regards to knee injuries?


A Usually a varus force with axial loading will produce a lateral condylar
fracture of the tibia
B Elderly patients with a lateral tibial plateau fracture usually require a
bone scan as part of their investigation
C A Segond fracture is a marker of collateral ligament injury and ACL
disruption
D Popliteal artery is relatively protected from injury due to its anatomical
position

30. In regard to elbow dislocation, which of the following is FALSE?


A It is associated with significant soft tissue injury and resultant
instability.
B It is one of the three most common large joint dislocations – the others
being glenohumeral and patellofemoral
C Patients typically present with their arm held in near full extension
D The commonest associated neurovascular injury is to the Ulnar nerve.

31. In the child with Slipped Upper Femoral Epiphysis (SUFE),


which statement is FALSE?
A SUFE is a Salter Harris type fracture.
B Bilateral SUFE in a child under 10 years of age is more likely to be
associated with endocrine / metabolic disorders.
C The ‘Klein line’ is helpful in diagnosing SUFE
D A ‘Frog leg’ plain XR view of the hips is a helpful investigation to
diagnose SUFE

32. In the child with Leg-Calve-Perthes Disease (LCPD), which


statement is TRUE?
A It is a form of capital femoral epiphysis avascular necrosis secondary to
trauma
B CT imaging is preferred above plain ‘frog leg’ XR’s of the hips
C Patients with suspected LCPD disease require admission for further in
patient management.
D Is clinically indistinguishable from acute synovitis

29 30 31 32
c c d d
154

33. Features seen on a child’s cervical spine XRay that are different
from an adult’s include all of the following EXCEPT
A Anterior pseudosubluxation C2 on C3 can be normal until 8 years
B Increased depth of anterior soft tissue space
C Predental space (C1) up to 5mm
D Prominent lordosis

34. Which of the following statement regarding femoral neck


fractures is TRUE?
A They are more common in men
B Intracapsular fractures are more common than extracapsular fractures
C Displaced fractures should be treated with skin traction in the ED
D Intermittent catheterization is preferable to indwelling catheters

35. Radial head fractures are commonly associated with all of the
following EXCEPT
A Lateral collateral ligament injury
B Medial collateral ligament injury
C Coronoid fracture
D Olecranon fracture

36. With regards to isolated medial epicondyle fractures, which is


FALSE?
A Patients present with pain on flexion of the digits
B They are extraarticular injuries
C The medial epicondyle may become entrapped in the joint space in
Medial
Collateral ligament tears.
D They affect adults more commonly than children

33 34 35 36
d d a d
155

37. Which association is the least likely?


A Brachial plexus injury and fracture of the coracoid process
B Pseudosubluxation in proximal humeral fractures
C Wrist drop in Type III supracondylar fractures
D Volkmanns Ischaemic Contracture in supracondylar fractures

38. Which is FALSE regarding supracondylar fractures?


A Posterior displacement of the distal fragment are more common than
anterior displacement
B Fat pad sign may be negative in severe supracondylar fractures
involving capsular rupture
C Injuries to the anterior interosseous nerve are easily missed and are
detected by an inability to use FDP on the radial side
D An absent radial pulse as an isolated finding generally warrants urgent
surgical exploration

39. Regarding arm injuries which of the following is TRUE?


A Radial neck fractures with >10° angulation should be reduced.
B Colles fractures in post-menopausal women require only orthopaedic
follow-up.
C A Barton fracture with minimal displacement will require operative
reduction.
D Galeazzi fracture should not be diagnosed when there is <4mm of
distal radio-ulnar separation.

40. In knee injuries which of the following is TRUE?


A A common triad of injuries is ACL/MCL and medial meniscus.
B Proximal tib-fib dislocation occurs with a twisting force to the
extended knee.
C Tears of the patellar tendon need repair due to the high stress of
quadriceps contraction.
D 60% of adolescents with a knee haemarthrosis on x-ray will have an
osteochondral fracture.

37 38 39 40
c d c d
156

41. Which of the following is FALSE Regarding the systemic form of


Juvenile Chronic Arthritis, (Stills disease)?
A A temperature of 39.5 for two weeks is required for the diagnosis of
the systemic form
B Pericarditis is an association
C Lymphadenopathy is common
D The child generally appears well

42. Regarding Paediatric Arthritis,


A The arthritis of psoriatic arthritis always follows the skin lesions
B Enthesitis is associated with HLAB27
C Iritis is associated particularly with the polyarticular form
D Constitutional features are common with the pauci-articular form

43. Regarding hip injuries, which is TRUE?


A Garden I and II fractures are non displaced fractures and can be
managed conservatively
B Intracapsular fractures are more common than extracapsular fractures
C The hip is least stable when it is flexed and abducted
D Superior femoral head fractures are often associated with anterior hip
dislocations

44. Regarding knee injuries, which is FALSE?


A Lateral tibial plateau fractures are associated with MCL and ACL
injury
B The knee joint is the most commonly injured joint in the body
C Patella often dislocated laterally due to the weakening of the vastus
lateralis tendon insertion
D Dislocation of the knee is associated with popliteal artery and tibial
nerve injury

41 42 43 44
d b d c
157

45. Which of the following statements is FALSE?


A The tibia shaft is the most commonly fractured long bone
B Lateral malleolar fractures are the commonest ankle fracture
C The knee is the largest and most complicated joint
D Non-union is the most common complication of hip fractures

46. Regarding forearm fractures which is FALSE?


A All Galeazzi fracture-dislocations require surgery
B All Monteggia fracture-dislocations require ORIF
C Radial head fractures with up to 200 tilts can be managed
conservatively
D Smith’s fracture is managed with below elbow plaster with wrist in
pronation and full dorsiflexion

47. Regarding Segond fractures which is TRUE?


A Small avulsio-fracture proximal medial tibia
B Signifies tear of the menisco-tibial attachment of the middle 1/3 of the
medial capsular ligament
C All have anterior cruciate tears
D Few have meniscal tears

48. Which of the following is FALSE in regard to Plaster of Paris?


A Created by heating Gypsum at 3500 C to form Calcium sulphate
hemihydrate
B Hardens in an exothermic reaction when exposed to water
C A forearm cast will be fully dry by 36 hours post-application
D A Forearm cast will develop 50% of the final full strength by one hour
postapplication

45 46 47 48
e d c e
158

49. Which of the following is FALSE in regard to the Mangled


Extremity Severity Score (MESS)?
A The presence and degree of other major injuries is a parameter that
contributes to the final score
B Limb ischemia is a parameter that contributes to the final score
C A score of 7 or more is associated with 100% chance of amputation
D The degree of soft tissue injury is a parameter that contributes to the
final score

50. Which of the following is TRUE?


A Clinical examination is more sensitive than x-ray in picking up a
fracture
B A 'fat pad' sign of a fracture is less reliable in obese individuals
C x-ray is less specific than clinical examination in regards to fracture
diagnosis
D X-rays with 2 views is 95% sensitive and specific for major joint
dislocation

51. Which philosopher is MOST likely to be useful to FACEM


examination candidates?
A Hegel
B Kant
C Nietzsche
D Fromm

52. Which of the following pairs is CORRECT?


A Femoral head fractures – Pipkin classification
B Femoral head fractures -Gardiner classification
C Patellar fractures – Hohl classification
D Ankle factures – Boehler classification

49 50 51 52
a a b a
159

53. Which of the following is CORRECT when treating neck of


femur fractures?
A Alignment maintained by pre operative skin traction helps post
operative mobility
B Indwelling urinary catheters are not associated with post operative
infection
C Antibiotics at time of anaesthetic induction reduce post operative
infection rates
D Patients do not need nursing on pressure mattresses in the pre operative
period where surgery is conducted within 24 hrs

54. Which of the following fracture-eponym combinations is NOT


correct?
A Tillaux # - avulsion # of tibia at inferior tibiofibular joint
B Essex-Lopresti – fractured radial head and dislocated distal radioulnar
joint
C Hume # - fractured distal radius with dislocation of distal radioulnar
joint
D Bennet # - # dislocation at the base of the 1st metacarpal

55. With regard to scaphoid #’s which statement is TRUE?


A Risk of # if not seen on initial X-ray is 10-20%
B Most common location is proximal pole ≈ 50%
C Can get AVN of tubercle #s
D MRI sensitivity approaches 100%

56. Which of the following is TRUE?


A Most tibial plateau fractures involve the medial plateau
B A Segond fracture suggests PCL disruption and anterolateral rotator
instability
C The younger the age of onset of Perthes disease, the better the
prognosis
D The more proximal the scaphoid fracture, the lower the incidence of
avascular necrosis

53 54 55 56
c c d c
160

57. Which of the following is TRUE?


A An ipsilateral Pelvic fracture is commonly associated with an L5
transverse process fracture
B The posterior column of the thoracolumbar spine involves the posterior
longitudinal ligament
C Intracapsular hip fractures are more common than extracapsular
fractures
D The Hawkins classification can be used for hip fractures

58. In regards to Slipped Upper Femoral Epiphysis (SUFE), which is


CORRECT?
A More common in females
B In chronic slip passive flexion is associated with external rotation
C In unstable SUFE patient can still weight bear
D More common in children under the age of 10

59. What is the key event required for a Triplane Ankle Fracture to
Occur?
A Inversion Injury
B Ossification of Epiphysis
C Axial Loading Injury
D Partial Fusion of Growth Plate

60. Regarding Nerve Injuries that characteristically accompany


certain Orthopaedic Injuries which pairing is CORRECT?
A Lunate dislocation - Ulnar nerve
B Femoral Shaft Fracture- Femoral Nerve
C Shoulder Dislocation- Radial Nerve
D Knee Dislocation- Peroneal Nerve

57 58 59 60
a b d d
161

61. Regarding Compartment Syndrome which bone fracture is


MOST commonly implicated?
A Femur
B Radius
C Tibia
D Humerus

62. Which is a CORRECT indication for replantation post


amputation?
A Single digit amputations proximal to the FDS insertion
B Sharp amputations with minimal to moderate avulsion proximal to the
elbow
C Multiple level amputations
D Self- Inflicted amputations

63. Which of the following regarding elbow x rays is FALSE?


A The anterior humeral line passing through the middle third of the
capitellum indicates a supracondylar fracture
B A visible posterior fat indicates an intracapsular fracture
C A small anterior fat pad is normal
D The radio-capitellar line should pass through the middle 1/3 of the
capitellum

64. Which of the following statements regarding ankle fractures is


TRUE?
A A Maisonneuve fracture is produced by forced external rotation
B The Pott’s system is based on the level of the fibula fracture
C The Weber classification is based on the number of malleoli involved
D The Ottawa ankle rules have not been prospectively validated in
children

61 62 63 64
c b a a
162

65. All of the following is true with regards to Perthes disease,


EXCEPT:
A All children with irritable hips should be referred to an orthopaedic
surgeon
B It is mostly unilateral
C It occurs more often in malnourished children
D When followed up 20 years later; most patients are pain free and active

66. In regard to slipped upper femoral epiphysis (SUFE):


A On an AP X-ray, a line drawn up along the superior femoral neck will
not intersect the femoral head
B Obesity does not play a role in its pathogenesis
C Ultrasound has low sensitivity to detect slip of the epiphysis
D Most patients will develop avascular necrosis of the hip

65 66
a a
163

Neurosurgery
1. A 43-year-old man experiences lower back pain after lifting a heavy object off the
ground. The following morning, he notices that the pain has begun to radiate down the
posterolateral aspect of the right leg and across the top of the foot to the big toe. The pain
is severe, electric in quality, associated with paresthesia over the same distribution, and
made worse by coughing. On examination, it is found that he has an area of diminished
sensation to pinprick over the dorsum of the right foot and mild weakness in his right
extensor hallucis longus muscle. The deep tendon reflexes are all intact. What is the most
likely diagnosis?
(A) Lumbar spinal fracture with compression of the caudaequina
(B) Herniated lumbar disk on the right at the level of L4–L5
(C) Herniated lumbar disk on the left at the level of L4–L5
(D) Herniated lumbar disk on the right at the level of S1–S2
(E) Intermittent claudication

2. A 48-year-old woman has a lower back pain and hypoesthesia in the left S1 dermatomal
distribution (left calf and lateral left foot). What is the most likely cause?
(A) A lesion at the right L4–L5 interspace
(B) Pathology where the nerve exits the spinal canal immediately above the pedicle of S3
vertebra
(C) A herniated nucleus pulposus
(D) Compression by the L5 lamina
(E) A lesion outside the vertebral column

3. A 35-year-old secretary complains of severe pain in the neck that radiates down the
right arm. The pain is electric in quality and affects specifically the radial aspect of the right
forearm and the thumb. She also describes numbness and paresthesia over the same
distribution. On physical examination, she is found to have an area of diminished
sensation to pinprick over the right wrist and thumb. The right biceps tendon reflex is
diminished, but there is no loss of muscle strength. She has right C5–C6 disk compression
and radiculopathy affecting which of the following?
(A) The right C4 root
(B) The right C4 mixed spinal nerve
(C) The right C4 anterior primary rami
(D) The right C6 root
(E) The right C6 spinal ganglion

4. A 57-year-old woman presents to the emergency department with new-onset seizures.


She was witnessed by her husband to have a generalized seizure lasting approximately 1
minute. She has smoked 1 pack of cigarettes a day for over 40 years. In the past 3 months,
she has lost 25 lb in weight. On examination, she appears thin and nervous but findings on
her neurologic examination are otherwise essentially within normal limits. Plain
radiographs of the chest obtained in the emergency department show a 4-cm nodule in
the upper lobe of her right lung. To exclude cerebral metastasis as a cause of her seizure,
what should the next test requested be?
(A) An electroencephalogram (EEG)
(B) A CT scan of the brain with intravenous contrast
(C) Aspinaltap to measure opening pressureand obtain CSF for cytology
(D) An MRI of the brain with intravenous contrast
(E) Doppler ultrasound
1 2 3 4
B C D D
164

5. A 64-year-old man presents with headache and left-sided upper extremity


weakness. The MRI findings suggest that this is a glioblastomamultiforme. This is
because the tumor exhibits which of the following?
(A) It is regular in shape.
(B) It is well demarcated from surrounding brain tissue.
(C) It shows a ring pattern of enhancement with intravenous contrast and has a
nonenhancing necrotic center.
(D) It shows an absence of surrounding white-matter edema.
(E) It arises from the carotid body

6. A 63-year-old woman presents for workup to determine the reason for a gradual
hearing loss over approximately 5 years and intermittent tinnitus over the last
several months. Findings on physical and neurologic examination are
entirely within normal limits, except for the presence of sensorineural hearing loss
in the left ear. She has no cranial nerve deficits. An MRI of the brain with
gadolinium reveals the presence of an extra-axial tumor in the region
of the left cerebella-pontine angle. What is the most likely diagnosis?
(A) Epidermoid tumor (cholesteatoma)
(B) Glioblastomamultiforme
(C) Meningioma
(D) Acoustic neuroma
(E) Glomus tumor

7. In the management of a 64-year-old woman struck by a car, mannitol is given to


do which of the following?
(A) Increase CSF formation
(B) Increase the respiratory rate
(C) Increase the pulse rate
(D) Replace extensive fluid loss
(E) Lower raised ICP
8. Following a sudden impact in an accident, the 34-year-old race car driver
becomes unconscious and is admitted to the hospital. A CT scan is performed, and
a right space-occupying lesion is noted (Fig. 11–1). What is the most likely
diagnosis?
(A) Corpus callosum injury
(B) Pituitary apoplexia
(C) Acute subdural hematoma
(D) Acute epidural hematoma
(E) Chronic subdural hematoma

5 6 7 8
c D E C
165

9. A 64-year-old woman complains of gait imbalance, headache and deterioration


of mental status over the past several months. Her vision is normal. A CT scan
reveals hydrocephalus, but the lumbar puncture pressure is unexpectedly low.
What does she have?
(A) Meningitis
(B) Normal-pressure hydrocephalus
(C) Sigmoid sinus thrombosis
(D) Echinococcus
(E) Glioblastoma multiforme

10. A 23-year-old woman complains of progressive loss of vision and papilledema.


Investigations show normal findings on CT scan. A lumbar puncture shows marked
elevation of pressure. What is the most likely diagnosis?
(A) Pseudotumorcerebri
(B) Corpus cavernous thrombosis
(C) Cavernous sinus thrombosis
(D) Retinoblastoma
(E) Chordoma

11. During her eighth month of pregnancy, a 29-year-old woman is noted to have
hydramnios. Further testing shows anencephalus. In this case hydramnios is
caused by which of the following?
(A) Impairment of the fetus’s swallowing mechanism
(B) Tumor of the fetus’s brain
(C) A secretory peptide from the placenta
(D) Excess antidiuretic hormone (ADH) from the fetus
(E) Renal agenesis

12. A 28-year-old man presents with a history of chronic headache. The headache
is intermittent, severe, poorly localized, and most often present when he arises in
the morning. He suffered a severe blow to the head and sustained a skull fracture
at the age of 15. Findings on his physical and neurologic examinations are within
normal limits. An MRI of the brain with gadolinium reveals the presence of a large,
nonenhancing extra-axial cyst in the region of
the right temporal tip. This most likely represents which of the following?
(A) An arachnoid cyst
(B) A cystic astrocytoma
(C) Rathke’s cleft cyst
(D) A Dandy-Walker cyst (failure of proper formation of the foramina of Lushka
and Magendie)
(E) Polycystic disease

9 10 11 12
B A A A
166

13. A 15-year-old boy complains of right-sided weakness and gait impairment. ACT
scan shows a large, nonenhancing cyst in the posterior cranial fossa, with an
enhancing tumor nodule in the left cerebellum. What is the most likely diagnosis?
(A) An arachnoid cyst
(B) A cystic astrocytoma
(C) Rathke’s cleft cyst
(D) Glioblastomamultiforme
(E) A large sebaceous cyst ans:B

14. A 43-year-old man is treated with pyridostigmine for facial, ocular, and
pharyngeal weakness due to myasthenia gravis. Which statement is true of
pyridostigmine?
(A) It is unrelated to neostigmine.
(B) It has far more side effects than neostigmine.
(C) Pyridostigmine and neostigmine reverse depolarizing neuromuscular blockade.
(D) It causes greater muscarinic effect than neostigmine.
(E) It is an anticholinesterase agentans:E

15. A 17-year-old male presents with 3-month history of headache, weight gain,
decreased concentration, polyuria, and polydypsia. His headaches are mostly in
morning and involvesthe frontal region. On examination he was found to have
bitemporal visual field defect and no facial hair. MRI scan revealed a suprasellar
partially calcified cystic lesion with displacement of optic chiasm. The most likely
pathology is:
(A) Giant aneurysm of carotid artery
(B) Pituitary macroadenoma
(C) Glioblastomamultiforme
(D) Craniopharyngioma
(E) Testicular metastasisans:d

16. He underwent a craniotomy for resection of his lesion. Twelve hours


postoperatively, he developed diuresis of over 500 mL/h. The diagnosis of (DI) was
entertained. What laboratory findings are most consistent with the clinical
impression?
(A) Urine specific gravity of over 1010
(B) Serum sodium of less than 135
(C) Decreased both serum and urine osmoloaity
(D) Increased serum osmolality and decreased urine osmolality
(E) Increased both serum and urine osmolalities

13 14 15 16
B E D D
167

17. A 45-year-old woman was brought to emergency department for sudden onset
of severe headache associated with photophobia, nausea, and transient loss of
consciousness. On examination, she is awake and alert with normal cranial nerve
function. She also exhibits normal muscle strength and sensation.Her past medical
history is significant for sickle cell disease (SCD) and hypertension. CT scan confirms
the diagnosis of SAH without any intraparenchymal abnormality.
What is the least likely cause of SAH?
(A) Aneurysmal bleed
(B) Sickle cell angiopathy
(C) Arteriovenous malformation (AVM)
(D) Hemorrhagic meningioma
(E) Blood dyscrasia

18. What is the most definitive diagnostic test in this condition?


(A) CT angiography
(B) Magnetic resonance angiography (MRA)
(C) Cerebral angiogram
(D) MR spectroscopy
(E) Positron emission tomography (PET) Scan

19. What is the most likely complication of angiography in this patient?


(A) Cerebral stroke
(B) Aneurysmal rupture
(C) Increased intracranial pressure
(D) Vascular wall damage
(E) Sickle cell crisis

17 18 19
D C E
173

Cardiothtoracic
l. About etiology of pneumothorax:
a- Spontaneous pneumothorax is always secondary to lung disease
b- Accidental pneumothorax needs a penetrating injury
c- Mechanical ventilation can result in pneumothorax
d- Central venous line insertion is not a reported cause

2. All of the following ore radiological features of pneumothorax


except:
a- Lung collapse on some side
b- Jet black opacity on some side
c- Shift of cardiac shadow to some side
d- Flat diaphragm on some side

3. The main line of treatment in closed pneumothorax without


mediastinal shift is:
a- Chest tube
b- Mechanical ventilation
c- Needle aspiration
d- Conservative treatment

4. The following aid(s) in diagnosis of open pneumothorax


a- Cyanosis
b- Congested non-pulsating neck veins
c- Harsh whistling sound following trauma
d- All of the above

1. C

2. C

3. D

4. D
174

5. For treatment of pneumothorax intercostals tube is inserted in


a- 5th space of parasternal line
b- 2nd space of parasternal line
c- 2nd space of midclavicular line
d- Any of the above

6. Circulatory failure in open pneumothorax is due to the following


except:
a- Mediastinal flutter
b- Exaggerated negativity of intrathoracic pressure
c- Associated great vessel injury
d- Diminished cardiac function due to respiration failure

7. All of the following are differential diagnoses for sudden chest pain
without shock except:
a- Tension pneumothorax
b- Massive pulmonary embolism
c- Massive myocardial infarction
d- Flail chest

8. The most life threatening chest wall lesion is:


a- Emphysema
b- Tension pneumothorax
c- Flail chest
d- Hemothorax

5. C

6 B
7 D

8 B
175

9. All ore causes of hemothorax except:


a- Trauma
b- Bronchogenic carcinoma
c- Rupture of emphysematous bullae
d- Anticoagulant therapy
e- Hemophilia

10. About treatment of hemothorax all are correct except:


a- there is no need to decorticate of all
b- Resuscitation and stabilization of general condition has the priority
c- Encysted hemothorax is on indication of thoracotomy
d- Fibrinolysis may prevent clotting giving chance to conservation

11. As regards pathology of hemothorax all are correct except:


a- Blood always coagulates completely
b- Blood is defibrinatned by continuous diaphragmatic motion
c- Hemothorax con result in large hydrothorax by transudation
d- lt can be complicated by fibrothorax or emphysema

12. In flail chest limitation of movement of flail part is done to:


a- Decrease risk of injury to intercostals structure
b- Prevent paradoxical movement and pendulum respiration
c- Prevent mediastinal flutter
d- Decreases pain
e- All of the above

9 C
l0 A
11 A
12 E
176

13. The following organisms cause a sever type of empyema


complicated by dense adhesions:
a- Staph
b- Streptococci
c- Pneumococci
d- E.Coli

14. All of the following ore indications of open surgical drainage of pus
in acute empyema except:
a- thick pus
b- Rapid recollection of pus
c- Streptococcal empyema
d- Extensive conditions

15. As regards Empyema necessitans all are correct except:


a- ls an encysted empyema
b- Gives expansible impulse on cough
c- Can perforate skin leading to skin sinus
d- is a subcutaneous abscess communicated with empyemic pleura

16. The following ore signs of chronic empyema except:


a- Paroxysmal fever and chills
b- Flattening of diaphragm
c- Sinus in chest wall discharging pus
d- Scoliosis

13 C
14 C
15 A
16 b
177

17. All of the following ore complications of chronic empyema except:


a- Empyema necessitans
b- Amyloidosis
c- Mesothelioma
d- Toxic arthritis

18. The following factor(s) predispose to chronic empyema:


a- Pulmonary tuberculosis
b- DM
c- Inadequate drainage
d- Bronchogenic carcinoma
e- All of the above

19. About bronchogenic carcinoma all are correct except:


a. Common in old smoker moles
b. Con present by any chest symptom
c. Usually inoperable of diagnosis
d. 5 year old survival rate is 60%

20. During the insertion of a subclavian catheter for hyperelimentation


in a patient with Crohn's disease the patient become dyspneic with a
respiratory role of 32/min, pulse rate of l20/min. and drop of the B.P.
to 80/60. The appropriate immediate action is:
a. Chest X-ray,
b. Lung scan.
c. lntubation ond mechanical ventilation.
d. Chest tube.
e. Vasopressors.

17 C
18 E
19 D
20 D
178

21. The treatment of choice in flail chest is:


a. Tracheotomy.
b. lntercostal nerve block.
c. Adhesive strapping.
d. Elevation of broken ribs with towel clips.
e. Positive pressure ventilation.

22. Open chest wounds are characterized by the following except that
they:
a. May cause "pendulum respiration".
b. May produce "mediastinal flutter.
c. May be associated with abdominal injuries.
d. Need immediate sealing of the wound.
e. Always require thoracotomy for definitive treatment.

23. Which of the following statements about traumatic haemothorax is


untrue?
a. May absorb spontaneously.
b. May be associated with signs of internal hemorrhage.
c. Cannot be diagnosed radiologically with less than 500 ml of blood.
d. Should be treated by repeated needle aspirations.
e. May require decortications of both lung and pleura.

24. The following statements about spontaneous pneumothorax are


true except that it :
A. May occur in the absence of any pulmonary lesion.
B. is readily diagnosed clinically.
C. Usually resolves spontaneously.
D. May require intercostal catheter drainage.
E .Usually requires exploratory thoracotomy.

21 E
22 E
23 A
24 E
179

25. ln a case of pleural effusion. The following statements are correct


except that:
a. The diagnosis can be made with as little as 100 ml of fluid.
b. The trachea may be displaced to the opposite side.
c. Aspiration of blood-stained fluid is highly suspicious of bronchogenic
carcinoma.
d. Open biopsy of the pleura is necessary.
e. Rapid aspiration of the fluid is not advisable.

26. Pleural fluid with the appearance of "anchovy sauce” pus is


characteristic of:
a. E. coli.
b. Staph. aureus.
c. Echinococcus.
d. Entomeba histolytica.
e. Hemothorax.

27. The primary treatment of Tuberculous empyema is:


a. Systemic chemotherapy.
b. Aspiration.
c. Closed tube drainage.
d. Open drainage.

25 A
26 D
27 A
180

Urology

l. Evolution of renal functions before IVU is best done by measuring:


a. Urine specific gravity.
b. Urea level.
c. Creatinine level.
d. None of the above.

2. The following is incorrect regarding urinary investigations:


a. With IVP the kidney may not be visualized if congenitally absent,
surgically removed.
b. The best advantage in performing CT in urinary tumors is to assess
exact extent and invasion.
c. U/S is indicated for any renal cyst.
d. IVP is not indicated after any abdominal trauma.
e. isotope imaging show fractional renal function.

3. Benign cyst by U/S all are correct except:


o. Is smooth containing clear fluid.
b. Leaves residual moss after aspiration.
c. No rapid recollection.
d. All ore true.

4. Urinary system investigations:


a. IVP is safe in renal impairment.
b. Cystogrom is the most diagnostic method of urinary bladder
carcinoma.
c. Vesicoureteric reflux is diagnosed by MCU.
d. Cystoscope of bladder carcinoma shows pseudotubercle, sandy
patches and ulceration.

1 2 3 4
b d b c
181

5. As regard polycystic kidney all are correct except:


a. Hereditary condition.
b. Infantile type is a rare condition inherited as autosomal recessive.
c. Moy cause renal hypertension.
d. Con not be complicated by infections (sterile condition).

6. As regard inheritance of polycystic kidney all are correct except:


a. Adult type (autosomal dominant.)
b. infantile type (autosomal recessive.)
c. Percentage of inheritance is l0%.
d. May be associated with other cystic changes.

7. Solitary cyst of the kidney may contain the following except:


a. Clear fluid.
b. Cholesterol crystals.
c. May contain altered blood.
d. Pyogenic material if infected.

8. As regard ectopia vesica all are correct except:


a. It is more common in females.
b. it may lead to concern bladder.
c. Usually present with ambiguous genitalia (undefined sex).
d. Always complicated by recurrent UTI.

5 6 7 8
d c b a
182

9. Congenital bladder diverticulum differs from acquired one in the


following except:
a. Congenital is a true diverticulum.
b. Acquired is always solitary.
c. Congenital may be o port of anomalous urachus.
d. Acquired is always symptomatic.

10. Double ureter may present with:


o. Recurrent UTI in non diabetic patient.
b. Recurrent stone formation especially triple phosphate.
c. May be asymptomatic.
d. All of the above.

11. Ectopio vesico is common to be associated with all of the following


anomalies except:
a. Spina bifida.
b. Rudimentary penis.
c. Congenital heart disease.
d. Cleft palate.

12. In ectopia vesica:


a. absent anterior abdominal wall.
b. Absent anterior bladder wall.
c. Associated late rupture of cloacal membrane.
d. All of the above.

9 10 11 12
b d c d
183

13. About urinary bladder diverticulae all are true except:


a. May be congenital or acquired.
b. May be complicated by infection, hydronephrosis or even neoplasm.
c. Most commonly traction diverticuloe.
d. None of the above.

14. About posterior urethral valve all ore true except:


a. Is best diagnosed by urethroscope.
b. Treatment is most commonly by transurethral resection.
c. Causes acute retention of urine.
d. All of the above.

15. in penile hypospadius:


a. The urethro opens midwoy between the scrotum and the onus.
b. The prepuce is deficient superiorly.
c. The penis is curved downwards.
d. No treatment is required till the age of l0 years.
e. Circumcision should be done during the neonatal period.

16. Complications of rupture kidney include:


a. Shock.
b. Renal failure if solitary kidney.
c. A-V fistula.
d. Pseudohydronephrosis.
e. All of the above

13 14 15 16
c b c e
184

17. As regard incidence of rupture bladder all are correct except:


a. It is more common in moles.
b. lntraperitoneal rupture is more common.
c. Extraperitoneal is always associated with fracture pelvis.
d. All of the above.

18. Treatment of rupture bladder includes the following except:


a. Resuscitation.
b. Water tight repair without drain.
c. Urethral catheter for 2 weeks.
d. 2ry survey and treatment of associated injuries.

19. Conservative treatment of rupture kidney is indicated in:


a. Complete tear.
b. Deep tear.
c. lntraperitoneal rupture with minimal shock.
d. Retroperitoneal rupture with minimal shock.
20. Tear drop sign is seen in:
a. Under screen in plain x-ray in rupture kidney.
b. in IVP in rupture kidney.
c. in U/S in rupture kidney.
d. None of the above.

17 18 19 20
b b d b
185

21. Rupture urinary bladder:


a. Most commonly extraperitoneal.
b. More common in moles.
c. May cause hemoturia.
d. All of the above.

22. About rupture urethra:


a. Commonly is prostatic urethro.
b. Causes bleeding per rectum.
c. Fracture of pubic and ischeal romi causes rupture membranous
urethra.
d. The 1st step of treatment is usually urethral catheterization.

23. An absolute indication of surgical treatment of renal injury is:


a. 20% devitalized renal parenchyma.
b. Major urinary extravasation.
c. Vascular injury.
d. Expanding perineal hematoma.

24. The triad of signs of rupture urethra includes the following except:
a. Perineal hematoma
b. Bleeding per urethra
c. Retention of urine
d. Bleeding per rectum

21 22 23 24
d c d d
186

25. Male 25 years old arrived hospital in severe shock after a road
traffic accident. Abdominal examination revealed tenderness & rigidity
in the loin and fullness in the suprapubic region. He failed to pass any
urine. He should be suspected lo have:
a. Rupture of the spleen
b. Rupture of the left kidney
c. Retroperitoneal hematoma
d. Extraperitoneal rupture of the bladder
e. lntrapelvic rupture of the urethra

26. Renal tuberculosis all are correct except:


a. The kidney is affected usually by hematogenous spread.
b. Earliest symptom is frequency.
c. Sterile pyuria increase suspicion.
d. Renal calcification excludes T.B.
e. May result in autonephrectomy.

27. The following may be a presentation of perinephric abscess:


a. Fever.
b. Psoas spasm.
c. Hiccough.
d. Throbbing pain in loin increased with breathing.
e. All of the above

28. As regards types of pyonephrosis all are correct except:


a. In primary type the kidney may be hugely enlarged.
b. 2ry type is due to superimposed infection and chronic obstruction.
c. Closed pyonephrosis is on emergency condition.
d. Urine analysis may be -ve in closed type.

25 26 27 28
b d e a
187

29. Complications of urinary bilharziasis include the following except:


a. lnfertility.
b. Corpulmonale.
c. Hydronephrosis.
d. Squamous cell carcinoma.
e. All of the above

30. All of the following findings give sure diagnosis of renal TB except:
a. Acid fast, alchol fast bacilli on ZN stain.
b. Sterile pyuria.
c. +ve culture on lowenstein medium.
d. +ve culture on bactec medium.

31. About urinary bilharziasis:


a. Most common organism is Schistosoma mansoni.
b. May lead to transitional cell carcinoma.
c. Characteristic sandy patches on macroscopic exam.
d. Nonspecific histopathological pattern may be seen.

32. All of the following lesions in bilharziasis are precancerous except:


a. Cystitis cystica.
b. Leukoplakia.
c. Cystitis glandularis.
d. Sandy patches.

29 30 31 32
e b c d
188

33. Disease of urinary bladder course:


a. Initial hematuria.
b. Terminal hematuria.
c. Total hematuria.
d. Con not cause hematuria.

34. About urinary tract infection all are true except:


a. It is the most common cause of phosphate stone.
b. The most common organism is proteus merabiles.
c. Causes of stasis are very common predisposing factors.
d. More in females.

35. Regarding urinary bladder bilharziasis all ore true except:


a. Diagnosed by presence of sandy patches, pseudotubercles or
ulcerations in cystoscope.
b. Leads to urinary bladder carcinoma.
c. Causes total hematuria.
d. All of the above.

36. Radiological signs of perinephric abscess include the following


except:
a. Enlargement of the renal shadow on the plain film
b. Homolateral scoliosis
c. Obliteration of the psoas shadow
d. Elevation and fixation of the diaphragm
e. Positive mother's sign

33 34 35 36
b b c b
189

37. The following statements about pyonephrosis are correct except


that it
a. Is a chronic retention of infected urine in the kidney
b. Usually results from coincident infection and obstruction as in renal
calculi and pyelonephritis
c. ls characterized by dilatation of the pelvicalyceal system with
extensive excavation of the parenchyma
d. Presents with pain, swelling, fever and pyuria
e. ls best treated by primary nephrectomy

38. Renal tuberculosis should be suspected in presence of:


a. Sterile pyuria
b. Renal calcification
c. indistinct outline of papillae in IVP
d. Urinary frequency and hematuria
e. Each of the above

39. Presentations of urinary stones may be the following except:


a. Renal pelvic stone pain restricted to loin.
b. Upper ureter stone has no radiating pain.
c. Bladder neck stone pain radiating to neck of penis.
d. Urinary stone may be asymptomatic especially triphosphate stone.

40. The following are common sites of impaction of ureteric stone:


a. Pelviureteric junction.
b. Crossing of iliac arteries.
c. Crossing of brood ligaments in females or vas in males.
d. Ureteric orifice.
e. All of the above.

37 38 39 40
e e b e
190

41. Conservative treatment of urinary stone include:


a. Ample fluid intake.
b. Antibiotics.
c. Analgesics.
d. Acidification of urine.
e. All of the above.

42. The following type of stone is radiolucent in (KUB) films:


a. Calcium stones.
b. Urate stones.
c. Cyteine stones.
d. Oxalate stones.

43. All of the following are predisposing factors to Ca oxalate stone


except:
a. Hyperparathyroidism.
b. increase consumption of mongo, tomato and milk.
c. Bone secondaries.
d. Giant tumor lysis.
e. Prolonged recumbency.

44. The most symptomatic stone is:


a. Oxalate.
b. Urate.
c. Phosphate.
d. Cysteine.

41 42 43 44
e b d a
191

45. The least symptomatic stone is:


a. Oxalate.
b. Urate.
c. Phosphate.
d. Cysteine.

46. DD of ureteric stone (opacity) in x-ray film include:


a. Gall bladder stone on rt side.
b. Phlibolith.
c. Calcified LN.
d. All of the above.

47. The stone causing strangury must iritate:


a. Ureter.
b. Pelviureteric junction.
c. Trigone.
d. Ureteric orifice.

48. Regarding urinary stones all are true except:


a. Urate stones ore translucent.
b. Oxalate stone is spiky.
c. Triple phosphate stones consist of magnesium, calcium and sodium.
d. None of the above.

45 46 47 48
c d c c
192

49. Predisposing factors of urinary stones include:


a. Urinary bladder diverticulae.
b. Recurrent urinary tract infection.
c. Metabolic causes.
d. All of the above.

50. Renal calculus formation has been shown to be related to all of the
following except:
a. Vitamin D metabolism.
b. Urea splitting bacteria.
c. lmmobilization.
d. Stenosis of ureteropelvic junction.
e. Malabsorption.

51. A ureteric stone needs surgical removal when:


a. Enlarging
b. Obstructing
c. Too large to pass
d. Causing urinary infection
e. All of the above

52. Which of the following statements regarding ureteric colic is false:


a. The pain is restricted to the loin when the stone is coming out of the
kidney
b. Pain radiates to the groin when the stone is in upper ureter
c. In lower ureteric stones, pain radiates to the perineum
d. All of the above

49 50 51 52
d e e d
193

53. The main aim in emergency treatment of anuria is to do the


following except:
a. Relieve obstruction.
b. Prevent infection.
c. Relieve pain.
d. Detect type of stone and remove it.

54. As regards treatment of renal malignancies all are correct except:


a. Urinary bladder is removed during radical nephrectomy.
b. When unresectable radiotherapy and chemotherapy may be tried in
Wilm's tumor.
c. Chemotherapy and radiotherapy are not effective against RCC as it is
adenocarcinoma.
d. RTH is the 2nd line after surgery in RCC.
e. Survival rate after surgery in Wilm's tumor is 5O%.
f. Radical nephrectomy for renal tumors is best done transabdominal.

55. All of the following are presentations of BPH:


a. Asymptomatic.
b. Chronic retention.
c. Hematuria.
d. Chronic renal failure.
e. All of the above.

56. As regards Prostatectomy all are correct except:


a. Best way for benign condition is TURP.
b. Is indicated in all cases with BPH.
c. Radical prostatectomy is indicated in locally active odenocarcinoma.
d. Local latent adenocarcinoma follow up.
e. Metastasizing adenocarcinoma hormonal therapy.

53 54 55 56
d a e b
194

57. Risk factors of transitional cell carcinoma include all of the


following except:
a. Smoking.
b. Cyclophosphamide.
c. Pelvic irradiation.
d. Exposure to benzidine.
e. Exposure to shistosomiasis.

58. All of the following primary sites metastasize classically with


osteolytic lesion except:
a. Kidney.
b. Prostate.
c. Bladder.
d. Breast.
e. Lung.

59. As regard pathology of BPH:


a. It arises from submucous glands.
b. It leads to shortening and compression of urethra.
c. It is fibromyoadenoma.
d. All of the above.

60. As regard DRE BPH differs from cancer prostate in that:


a. Cancer prostate feel hard and irregular.
b. BPH feels smooth, firm and elastic.
c. In BPH rectal mucosa is freely mobile.
d. All of the above.

57 58 59 60
e b d d
195

61. As regard complications of prostate cancer:


a. Most common site for distant metastasis is vertebral column.
b. Urine retention is a late complication.
c. Most common sexual complication after prostatectomy is retrograde
ejaculation.
d. All of the above.

62. Diagnosis of urinary bladder carcinoma:


a. The most effective diagnostic tool is cystoscopy.
b. Best assessment of penetration is by contrast enhanced CT.
c. irregular filling defect in cystogram.
d. All of the above.

63. The commonest presentation of nephroblastoma:


a. Abdominal mass.
b. Hematuria.
c. Renal pain.
d. Pulmonary metastasis.
e. Polycythemia.

64. Carcinoma of bladder all are true except:


a. Is usually very painful even in early stages.
b. Is usually associated with hematuria.
c. Most often occur in the vault of the bladder.
d. Is best diagnosed by cystoscopy.

61 62 63 64
d d a c
196

65. Radiological findings of benign hypertrophy of the prostate include


all of the following except:
a. Filling defect at base of the bladder.
b. Diverticulae of the bladder.
c. Uretheral dilatation.
d. Ureteral dilatation.

66. The earliest symptom of benign prostatic hyperplasia is:


a. Dysuria.
b. Nocturnol frequency.
c. Hesitancy.
d. Post micturition drippling.
e. Urine retention.

67. Bladder irrigation during TUR prostatectomy may produce:


a. Hemolysis.
b. Hyponatremia.
c. Hypercalcemia.
d. Hyperkalemia.
e. Alkalosis.

68. A 60-year-old male presenting with painless hematuria gives a


history of two similar attacks during the last six months. He had no
pain, dysuria or frequency and examination revealed no abnormality.
He should be suspected to be suffering from:
a. Polycystic kidney
b. Renal calculus
c. Carcinoma of the bladder
d. Hypernephroma
e. Senile enlargement of the prostate

65 66 67 68
d b b d
197

69. The following statements concerning hypernephroma are true


except that it:
a. Is a carcinoma arising from renal tubular cells
b. Commonly arises in one pole of the kidney
c. Usually manifests itself by recurrent renal hematuria
d. Never causes pain
e. May form a palpable moss

70. Histologically benign prostatic hyperplasia is:


a. Fibroadenoma
b. Fibromyoadenoma
c. None of the above

71. The formation of middle lobe in BPH arises from which zone:
a. Transitional zone
b. Central zone
c. Peripheral zone
d. None of the above

72. Localized carcinoma of the prostate is BEST treated by which of the


following modalities:
a. Bilateral orchiectomy
b. Estrogens
c. Orchiectomy followed by estrogens
d. Radical prostatectomy
e. Transurethral resection of the prostate

69 70 71 72
d c b d
198

73. What is not true of drug treatment of BPH:


a. Alfa adrenergic inhibition reduces bladder outlet obstruction
b. 5 Alpha reductase inhibitors decrease level of dihydrotestosterone
c. These drugs are expensive rather than effective
d. All are true

74. Atypical presentation of hypernephroma includes:


a. Fever of unknown origin
b. Polycythemia
c. Hemoptysis/pathological fracture
d. All of the above

75. The common malignant tumor of renal pelvis and ureter is:
a. Squamous carcinoma
b. Adenocarcinoma
c. Transitional cell carcinoma
d. Papillary carcinoma

73 74 75
d d c
199

Breast
1. One of the following statements as regard investigations for breast cancer is
true
A Fifty per cent of breast cancers are missed by population-based mammographic screening.
B Ultrasound can also be used as a screening tool.
C Magnetic resonance imaging (MRI) can be a useful imaging tool.
D Fine-needle aspiration cytology (FNAC) and core biopsy are equally useful diagnostically.

2. All of the following statements are true as regard benign breast disease,
except
A Benign breast disease is the most common cause of breast problems.
B Lipoma is a common condition of the breast.
C Traumatic fat necrosis can be mistaken for a carcinoma.
D 30 per cent of breast cysts recur after aspiration.

3. Which of the following conditions have an increased risk of breast


carcinoma
A Breast cyst
B Duct ectasia
C Atypical ductal or lobular hyperplasia
D Fibroadenoma.

4. In breast carcinoma, which one of the following statements is false?


A Ductal carcinoma is the most common variant.
B Lobular carcinoma occurs in 15 per cent.
C There may be a combination of lobular and ductal features.
D Colloid, medullary and tubular carcinomas carry a poor prognosis.

1 2 3 4
C B C d
200

5. In the treatment of breast cancer, which of the following statements are


false?
A There is a higher rate of local recurrence after conservative surgery and radiotherapy.
B After mastectomy, radiotherapy to the chest wall is not indicated.
C Sentinel lymph node biopsy should be done in clinically node-negative disease.
D Besides treating the patient, the role of axillary surgery is to stage the patient accurately.

6. An 18-year-old presents with a well circumscribed 2-cm mass in her right


breast. The mass is painless and has a rubbery consistency and discrete borders.
It appears to move freely through the breast tissue. What is the likeliest
diagnosis?
(A) Carcinoma
(B) Cyst
(C) Fibroadenoma
(D) Cystosarcomaphyllodes
(E) Intramammary lymph node

7. Galactorrhea, a milky discharge from the nipple in nonpregnant women, is


most likely to be associated with which of the following?
(A) Fibroadenoma
(B) Tubular adenoma
(C) Pituitary adenoma
(D) Hyperparathyroidism
(E) Breast abscess

8. A 36-year-old woman complains of a 3-monthhistory of bloody discharge


from the nipple. At examination, a small nodule is found, deep to the areola.
Careful palpation of the nipple areola complex results in blood appearing at
the 3 O’clock position. Mammogram findings are normal. What is the
likeliest diagnosis?
(A) Intraductal papilloma
(B) Breast cyst
(C) Intraductal carcinoma
(D) Carcinoma in situ
(E) Fat necrosis

5 6 7 8
B C C A
201

9. A 35-year-old premenopausal woman whose mother had breast cancer comes into
your office and has been told that she has fibrocystic breasts. On examination she
has multiple areas of thickening but no discrete mass. Of the following diagnostic
tests, which should be performed?
(A) Re-examination in 6 months
(B) Bilateral breast ultrasound
(C) Thermography
(D) Bilateral breast magnetic resonance imaging (MRI) with gadolinium
(E) Spot compression views if an area of discrete asymmetry or concerning calcifications is seen

10. During a routine screening mammography, a 62-year-old teacher is informed that


she has changes on her mammography, and she should consult her physician. She
can be reassured that the findings that indicate a benign condition are which of the
following?
(A) Discrete, stellate mass
(B) Fine, clustered calcifications
(C) Coarse calcifications
(D) Solid, clearly defined mass with irregular edges
(E) Discrete, non palpable mass that has enlarged when compared with a mass shown on a
mammogram taken 1 year previously.

11. After undergoing modified radical mastectomy for cancer of the right breast, a 52-
year-old female teacher becomes aware that the medial end of her scapula
becomes prominent in protraction movements at the shoulder. She also complains
of some weakness in complete abduction of the same shoulder. What nerve was
injured?
(A) Long thoracic
(B) Thoracodorsal
(C) Ulnar
(D) Intercostobrachial

12. A 50-year-old patient has recently undergone a mastectomy for a 2.5-cm


multicentric breast cancer with three positive axillary lymph nodes (stage IIB). A
metastatic survey is done, and is negative, and she receives adjuvant
chemotherapy. The most common site for distant metastasis would be:
(A) Brain
(B) Bone
(C) Lung
(D) Liver

9 10 11 12
D C A b
202

13. A 36-year-old woman presents with a substantial unilateral breast


enlargement. She had presumed that this was normal, but on examination, a
large, firm tumor is palpated by the attending physician. There is early
erosion on the skin. A favorable outlook can be anticipated if the lesion is
which of the following?
a. Sarcoma
b. phylloides tumor
c. Colloid carcinoma
d. Infiltrating carcinoma

14.The following statements about pulmonary metastases of breast cancer are


all true, except:
a. May be asymptomatic
b. May produce persistent cough
c. May produce dyspnea
d. May cause pleural effusion
e. If localized to one lung while the other lung shows good function,
pneumonectomy is indicated.

15.Haematogenous spread of breast cancer may produce any of the following,


except:
a. Brain metastases.
b. Fracture of the spine
c. Pleural effusion
d. Jaundice
e. Hypocalcemia

16.Concerning sampling of a breast lump, all the following statement are true,
except
a. The most accurate is excisional biopsy
b. Fine need aspiration cytology does not require general anaethesia
c. Fine need aspiration cytology shows tissue architecture.
d. Frozen section biopsy is sometimes inconclusive.
e. Trucut biopsy obtains a core of tissue.

13 14 15 16
B E E c
203

17.The following statements about lymphatic drainage of the breast are all true,
except
a. Lymph from medial half of the breast may drain in internal mammary
nodes.
b. From anywhere in the breast lymphatics drain mostly to axillary nodes
c. Division of axillary nodes into levels I, II and III depends upon the
anatomical relationship with the pectoralis minor.
d. Level one axillary nodes is the highest in the axilla.
e. The sentinel node is the earliest axillary node to be involved with cancer
spread.

18.Regarding duct carcinoma in situ, one statement only is true


a. May be discovered on screening mammography for asymptomatic women.
b. It’s not a true malignancy
c. Close observation is all that is needed
d. Is always bilateral

19.Early breast cancer


a. Means T1N1 tumor or less
b. Means that cure is impossible
c. Means that microscopic metastasis are not present
d. Primary treatment is by surgery and/or radiotherapy
e. Adjuvant chemotherapy is not indicated.

20.For Modified radical mastectomy, all the following are true, except
a. The areola and nipple are removed
b. The whole breast tissue is removed
c. Block dissection of axillary nodes requires excision of axillary vein
d. Nerve to serratus anterior should be spared
e. The pectoralis major is spared.

17 18 19 20
D A D C
204

21.The following statements about post-mastectomy arm edema, all are true,
except
a. Is always transient
b. Adding radiotherapy to the axilla after clearance of lymph nodes increases
the risk of edema
c. Rough dissection of the axillary vein at surgery is one of the causes
d. Insertion of an IV line in the ipsilateral upper limb is a predisposing factor
e. May be caused by recurrence in the axilla.

22.When screening females for breast carcinoma, what if the most significant
risk
a. Three previous breast biopsies in premenopausal females.
b. More than 2 first degree relatives with ovarian or breast carcinoma
c. Hyperplasia in breast biopsy
d. None of the above.

23.What is the hereditary factor percentage in female breast cancers


a. 20-30%
b. 45-50%
c. 6-10%
d. 75-80%
e. None of the above.

24.The axillary lymph nodes are divided into three levels I,II, III by
a. The axillary artery
b. The axillary vein
c. The pectoralis minor
d. The clavicle
e. The nerve to latissimus dorsi

21 22 23 24
A B C C
205

25.About acute lactational breast abscess, one statement only is true


a. Is commonly multilocular
b. Weaning the baby is mandatory
c. Aspiration pus by a wide bore needle is superior to incisional drainage as it
avoids injury to lactiferous ducts
d. Mammography is indicated if an abscess in suspected
e. Treatment is essentially conservative by antibiotics

26.The following about fibrocystic breast are all true, except


a. May be asymptomatic
b. May cause nipple retraction
c. Is a cause of breast pain
d. Produces breast nodularity
e. May cause discharge per nipple

27.How many lactiferous ducts drain into the nipple of the mature female
breast
a. 5-10
b. 15-20
c. 25-30
d. 35-40

28.During pregnancy, alveolar epithelium develops in the breast which is


responsible after delivery for the production of milk. Which portion of
alveolar epithelial cell is responsible for production of the fat present in
human milk
a. Endoplasmic reticulum
b. Mitochondria
c. Cell membrane
d. Cytoplasm

25 26 27 28
A C B D
206

29.The medial mammary artery is a tributary of the


a. 2nd, 3rd, 4th intercostal arteries
b. Internal mammary artery
c. Thoraco-acromial artery
d. Posterior intercostal arteries

30.Which of the following hormones is primarily responsible for differentiation


of breast ductal epithelium
a. Estrogen
b. Testosterone
c. Progestrone
d. Prolactin

31.Absence of the breast (amastia) is associated with


a. Turner’s syndrome
b. Kleinfelter’s syndrome
c. Poland syndrome
d. Fleischer’s syndrome

32.The treatment of choice of Zuska’s disease


a. Observation and NSAIDs
b. Antibiotics, incision and drainage
c. Wide resection of the affected areas
d. Mastectomy

29 30 31 32
A C C B
207

33.A clinically positive subclavicular lymph nodes is a


a. Level I
b. Level II
c. Level III
d. Level IV

34.Which of the following conditions leads to gynecomastia due to an increased


production of estrogen
a. Klinefelter’s syndrome
b. Hepatocellular carcinoma
c. Aging
d. Renal failure

35.The treatment of choice of mondor’s disease


a. Observation and NSAIDs
b. Antibiotics, incision and drainage
c. Wide resection of the affected area
d. Mastectomy

36.The appropriate therapy of paget’s disease of the nipple


a. Topical steroid cream
b. Topical antifungal medication
c. Intralesional steroid injection
d. Resection

33 34 35 36
C B A D
208

37.In the ANDI (abberations of normal development and involution)


classification, a 2 cm fibroadenoma is considered
a. Normal
b. A disorder
c. A disease
d. A premalignant disease

38.Which of the following conditions increases a woman’s risk of breast cancer


a. Sclerosing adenosis
b. Fibroadenoma
c. Atypical lobular hyperplasia
d. Intraductal papilloma

39.A 35 year old woman with BRAC-1 gene mutation seeks your advice about
her known increased risk of breast cancer. You should recommend
a. Mammograms and physical examination every 6 months until she is 50,
then bilateral prophylactic mastectomy
b. Mammograms and physical examination every 6 months + tamoxifen
c. Prophylactic bilateral mastectomy and if she has completed childbearing,
prophylactic bilateral oophorectomy.
d. None of the above.

40.Which of the following about lobular carcinoma in situ is true


a. In general, LCIS occurs at an older age than ductal carcinoma in situ
b. The majority of women with LCIS are premenopausal
c. LCIS is bilateral in 10-20% of women
d. Invasive ductal carcinoma can be expected to occur an average of 5-10
years later in approximately 75% of women with LCIS.

37 38 39 40
B C C B
209

41.Moderate ductal hyperplasia of the breast is characterized by the


microscopic finding of
a. 3-4 cell layer above the basement membrane
b. 5 or more cell layers above the basement membrane
c. Obstruction of >50% of the ductal lumen by hyperplastic cells
d. Obstruction of >70%of the ductal lumen by hyperplastic cells.

42.Which of the following is appropriate treatment for 3 cm fibroadenoma


a. Resection
b. Cryo-ablation
c. Observation
d. All of the above.

43.All the following are recognized side effects of radiotherapy except:


o- Local burn.
b- Pulmonary fibrosis.
c- End arteritis.
d- None of the above.

44.Adjuvant chemotherapy is indicated in:


o- +ve LN biopsy.
b- -ve hormonal receptors.
c- Poor prognosis of the case.
d- All of the above.

41 42 43 44
B D D D
210

45.Chemotherapy is the primary palliative treatment in the following


conditions:
o- Visceral metastasis.
b- Advanced case in premenopausal women.
c- -ve hormonal receptors with distant metastasis.
d- All of the above.

46.The main presentation of Paget disease is:


o- Mass under the nipple
b- Bleeding per nipple
c- Unilateral red scaly nipple
d- Bilateral itchy red vesicles.

47.Lobular carcinoma of breast All true except:


a- Is more common than ductal carcinoma.
b- In situ state is considered risk factor for malignancy.
c- With in situ carcinoma mammography of both breasts is mandatory.
d- Usually bilateral and multicentric.

48.As regard incidence of cancer breast all correct except:


e- Most common female malignancy.
f- Rare in nulliparous females.
g- Very rare below 20 years.
h- Males are rare to be affected but with grave prognosis.

45 46 47 48
D C A B
211

49.Massive swellings of the breast include all the following except:


o- phylloides tumor
b- Atrophic scirrhous carcinoma.
c- Diffuse hypertrophy.
d- Giant fibroadenoma.

50.Tamoxifen use in breast cancer causes all, Except:


o- Decreases recurrence in affected breast.
b- Decreases incidence in contralateral breast.
c- lncreased incidence of endometrial cancer.
d- lncreased incidence of myocardial infarction.

51.In curable breast cancer in premenopausal females may be treated by the


following except:
a- Local mastectomy
b- Radiotherapy
c- Estrogen administration
d- Chemotherapy
e- Tamoxifen

49 50 51
B A C
212

52. The most useful investigation for breast with prosthesis is:
a. Mammography.
b. US.
c. FNAB.
d. MRI.
e. Hormonal receptors.

53. Triple assessment includes the following except:


a. Clinical evaluation.
b. lmaging.
c. Laboratory investigations.
d. Cytology.

54. Etiology of breast abscess includes:


a- Mostly in lactating women.
b- Due to Staph from baby mouth.
c- Bad hygiene.
d- All of the above.

55. Signs of pus formation are the following except:


a- Hectic fever.
b- Edema of overlying skin.
c- No response to medical treatment for 48 hours.
d- Fluctuation occurs early in breast abscess.

52 53 54 55
d c d d
213

56.Best treatment of chronic breast abscess is:


a- Prolonged antibiotic courses.
b- Repeated aspiration.
c- lncision ond drainage.
d- Excision of the whole abscess.

57.Clinical findings of fibroadenosis include:


a- Cyclic pain.
b- Cyclic swelling.
c- Cyclic discharge.
d- All of the above.

58. Cyst of BIood Good is:


a- Retention cyst of duct papilloma.
b- Precancerous lesion of breast.
c- Giant fibroadenomo.
d- Hoemorrhagic cyst containing altered blood.

59. Treatment of fibroadenosis all true except:


a- Psychotherapy has a main role.
b- Primrose improves the condition.
c- Initial treatment should include antiestrogen.
d- Surgery is reserved to complicated cases.

56 57 58 59
d d d c
214

60. The commonest route of infection of breast abscess is:


a- Along blood vessels
b- Retrograde infection along lymphatic vessels
c- Along natural passages (nipple)
d- Along artificial passages like fissures or crocks of nipple ond areola
e- Local extension from infection of the chest wall muscles or ribs
61. Mondor’s disease is:
a- An obscure type of thrombophlebitis particularly affecting veins of the breast.
b- Lymphoedema of the arm.
c- Chondritis of a costal cartilage.
d- Pectus excavatum.

62 Bleeding on zonal pressure is highly suggestive of:


a- Breast abscess.
b- Duct papilloma.
c- Cancer of breast.
d- phylloides tumor.

63. Best treatment for duct papilloma is:


a- Follow up.
b- Local excision.
c- Local excision with safety margin.
d- Chemotherapy ond/or radiation.

60 61 62 63
c a b c
215

64.The following is treatment of choice in pericanalicular fibroadenoma:


a- lrradiation.
b- Enucleation.
c- Removol en block.
d- Simple mastectomy.

65. Probe tesl can differentiate phylloides from:


a- Fibroadenosis.
b- Breast mouse.
c- Cancer breast.
d- Chronic breast abscess.

66. Risk factors of cancer breast include all the following except:
a- Mutation in suppressor genes (BRCA l, Il).
b- Early menarche.
c- Carcinoma in situ.
d- Prolonged lactation.
e- White races.

67. Increased occurrence of cancer breast in upper loteral quadrant is due to:
a- increased oestrogen receptors.
b- increased breast moss.
c- Both of the above.
d- None of the above.

64 65 66 67
b c d d
216

68.Peu d'orange is due to:


a- Skin metastasis.
b- Lymphatic obstruction.
c- Thrombophlebitis.
d- Any of the above.

69. Cancer en cuirasse is:


a- Skin nodule.
b- Lymphedema of breast skin.
c- Both.
d- None of the obove.

70The cause in skin dimpling in any fibrotic affection of:


a- Milk duct.
b- Milk acini.
c- Cooper's ligament.
d- All of the above.

71. Contraindications to conservative surgery include:


a- Large tumour.
b- Paget’s disease of nipple.
c- Previous irradiation.
d- Distant metastasis.
e- All of the above.

68 69 70 71
b c c e
217

72. The following is contraindication to conservative breast surgery:


a- Palpable mobile axillary LNs
b- Tumour 3 cm
c- Mastitis carcinomatosis
d- Patient aged 50 years

73. fibrocystic disease of the breast , all true except:


a.very common in female breast

b. may be asymptomatic

c. common on lt side

d. nipple retraction may be the only clinical criteria

e. easily confused with cancer

74. Treatment of cyclic mastalgia


a. exclusion of malignancy and assurance

b. avoid breast support

c.giving up caffeine consumption

d.bromocriptine 2.5 BID

e. Danazole 100-200 BID

75. breast cyst all true except


a.commonest is galactocele

b.malignancy is suspected if the cyst is tense

c. malignancy is suspected if the lump doesnot totally disappear after complete aspiration

d. multiplicity of cyst raise suspiscion of malignancy

e.cyst excision is necessary in most cases

72 73 74 75
C D B D
218

76. Mammary duct ectasia , all are true except


a. may produce mass similar to carcinoma in axillary tail

b. secretion is viscid thick

c. mammographic finding may mimic cancer

d. anerobic infection are common

e. may produce non lactitional mastitis and abscess

77. Commonest histologic type of breast cancer is


a.duct carcinoma in situ b.lobular carcinoma in situ

c.invasive lobular carcinoma d. medullary carcinoma

e. infilterating duct carcinoma not otherwise specified

78. a women has 3 cm breast mass non palpable LNs and no clinical evidence of
distant metastasis, which is true?
a. stage II breast cancer

b.grade I breast cancer

c. pathological evidence of cancer is necessary

d. consider fibroadenoma and give treatment

e.excision of the mass only is enough

79. About mammography all are true except


a. expose the patient for some radiation

b. is safe and can be repeated at any time

c.malignancy can be proved my microcalcification

d. thickened skin and retracted nipple can be noted in cancer patient

e. irregular outline of the mass is another criteria of malignancy

76 77 78 79
A E C b
219

80. As regard prognosis of breast cancer All are true except


a. worsen by detection of positive axillary LNs

b. better when strong positive estrogen receptor

c. better in lower inner quadrant

d.worse for invasive duct carcinoma than phylloids tumor

e.affected by tumor grading

81. Paget disease of the breast all are true except


a. Is an eczema of the skin

b. Sometimes there is palbable mass

c. mastectomy is indicated

d. the disease is not related to paget disease of bone

e. is a duct carcinoma arising from main lactiferous ducts

82. Axillary LNs all are true except


a. anterior, posterior and lateral group are level II

b. Central l/n are related to intercostobrachial nerve

c.apical group of l/n are level III

d.positive axillary l/n affect the prognosis of breast cancer

80 81 82
C E A
220

GENERAL TOPICS
1. The most effective treatment of anaphylactic shock is :
a) lV anti histaminic.
b) lV corticosteroids.
c) lV antibiotics
d) lntracardiac adrenaline.

2. The following conditions may lead to shock except:

a) Penicillin injection.
b) Myocardial infarction.
c) Quinsy.
d) Loss of 8% blood volume.
e) None of the above.

3. As regard vasovagal shock all are true except :

a) Usual cause is trauma to trigger area.


b) Psychic trauma is o recognized cause.
c) Atropine is the gold standard treatment of the condition.
d) Usually there is peripheral pooling of blood.

4. The following disease may be transmitted during blood transfusion


except

a) CMV
b) Malaria
c) Hepatitis A
d) Brucellosis

1 2 3 4
B D C c
221

5. Secondary hemorrhage ls usually due to:

a) Trauma
b) Slipped ligature
c) infection
d) All of the above
6. The most urgent measure in the management of o severely
injured patient in the reception room is:

a) o. Control of active bleeding.


b) b. Taking blood sample for grouping and cross matching.
c) c. Establishing on intravenous line.
d) d. Securing clear airway and adequate pulmonary ventilation.
e) e. providing tetanus prophylaxis.

7. Septic shock responds best to

a) Massive antibiotics.
b) intravenous infusion.
c) Adrenocortical steroids.
d) Drainage of septic collections.
e) Vasopressors.

8. A comatosed patient who has sustained multiple closed


injures is admitted with severe hypotension. The
hypotension is most probably due to :

a) lntracranial hematoma.
b) Cerebrol-concussion.
c) internal hemorrhage.
d) Neurogenic shock.
e) Cardiogenic shock.

5 6 7 8
C d D c
222

9. The best guide for the required blood transfusion ln


hemorrhagic shock ls :
a) Arterial B.P.
b) Pulse rate.
c) Haematocrit.
d) Central venous pressure (CVP)
e) Urine output per minute.

10. Most nosocomial infections involve the

a) Surgical wound.
b) intravenous sites.
c) Respiratory tract.
d) Urinary tract.
e) Deep veins of the leg.

11. Congenital dermoid cysts are characterized by the


following features except

a) Occur of lines of embryological fusion.


b) Are commonest on the face.
c) Are often attached to the overlying skin.
d) May cause hollowing of subjacent bone.
e) Are lined by stratified squamous epithelium.

12. Sebaceous cyst is characterized by the following except


that it:

a) ls due to obstruction of o sebaceous gland.


b) ls lined by stratified squamous epithelium.
c) Contains a yellow pultaceous greasy material known as sebum.
d) Moy occur on the palms ond soles
e) ls olways anchored to the overlying skin of the punctum of the obstructed
gland.

9 10 11 12
D D C D
223

13. The most serious complication of blood transfusion is:

a) Pyrogenic reactions.
b) Thrombophlebitis of recipient vein.
c) Circulatory overloading.
d) incompatibility reactions.
e) Viral hepatitis.

14. The earliest sign of hypocolcaemia is

a) Carpo-pedal spasms.
b) Positive Chovstek's sign.
c) Positive Trausseou's sign.
d) Tingling of fingers and circumoral region.
e) Defective blood coagulation,

15. The untrue statement about desmoid tumors of the


abdominal wall is that they :

a) Arise from the rectus muscle or its sheath.


b) Occur most often in multiporous women.
c) Are locally invasive.
d) May give rise to metastasis.
e) Should be treated by wide local excision.

16. The earliest finding in tetanus is:

a. Risus sardonicus.
b. Trismus.
c. Dysphagia.
d. Stridor.

13 14 15 16
D D D c
224

17. Most common hand infection is:

a. Distal pulp space infection (felon).


b. Paronychia.
c. Ulnar bursitis.
d. Thenar space infection.

18. The most desirable position to immobilize the hand:

a. Wrist is flexed, MCP joints ore extended and lP joints ore flexed.
b. Wrist is flexed. MCP joints ore flexed and lP joints ore extended.
c. Wrist is extended, MCP joints ore extended and lP joints ore flexed.
d. Wrist is extended. MCP joints ore flexed and lP joints ore flexed.
e. Wrist is extended, MCP joints ore flexed and lP joints ore extended.

19. Most common site of actinomycosis:

a. GlT.
b. Head ond neck.
c. Breast.
d. Lungs.

20. The most frequent GIT complication of an extensive burn


is :

a. Acute gastritis.
b. Acute dilatation of the stomach.
c. Curling's ulcer.
d. Paralytic ileus.
e. Diarrhea.

17 18 19 20
B E B B
225

21. Which statement is incorrect concerning electrical burns


?

a. Usually hove o small surface area.


b. Are always superficial.
c. Are often associated with massive muscle necrosis.
d. Moy cause reddish discoloration of the urine.
e. Are best treated by immediate excision and grafting.

22. Keloids are characterized by the following, except:

a. Consist of dense overgrowth of scar tissue.


b. Develop after wounds, burns and vaccination marks.
c. Are particularly common in negroes and pregnant females.
d. Occur most often on the face. neck and front of the chest.
e. May turn malignant.

23. The most frequent fracture of the face involves the:

a. Zygoma.
b. Maxilla.
c. Orbital floor.
d. Nasal bones.
e. Mandible.

24. ln the rule of nines:

a- Front of trunk ) 27%


b- Head and neck > 18%
c- Each lower limb ) I8%
d- All of the above

21 22 23 24
B E E c
226

25. All of the followings ore factors affecting wound healing:

a. Age of the patient


b. General condition
c. Corticosteroid
d. Type of wound and type of closure
e. All of the above.

25. E
227

PLASTIC SURGERY
1.Themain advantage of Lund and Browder charts over rule of nines is:
a- lt is easier
b- More accuroteto regarding sex of patient
c- More accuroteregarding age of patient
d- All of the above

2. About 3rd degree (full thickness) burn:


a- Completely painless
b- No graft is needed
c- Healing occurs from skin appendages
d- Usually dry, white or block

3.ln the rule of nines:


a- Front of trunk = 27%
b- Head and neck = 18%
c- Each lower limb = I8%
d- All of the above

4.About management of burns:


a- Biological dressing decreases all complications of burn
b- Parkland's formula is a good rule for fluid replacement
c- Occlusive dressing is easier to perform
d- Exposure method is suitable for face and buttocks
e- All of the above

1 2 3 4
C D C E
228

5.About healing intentions:


a. Ulcers heal by first intention
b. Second intention gives better cosmoses than first
c. First intention occurs on clean cut edges without gaps or tissue loss
d. All of the above

6. All of the followings are factors affecting wound healing:


a. Age of the patient
b. General condition
c. Corticosteroid
d. Type of wound and type of closure
e. All of the above.

7.The most common site for the following diseases is correctly matched except:
a- Keloids :face, neck and skin over sternum
b- Hypertrophic scars :extensor surfaces
c- Hemangiomas:head and neck
d- Basal cell carcinoma:face

8. Langenbeek's repair:
a- lsa method of grafting in burns
b- lsa method of repair after surgical excision of Sq.C.C.
c- lsa method of repair of cleft lip
d- lsa method of repair of cleft palate

5 6 7 8
C E B D
229

9.About cleft upper lip all ore correct except:


a- Median is the most common
b- Lateral is due to non-fusion of median nasal process (future frenulum) with
maxillary process
c- Lateral is more common on left side
d- Best time for repair is the earliest possible time (3 months)
e- Repair is mainly for cosmetic purposes

10.About cleft palate all are correct except:


a- Usually associated with other congenital anomalies
b- May range from cleft uvulaup to tripartite deformity
c- Causes nasal regurgitation, nasal deformity
d- Best time for repair ls 5 years old

11.The following are subtypes of capillary hemangioma except:


a- Strawberry hemongioma
b- Portwinehemongioma
c- Cirsoid aneurism
d- Salmon Patch

12.A subtype of capillary hemangiomaraised above skin surface:


a- Strawberry hemangioma
b- Portwinehemangioma
c- Salmon patch
d- None ofthe above

9 10 11 12
A D C A
230

13. First-intention healing of clean incised wounds is characterized by the following


except:
a. An inflammatory reaction during the first few days.
b. Epithelialization within 48 hours.
c. Fibroblastic proliferation and capillary budding during the next week.
d. Region of normal tensile strength within 4 weeks.
e.Production of thin linear scar.

14.Woundhealing is not impaired by


a. Anemia.
b. Hypoproteinemia.
c. Ascorbic acid deficiency.
d. Cortisone administration.
e. lmmunosuppressivetherapy.

15.Metabolicchanges after burning are due to the following except :


a. The endocrine responses to injury.
b. Local fluid loss into the burnt area.
c. Reduced heat loss.
d. lncreased insensible water loss.
e. Bacterialinfection.

16.Themostfrequent gostrointestinalcomplication of an extensive burn is :


a. Acute gastritis.
b. Acute dilatation of the stomach.
c. Curling's ulcer.
d. Paralytic ileus.
e. Diarrhea

13 14 15 16
D A C B
231

17.The most useful measure in preventing renal shut-down in post burn


patients is:
a. Monitoring an hourly urine output between 30 and 50 ml.
b. Alkalinization of the urine.
c. Mannitoladministration.
d. Administration of colloids.
e. Blood transfusion.

18. Which statement is incorrect concerning electrical burns ?


a. Usually have asmall surface area.
b. Are always superficial.
c. Are often associated with massive muscle necrosis.
d. Moy cause reddish discoloration of the urine.
e. Are best treated by immediate excision and grafting.

19.Keloids ore characterized by the following, except:


a. Consist of dense overgrowth of scar tissue.
b. Develop after wounds, burns and vaccination marks.
c. Are particularly common in negroes and pregnant females.
d. Occur most often on the face. neckand front of the chest.
e. May turn malignant.

20. A paraplegic bed-ridden patient developed a large deep bed-sore over his
sacrum. The best regimen is by wide excision and :
a. Local applications.
b. Thierschgrafting.
c. Wolfe-grafting.
d. Localskin flops.
e. Myocutaneousflap.

17 18 19 20
A B E E
232

21.lnunilateral hare-lip, the following statements are true except that it:
a. Affects the upper lip only.
b. Moy be partial or complete.
c. ls due to failure to fusion between the maxillary process with the fronto-nasal
process and the maxillary process of the opposite side.
d. lsalways associated with nasaldeformity.
e. Produces no serious speech defects.

22. Concerning complete cleft palate, the untrue statement is that it :


a. ls due to failure fusion of the palatal shelves of the maxillary processes with each
otherand with the frontonasal process.
b. ls often associated with cleft-lip and broadening of the face.
c. lnterferes with nutrtionand speech.
d. Predisposes to upper respiratory tract infections.
e. Requires surgical repair after the second year of life.

23.The most frequent fracture of the face involves the:


a. Zygoma.
b. Maxilla.
c. Orbitalfloor.
d. Nasal bones.
e. Mandible.

24. A 6 year old boy has a 2 cm lesion on Right cheek. The lesion has been
enhancing since birth. On Auscultation lesion is pulsatile, it has a thrill and it has a
harsh sound on auscultation. It has multiple feeding arteries. The best treatment
option for this lesion would be

a) External beam radiotherapy


b) Ligation of the feeding arteries
c) LASER therapy
d) Embolisation followed by surgery

21 22 23 24
E E E D
233

25. Which of the following is not true about keloids?

a) Keloid is excessive scar formation within and outside the boundaries of skin incision
b) Recurence rate of keloid after excision is 10-20%
c) Exact cause of keloid formation is not known but fibroblast dysfunction is commonly
attributed
d) Adjunctive treatment besides surgery gives the best results

26. Which of the following is true about Langer's line of the skin?

a) These are in the direction of muscle fibres.


b) These are perpendicular to the long axis of underlying muscle fibres
c) Most favourable incision is circular over an exposed area
d) Most favorable incision is a rhomboid shaped one over an exposed area

27. The repair of cleft lip should be done at which age


a) 1 month
b) 3 months
c) 6 months
d) 1 year

28.Fastest promoter of epithelisation in wound healing occurs with

a) Tight approximation of wound edges


b) Occlusive dressing
c) Leaving the wound open
d) Steroid Application

25 26 27 28
B B B B
234

Which of the following statements are true?

1 . The surface area of the peritoneal membrane is nearly equal to that of the skin.
T
2 . The parietal peritoneum is poorly innervated.
F
3. The peritoneum has the capacity to absorb large volumes of fluid.
T
4. The peritoneum has the ability to produce fibrinolytic activity.
T
5. When injured, the peritoneum produces an inflammatory exudate.
T

Which of the following statements are true?

6. Peritonitis in perforated duodenal ulcer is initially sterile.


T
7. Immunocompromised patients may present with opportunistic peritoneal infection.
T
8 Bacteroides are sensitive to penicillin.
F
9 In perforated duodenal ulcer there may be signs of peritonitis in the right iliac fossa.
T
10 Children can localise infection effectively.
F

Which of the following statements are true?

15 The oesophagus is 30 cm long.


F
16The oesophagus has three natural constrictions.
T
17 It is lined throughout by columnar epithelium.
F
18 The lower oesophageal sphincter (LOS) is a zone of high pressure.
T
19 The pressure at the LOS is 50 mmHg.
F
235

Dysphagia which of the following statements are false?


20 Difficulty on swallowing (dysphagia) is a cardinal symptom of oesophageal carcinoma.
T
21 Retrosternal pain on swallowing (odynophagia) is always of cardiac origin.
F
22 Heartburn is a common symptom of gastro-oesophageal reflux disease (GORD).
T
23 Dysphagia in the oral or pharyngeal (voluntary) phase, when patients say they cannot
swallow, is usually from neurological or muscular diseases.
T
24 Regurgitation and reflux are the same and are caused by obstruction to the oesophagus.
F

Which of the following statements regarding investigations in

dysphagia are false?

25 Barium swallow is the investigation of choice in GORD.


F
26 Flexible oesophagogastroduodenoscopy (OGD) is the initial investigation of choice in
suspected carcinoma.
T
27 Endosonography (EUS) should be carried out when a carcinoma is seen in the oesophagus.
T
28 Oesophageal manometry should be done when motility disorder is suspected.
T
29. 24 h pH recording is an accurate method of evaluating GORD.
T

Oesophageal emergencies,Which of the following statements are

true?

30In a suspected foreign body (FB) in the oesophagus, water-soluble contrast examination should
be carried out.
F
31 When a food bolus is stuck in the oesophagus, always suspect an underlying disease.
T
32 Most iatrogenic perforations of the oesophagus can be treated conservatively.
T
33 Most spontaneous perforations of the oesophagus (Boerhaave’s syndrome) require an
236

operation.
T
34 In Mallory–Weiss syndrome the tear is usually in the lower end of the oesophagus
F

Which of the following conditions are precancerous for esophagus?

35 Pharyngeal (Zenker’s) diverticulum


T
36 GORD
T
37 Achalasia
T
38 Corrosive stricture
T
39 Barrett’s oesophagus
T

One of the curative operations for carcinoma of the oesophagus is

called

40 Whipple’s operation
F
41 Anderson–Hynes operation
F
42 Heller’s operation
F
43 Ivor–Lewis operation
T
44 Hartmann’s operation
F

Which of the following statements are true?

45 The right gastric artery is a branch of the coeliac artery.


F
46 Vagal fibres to the stomach are afferent.
F
237

47 The parietal cells are in the body of the stomach and are the acid-secreting cells.
T
48 The venous drainage of the stomach ends in the inferior vena cava (IVC).
F
49The chief cells secrete pepsinogen.
T

Which of the following are predisposing factors for gastric ulcer?

50 Gastric acid
F
51 H. pylori infection
T
52 NSAIDs
T
53 Smoking
T
54 High socioeconomic groups.
F

Which of the following are true with regard to the clinical features of

peptic ulcers?

55 The pain never radiates to the back and this differentiates this from biliary colic.
F
56 Vomiting is a notable feature.
F
57 Bleeding is rare.
F
58 They may cause gastric outlet obstruction.
T
59 Weight loss is a typical symptom.
F

Which of the following are important in the causation of gastric

cancer?
238

60 H. pylori infection
T
61 Gastric atrophy
T
62 Pernicious anaemia
T
63 Previous gastric surgery
T
64 Smoking.
T

Which of the following statements regarding clinical features of

gastric cancer are true?

65 They may be non-specific in the early stages.


T
66 Anaemia can be a presenting symptom.
T
67 Troiser’s sign refers to a palpable ‘Virchow’s node’ in the right supraclavicular fossa.
F
68 This is a rare cause of GOO.
F
69 Trousseau’s sign is diagnostic.
F

Which of the following are associated with the molecular pathology

of gastric cancer?

70 Mutation or loss of heterozygosity in APC gene


T
71 Mutation in gene coding for beta-catenin
T
72 Mutations in gene coding for E-cadherin
T
73 Inactivation of p53
T
74 Microsatellite instability.
T
239

Which of the following are true with regard to spread of gastric

cancer?

75 Tumour reaching the serosa usually indicates incurability.


T
76 Blood-borne metastases commonly occur in the absence of lymph node spread.
F
77 Krukenberg’s tumours are always associated with other areas of transcoelomic spread.
F
78 Sister Joseph’s nodule is diagnostic of gastric cancer.
F
79 The lymphatic vessels related to the cardia have no relation to the oesophageal lymphatics.
F

Which of the following are unequivocal evidence of incurability in

gastric cancer?

80 Haematogenous metastases
T
81 Involvement of distant peritoneum
T
82 N3 nodal disease
F
83 Involvement of adjacent organs
F
84 Gastric outlet obstruction.
F

Which of the following are true regarding the treatment of gastric

cancer?
240

85 D1 resection is superior to a D2 resection.


F
86 The 5-year survival in the UK is between 50 and 75 per cent.
F
87 There is a definite role for neoadjuvant chemotherapy.
T
88 Gastrointestinal continuity is established after total gastrectomy by a Roux loop.
T
89 Radiotherapy to the gastric bed is a useful adjunct.
F

Which of the following statements are false?

90 The liver is fixed in its place by peritoneal reflections called ligaments.


T
91 The major part of the blood supply of the liver is derived from the hepatic artery.
F
92 The portal vein lies posterior to the hepatic vein and common bile duct.
T
93 The bile duct, portal vein and hepatic artery are contained in the lesser omentum.
T
94 The portal vein is formed by the union of the superior mesenteric and left gastric veins.
F

Which of the following statements are true?


95 The left hepatic duct has a longer extrahepatic course than the right.
T
96 The major venous drainage is by three veins – right, middle and left hepatic veins
draining directly into the inferior vena cava (IVC).
T
97 All the three major veins join the IVC within the liver parenchyma.
F
98 The functional lobes of the liver are divided by the falciform ligament.
F
99 The liver is divided into eight functional segments – I to IV in the left hemi-liver and V–VIII in
the right hemi-liver.
T

Which of the following statements are false?


241

100 The spleen lies in front of the left 10th, 11th and12th ribs.
F
101 The splenic artery arises from the coeliac axis.
T
102 The inferior mesenteric vein empties into the splenic vein.
T
103 The tail of the pancreas lies in the lienorenal ligament.
T
104 The inner surface of the spleen has two impressions – gastric and colic.
F

Splenectomy , which of the following statements are true?

105Partial splenectomy can result in splenic regeneration.


T
106 During splenectomy the tail of the pancreas can be damaged.
T
107A gastric or pancreatic fistula can occur as a post-splenectomy complication.
T
108In a left hemicolectomy the spleen can be in danger.
T
109The risk of opportunist post-splenectomy infection (OPSI) is greatest after the first 6 months
of splenectomy.
F

Which one of the following statements is false?


110 Splenuli are present in approximately 10–30 per cent of the population.
T
111 Splenic artery aneurysm can occur as a complication of acute pancreatitis.
T
112 Plain abdominal X-ray is the ideal imaging modality.
F
113 A massively enlarged spleen is prone to infarction.
T
114 The splenic hilum is a common site for the development of a pseudocyst of the pancreas.
T

Which of the following statements regarding the treatment of


242

gallstones are true?

115 Patients with asymptomatic gallstones should routinely be advised to have a cholecystectomy.
F
116 Ninety per cent of patients with acute cholecystitis respond to conservative treatment.
T
117 Antibiotics are not required in the management of acute cholecystitis in the absence of
jaundice.
F
118 Urgent laparoscopic cholecystectomy in a patient with acute cholecystitis is associated with
a five times greater conversion rate compared with elective surgery.
T
119 Acalculous cholecystitis has a mild clinical course.
F

Which of the following statements regarding obstructive jaundice are

true?

120The incidence of symptomatic bile duct stones varies from 5 to 8 per cent.
T
121 Charcot’s triad consists of pain, stones and jaundice.
F
122 Primary sclerosing cholangitis (PSC) is associated with hypergammaglobulinaemia and
elevated smooth muscle antibodies.
T
123 Clonorchiasis can predispose to bile duct carcinoma.
T
124 Bismuth type 3 biliary stricture is a hilar stricture.
T

Which of the following regarding gall bladder carcinoma are true?


125 It is a rare disease.
T
126 The majority are adenocarcinomas.
T
243

127The CA19–9 is elevated in 80 per cent of the cases.


T
128 A palpable mass is an early sign.
F
129 Prognosis is generally good.
F

Which of the following statements are true?

130 The pancreas weighs 200 g.


F
131 The uncinate process lies behind the superior mesenteric vessels.
T
132 The vast majority of pancreatic tissue is composed of exocrine acinar tissue.
T
133 Of the endocrine cells, 75 per cent are B cells, 20 per cent are A cells and the remainder are D
cells
T
134 The accessory pancreatic duct drains the head and uncinate process.
T

Which of the following statements are false?

135 Ultrasonography (US) is the initial investigation of choice in the jaundiced patient.
T
136 When doing a computed tomography (CT) scan, initially an unenhanced scan must be done
followed by a scan after intravenous contrast injection (CECT).
T
137 While doing a magnetic resonance cholangiopancreatography (MRCP), intravenous secretin
injection helps to determine any obstruction to the pancreatic duct.
T
138 An increase in serum amylase is diagnostic of acute pancreatitis.
F
139 Endoscopic retrograde cholangiopancreatography (ERCP) should always be preceded by a
plain radiograph.
T

Which of the following statements are true?

Pancreatic injury
244

140Pancreatic injury is common following blunt abdominal trauma.


F
141Pancreatic injury is often accompanied by damage to the liver, spleen and duodenum.
T
142 The serum amylase is raised in most cases of pancreatic injury.
T
143 A CECT scan will delineate the damage.
T
144 In doubtful cases, urgent ERCP is helpful.
T

Which of the following statements is false?

145 All patients with pancreatic trauma should undergo an exploratory laparotomy.
F
146 Pancreatic duct disruption requires surgical exploration.
T
147 Severe injury to the duodenum and the head of the pancreas requires a
pancreatoduodenectomy.
T
148 After conservative management for pancreatic injury, duct stricture and pseudocyst may
occur as complications.
T
149 During splenectomy, iatrogenic injury to the pancreatic tail can occur.
T

Acute pancreatitis, Which of the following statements is false?

150 Acute pancreatitis accounts for 3 per cent of hospital admissions in the UK for abdominal
pain.
T
151 Acute pancreatitis is classified into mild and severe.
T
152 80 per cent of cases are mild acute pancreatitis, with a mortality rate of 1 per cent.
T
153 20 per cent are severe acute pancreatitis, with a mortality of 20–50 per cent.
T
154 In all cases of acute pancreatitis, there is a marked rise in serum amylase.
F

About anatomy of small bowel


245

155 The small bowel is approximately 10 metres long.


F
156 The colon is approximately 3 metres long.
F
157 The most fixed part of the small bowel is the duodenum
T
158The jejunum is wider, thicker and more vascular than the ileum.
T
159Peyer’s patches are contained in the ileum.
T

Diverticula of small intestine

160 Duodenal diverticulum may result from a long-standing duodenal ulcer.


T
161 Jejunal diverticula may give rise to malabsorption problems.
T
162 A Meckel’s diverticulum can cause severe lower gastrointestinal haemorrhage.
T
163 A suspected Meckel’s diverticulum is best imaged by a barium meal and follow through.
F
164 Pain originating in a Meckel’s diverticulum is located around the umbilicus
T

Diverticular disease of colon

165In the Western world, 60 per cent of the population over the age of 60 have diverticular
disease.
T
166A low-fibre diet causes the disease.
T
167 These diverticula consist of mucosa, muscle and serosa.
F5
168Those with perforation have a 10 times higher mortality than those with an inflammatory
mass.
T
169 Sepsis is the principal cause of morbidity.
T

complication of diverticular disease of the colon are:


246

170 Paracolic abscess


T
171 Fistulae
T
172 Lower gastrointestinal haemorrhage
T
173Carcinoma
F
174 Stricture.
T

In complicated diverticular disease?

175Urinary symptoms may be the predominant presentation at times.


T
176 Profuse colonic haemorrhage may occur in 17 per cent.
T
177 Fistulae occur in 5 per cent of cases.
T
178 The commonest fistula is coloenteric.
F
179 In acute diverticulitis, CT scan is the ‘gold standard’ for imaging.
T

In the surgical treatment of diverticular disease

180 Colonoscopy must be carried out in all elective cases.


T
181Barium enema is essential prior to elective operation.
T
182 Primary resection and end-to-end anastomosis mustt be carried out in all cases.
F
183 Hartmann’s operation is the procedure of choice in perforated diverticulitis.
T
184 In vesicocolic fistula, a one-stage operation can usually be done.
T

In ulcerative colitis (UC)?


247

185 In 95 per cent of cases, the disease starts in the rectum and spreadsproximally.
T
186 It is a diffuse disease affecting all the layers of the large bowel.
F
187 Granulomas are a typical microscopic feature.
F
188 The transverse colon is affected in toxic megacolon.
T
189 Patients may present as an emergency with fulminating colitis in 5–10 per cent.
T

complication of UC include

190 Carcinoma
T
191 Primary sclerosing cholangitis
T
192 Internal fistulae
F
193Ankylosing spondylitis
T
194 Perforation.
T

barium enema finding in UC:

195 Loss of haustrations


T
196 Narrow contracted colon
T
197 Increase in the presacral space
T
198 Cobblestone appearance
F
199 Backwash ileitis.
T

The indications for surgery in UC:

200 Severe fulminating disease not responding to vigorous medicaltreatment.


248

T
201 Severe dysplastic change or cancer on biopsy.
T
202 Non-compliance of medical treatment.
F
203Chronic steroid-dependent disease requiring large doses.
T
204 Extraintestinal disease.
T

regarding surgery in UC:

205 In the emergency situation, total abdominal colectomy and ileostomy should be the
procedure of choice.
T
206 Proctocolectomy and ileostomy are associated with the lowest complication rate.
T
207 Restorative proctocolectomy with an ileoanal pouch should be considered in all patients.
T
208 Colectomy with ileorectal anastomosis is the most favoured procedure.
F
209 Ileostomy with a continent intraabdominal pouch is not often done.
T

About Crohn’s disease (CD):

210The ileum is affected in 60 per cent of cases.


T
211 It affects the entire thickness of the bowel.
T
212 Non-caseating granulomas are found in only 60 per cent of patients.
T
213 One in 10 patients have a first-degree relative with the disease.
T
214 A patient can be cured of CD once the diseased small or large bowel is removed.
F

About imaging in CD :

215 Small-bowel enema is the imaging of choice in small-bowel disease.


T
249

216 Barium enema and colonoscopy should be done for large-bowel disease.
T
217 MRI is the ‘gold standard’ for perianal fistulae.
T
218 CT scan is used for suspected intraabdominal abscess and internal fistulae.
T

About inflammatory bowel disease (UC and CD) :

219Patients must be managed jointly by the physician and surgeon.


T
220Surgery, when indicated, must be as radical as possible.
F
221 Patients must be given a good trial of optimum medical treatment prior to surgery.
T
222 There is more chance of a cure after surgery in UC than in CD.
T
223 In emergency presentation, patients must be vigorously resuscitated prior to operation and
managed in the ITU postoperatively.
T

In Familial Adenomatous Polyposis:

224 20 per cent of FAP arise as a result of new mutations in the adenomatous polyposis coli
(APC) gene.
T
225 Large-bowel cancer occurs 15–20 years after the onset of the disease.
T
226 On surveillance, if there are no polyps by the age of 30 years, FAP is unlikely.
T
227 Polyps do not develop anywhere else other than the colon.
F
228 Colectomy with ileorectal anastomosis may result in rectal cancer later.
T

Colon cancer

229 Right colonic cancers present with features of anaemia.


250

T
330 Left colonic cancers present with rectal bleeding and obstructive
symptoms.
T
331 Even for an experienced colonoscopist, the failure rate to visualise the caecum is 10 per cent.
T
332 Intravenous urography (IVU) should be routinely done.
F
333 Synchronous cancers occur in 5 per cent.
T

In large bowel cancer:

334 Thorough preoperative assessment and staging should be done with


colonoscopy, US and spiral CT.
T
335 Resection is not done if the patient has liver metastases.
F
336 If, at operation, hepatic metastases are found, biopsy should be done.
F
337 Hepatic resection for metastases should be considered as a staged
procedure.
T
338 Over 95 per cent of colonic carcinomas can be resected.
T

Causes of adynamic intestinal obstruction include:

339 Paralytic ileus


T
400 Hernia
F
401 Mesenteric vascular obstruction
T
402 Pseudo-obstruction
T
403 Adhesions.
F

Increased postoperative adhesions occurs with:


251

404 Ischaemic areas


T
405 Good surgical technique
F
405 Foreign material
T
406 Covering anastomosis and raw areas
F
407 Crohn’s disease.
T

Regarding intussusception:

408 It is most common in children with a peak incidence between 5 and 10


months of age.
T
409 About 10 per cent of infantile intussusceptions are idiopathic.
F
410 It causes obstruction but not strangulation.
F
411 Meckel’s diverticulum can be a cause in older children.
T

About sigmoid volvulus:

412 This is the most common site of volvulus in adults.


T
413 The predisposing factors include constipation, long pelvic mesocolon and narrow attachment
of the mesocolon.
T
414 The rotation is usually in a clockwise direction.
F
415 Flatus tube decompression is associated with a low rate of recurrence.
F
416 There is no role for emergency surgery.
F

417. In Glasgow coma scale, Eye opening scale is 1 to 4, Best verbal response scale is to 5. Best
motor response scale is 1 to 6. Thus, the lowest scale is 3, while the highest scale (fully
conscious is 15).
252

Ans. T

418. Muscles of the leg are enclosed in a tight sleeve that is formed of the deep fascia. Fascial septa
divide these muscles into two compartments.

Ans. F

419. Rigid fixation of a fracture is needed for healing of an arterial repair, otherwise it will be
disrupted or contused and thrombosed.

Ans. T

420. This patient is suspected to have injury of the bulbous urethra. If the patient voids, urine will
spread in the subcutaneous tissues, deep to the membranous part of the superficial fascia.

Ans. T

421. The most important factor that affects wound healing is vascularity of its edges.

Ans. T

422. A hernia must be repaired with absorbable material, so as to resist stresses for life.

Ans. F

423. Delayed primary closure of the skin means skin closure when pus stops to drain from the
wound.

Ans. F

424. Incised wounds treatment is by hemostasis, and leaving the wound open to drain.

Ans. F

425. The extent of tissue damage by a missile depended upon its weight.

Ans. T

426. The extent of tissue damage by a missile is proportionate to its velocity.

Ans. T

427. Stab wound usually fracture bones and thus add to tissue damage.
253

Ans. F

428. Stab wounds are inflicted by knives and may injure viscera.

Ans. T

429. In the presence of contamination, delayed primary closure of an incised wound reduces the
possibility of infection.

Ans. T

430. In healing by secondary intention wound contraction and granulation tissue doas not play
major roles.

Ans. F

431. Shoch with bradycardia is Hypovolaemic shock.

Ans. F

432. Disseminated intravascular coagulation (DIC) is a complication of shock particularly the


hypovolemic type.

Ans. F

433. Metabolic acidosis is commonly seen in cases of shock though the pH is normal a low
bicarbonate level indicates metabolic acidosis

Ans. T

434. Adult respiratory distress syndrome (ARDS) is one example of the multiple organ failure
syndrome which occurs with shock.

Ans. T

435. Shock particularly the septic type major burns and severe pancreatitis can all trigger the
release of cytokines that lead to systemic inflammatory response syndrome(SIRS) the to
multiple organ failure (MOF).

Ans. T

436. Renal failure that is caused by shock is irreversible.


254

Ans. F

437. In cardiogenic shock, the neck veins are collapsed.

Ans. F

438. Treatment of cyclic mastalgia can be avoidance of breast support.

Ans. F

439. A Fibroadenoma is a mixed epithelial and mesenchymal benign tumour. The peakage of
appearance is late teens and early twenties.

Ans. T

440. Regarding breast cysts, Multiplicity of cysts raises suspicion of malignancy.

Ans. F

441. The commonest histological type of breast cancer is Ductal carcinoma in situ.

Ans: F

442. In breast cancer, T stands for tumour size.A tumour between 2-5 cm is T2.

Ans. T

443. Mammography indicated for regular screening of women who are at high-risk of
development of breast cancer.

Ans. F

444. Breast cancer is better for tumors in the medial half than the lateral one of the breast.

Ans. F

445. Paget’s disease of the nipple is lobular carcinoma arising at the opening of a lactiferous duct
on the nipple.

Ans. F

446. In a young female patient, a Lanz incision is preferred as this is more in keeping with
255

langer's lines.

Ans. T

447. A Kocher's incision is a subcostal incision used for open cholecystectomy.


Ans. T

448. A Collar incision is made for submandibular gland excision and block dissection.

Ans. F

449. Esophageal carcinoma is commoner in smokers, in people with increased alcohol intake
oesophageal strictures or certain dietary deficiencies.

Ans. T

450. A pharyngeal pouch occurs in a weakness in the superior constrictor muscle. It is also known
as killian's dehiscence.

Ans. F

451. A-Achalasia is Treated by injection of botulinum toxin.

Ans. T

452. In venous type ulcer the duplex show good arterial flow,so compression bandaging would be
suitable treatment.

Ans. T

453. In an arterial ulcer with stigmata of chronic arterial insufficiency, Revascularization should
be attempted, either endovascularly or surgically.

Ans. T

454. Inter-sphincteric abscesses can be incised, drained, and laid open. Trans-sphincteric
abscesses are usually laid open to reduce the risk of incontinenence following damage to both
internal and external sphincter mechanisms in this case seton drainage.

Ans. T

455. Sub-phrenic abscess is a known complication of diverticular disease, usually occurring


approximately 1-3 days following the initial episode.
256

Ans. F

456. Amylase of 1000 somorgyi U/dl is characteristic of acute pancreatitis.

Ans. T

457. Epstein-Barr virus is associated with Burkitt's lymphoma in the presence of malarial
infection.

Ans. T

458. Aspergillus flavus releases afatoxin which occur as a dietary contaminants and are associated
with hepatocellular regeneration.

Ans. F

459. Primary hydrocele is most common at young age.

Ans. F.

460. Fournier's gangrene is necrotizing fasciitis of the scrotum.

Ans. T.

461. This man has a Glasgow coma scale (GCS) of 3.He is unable to protect his own airway;
securing the airway is therefore paramount

Ans. T

462. Hyperplasia is increased tissue or organ size due to increased cell size in response to
increased functional demand.

Ans. F

463. Atrophy is the decrease in size of an organ or tissue by reduction in cell size and /or reduction
in cell numbers.

Ans. T

464. Agenesis is the failure of development of an organ or structure. This differs from atresia
which is failure of development of a lumen in a normally tubular structure.

Ans. T
257

465. Transitional cell carcinomas of the renal pelvis resemble those affecting the bladder, but are
much less common they tend to invade the renal parenchyma and have a tendency to distal
spread.

Ans. T

466. HAEMORRHAGIC SHOCK class 1 is loss of 0-15% of circulating volume; there are no
obvious changes apart from the patient perhaps feeling uncomfortable and restless.

Ans. T

467. Painless jaundice together with a history of weight loss is strongly suggestive of a malignancy.

Ans. T

468. Sharing of needles among intravenous drug misusers is a serious risk factor for hepatitis B
and C both of which predispose to the development of hepatocellular cancer in later years
Alpha-fetoprotein is a marker for this cancer although it is also raised in hepatoma.

Ans. T

469. Colo-colic intussusception typically presents as in this case and the tumor acts as the apex.
Half of all adult intussusceptions are due to carcinoma.

Ans. T

470. Hyperparathyroidism presents with the symptoms of hypocalcaemia including paraesthesia,


cramps, tetany, circum-oral tingling, and convulsions.

Ans. F

471. Secondary hyperparathyroidism is caused by chronic renal failure in response to a low


calcium level, and is characterized by a low serum and a raised PTH level.

Ans. T

472. A recent general anesthetic and opioid-based analgesia causing constipation are two factors
which predispose to developing urinary retention post-operatively.

Ans. T

473. Umbilical hernias in young children should be treated surgically.


258

Ans. T

474. Laparoscopic repair of inguinal hernias is recommended for the management of bilateral
inguinal hernias unless there are any contraindications present.

Ans. T

475. Squamous cell carcinoma is most common in sun-exposed areas frequently bleeds and is
characterized by an undermined edge.

Ans. T

476. Squamous cell characterized by a rolled pearly edge. It dose not tend to bleed as often as
melanoma.

Ans. F

477. Full-thickness skin grafting can be used on exposed tendon, but is not good for exposed bone
or cartilage.

Ans. T

478. Local flaps provide their own blood supply and can transfer skin, fascia, muscle, and bone.

Ans. T

479. Free flaps can not provides their own blood supply, and can be used with local flap.

Ans. F
480- Haemorrhoidal bleeding is painless and not mixed with stools it is bright red as its
origin from the arteriovenous cushions of the anal canal
Ans: T

481-Rubber band ligation is most useful in stopping bleeding from first and second degree
haemorrhoids for third degree haemorrhoids, band ligation is enough to control the prolapse
Ans: F
480.
482- Haemorrhoidectomy causes causes considerable postoperative pain. This may result in
reflex retention of urine,rather than incontinence
Ans: T

483-Faecal incontinence can occur if the whole anal sphincter mechanism is divided Anal stenosis
may follow haemorrhoidectomy if the surgeon leave intact muco-cutaneous bridges between the
259

raw areas
Ans: F

484-The majority of abscesses start as infection of an anal gland.These glands are present in the
intersphincteric space and have their ducts open at the dentate line
Ans: T

485- Anal fitulotomy or fistulectomy wounds should be sutured


Ans: F

486- Surgery for a high fistula requires division of the entire anal sphincter mechanism and
would produce incontinence if done in one stage
Ans: T

487- The majority of anal canal cancers are of th squamous carcinoma type
Ans: T

488- The caudate lobe is segment I.it lies between the porta hepatic and the inferior vena
cava(IVC)posteriorly
Ans: T

489-The enzyme alkaline phosphatase is elevated in cases of bill stasis and with bone diseases
particularly bone resorption and destruction.
Ans: T

490-Being a retroperitoneal organ an isolated injury of the kidney is likely to stop bleeding
spontaneously because of the tamponade effect of the posterior peritoneum on the haematoma
Ans: T

491- The main danger of liver injury is bleeding However bleeding can stop spontaneously
Ans: F

492- Acute appendicitis always causes a liver abscess if it happents infection gets to the liver
through lymphatics
Ans: F

493-An amoebic liver abscess characteristically contains clear fluid


Ans: F
494- The normal portal pressure is 8-12cm water
Ans: T

495- Schistosomiasis carries a worse prognosis than post-hepatitis cirrhosis


Ans: F
260

496- Viral hepatitis A resolves spontaneously without ill effects.One the hand types B and C can
cause liver cirrhosis and predispose to the development of hepatocellular carcinoma
Ans: T

497- Haemangiomas of the liver are common findings on imaging studies (CT or ultrasound)that
are done for other indications
Ans: T

498- carcinoembryonic abtigen is elevated in patients with GIT and breast cancer
Ans: T

499- The tumour marker of HCC is alpha fetoprotein


Ans: T

500- The blood supply of HCC is the hepatic artery rather than the portal vein
Ans: T

501- Venous drainage of the testis is systemic haematogenous spread of testicular cancer goes to
the lungs rather than the liver
Ans: T

View publication stats

You might also like